Monday, August 20, 2018

Guest Post: Tam Hunt questions Carlo Rovelli about the Nature of Time

Tam Hunt.
[Tam Hunt, photo on the right, is a renewable energy lawyer in Hawaii and an “affiliate” in the Department of Psychological and Brain Sciences at UC Santa Barbara. (Scare quotes his, not mine, make of this what you wish.) He has also published some papers about philosophy and likes to interview physicists. The below is an email interview he conducted with Carlo Rovelli about the Nature of Time. Carlo Rovelli is director of the quantum gravity group at Marseille University in France and author of several popular science books.]

TH:Let me start by asking why discussions about the nature of time should matter to the layperson?

CR: There is no reason it “should” matter. People have the right to be ignorant, if they wish to be. But many people prefer not to be ignorant. Should the fact that the Earth is not flat matter for normal people? Well, the fact that Earth is a sphere does not matter during most of our daily lives, but we like to know.

TH: Are there real-world impacts with respect to the nature of time that we should be concerned with?

CR: There is already technology that has been strongly impacted by the strange nature of time: the GPS in our cars and telephones, for instance.
Carlo Rovelli.

TH: What inspired you to make physics and the examination of the nature of time a major focus of your life's work?

CR: My work on quantum gravity has brought me to study time. It turns out that in order to solve the problem of quantum gravity, namely understanding the quantum aspects of gravity, we have to reconsider the nature of space and time. But I have always been curious about the elementary structure of reality, since my adolescence. So, I have probably been fascinated by the problem of quantum gravity precisely because it required rethinking the nature of space and time.

TH: Your work and your new book continue and extend the view that the apparent passage of time is largely an illusion because there is no passage of time at the fundamental level of reality. Your new book is beautifully and clearly written -- even lyrical at times -- and you argue that the world described by modern physics is a “windswept landscape almost devoid of all trace of temporality.” (P. 11). How does this view of time pass the “common sense” test since everywhere we look in our normal waking consciousness there is nothing but a passage of time from moment to moment to moment?

CR: Thanks. No, I do not argue that the passage of time is an illusion. “Illusion” may be a misleading word. It makes it seem that there is something wrong about our common-sense views on time. There is nothing wrong with it. What is wrong is to think that this view must hold for the entire universe, or that it is valid at all scales and in all situations. It is like the flat Earth: Earth is almost perfectly flat at the scale of most of our daily life, so, there is nothing wrong in considering it flat when we build a house, say. But on larger scales the Earth just happens not to be flat. So with time: as soon as we look a bit farther than our myopic eyes allow, we see that it works differently from what we thought.

This view passes the “common sense” test in the same way in which the fact that the Earth rotates passes the “common sense” view that the Earth does not move and the Sun moves down at sunset. That is, “common sense” is often wrong. What we experience in our “normal waking consciousness” is not the elementary structure of reality: it is a complex construction that depends on the physics of the world but also on the functioning of our brain. We have difficulty in disentangling one from the other.

“Time” is an example of this confusion; we mistake for an elementary fact about physics what is really a complex construct due to our brain. It is a bit like colors: we see the world in combinations of three basic colors. If we question physics as to why the colors we experience are combination of three basic colors, we do not find any answer. The explanation is not in physics, it is in biology: we have three kinds of receptors in our eyes, sensible to three and only three frequency windows, out of the infinite possibilities. If we think that the three-dimensional structure of colors is a feature of reality external to us, we confuse ourselves.

There is something similar with time. Our “common sense” feeling of the passage of time is more about ourselves than the physical nature of the external world. It regards both, of course, but in a complex, stratified manner. Common sense should not be taken at face value, if we want to understand the world.

TH: But is the flat Earth example, or similar examples of perspectival truth, applicable here? It seems to me that this kind of perspectival view of truth (that the Earth seems flat at the human scale but is clearly spherical when we zoom out to a larger perspective) isn’t the case with the nature of time because no matter what scale/perspective we use to examine time there is always a progression of time from now to now to now. When we look at the astronomical scale there is always a progression of time. When we look at the microscopic scale there is always a progression of time.

CR: What indicates that our intuition of time is wrong is not microscopes or telescopes. It’s clocks. Just take two identical clocks indicating the same time and move them around. When they meet again, if they are sufficiently precise, they do not indicate the same time anymore. This demolishes a piece of our intuition of time: time does not pass at the same “rate” for all the clocks. Other aspects of our common-sense intuition of time are demolished by other physics observations.

TH: In the quote from your book I mentioned above, what are the “traces” of temporality that are still left over in the windswept landscape “almost devoid of all traces of temporality,” a “world without time,” that has been created by modern physics?

CR: Change. It is important not to confuse “time” and “change.” We tend to confuse these two important notions because in our experience we can merge them: we can order all the change we experience along a universal one-dimensional oriented line that we call “time.” But change is far more general than time. We can have “change,” namely “happenings,” without any possibility of ordering sequences of these happenings along a single time variable.

There is a mistaken idea that it is impossible to describe or to conceive change unless there exists a single flowing time variable. But this is wrong. The world is change, but it is not [fundamentally] ordered along a single timeline. Often people fall into the mistake that a world without time is a world without change: a sort of frozen eternal immobility. It is in fact the opposite: a frozen eternal immobility would be a world where nothing changes and time passes. Reality is the contrary: change is ubiquitous but if we try to order change by labeling happenings with a time variable, we find that, contrary to intuition, we can do this only locally, not globally.

TH: Isn’t there a contradiction in your language when you suggest that the common-sense notion of the passage of time, at the human level, is not actually an illusion (just a part of the larger whole), but that in actuality we live in a “world without time”? That is, if time is fundamentally an illusion isn’t it still an illusion at the human scale?

CR: What I say is not “we live in a world without time.” What I say is “we live in a world without time at the fundamental level.” There is no time in the basic laws of physics. This does not imply that there is no time in our daily life. There are no cats in the fundamental equations of the world, but there are cats in my neighborhood. Nice ones. The mistake is not using the notion of time [at our human scale]. It is to assume that this notion is universal, that it is a basic structure of reality. There are no micro-cats at the Planck scale, and there is no time at the Planck scale.

TH: You argue that time emerges: “Somehow, our time must emerge around us, at least for us and at our scale.” As such, how do you reconcile the notion of emergence of time itself with the fact that the definition of emergence necessarily includes change over time? That is, how is it coherent to argue that time itself emerges over time?

CR: The notion of “emergence” does not always include change over time. For instance we say that if you look at how humans are distributed on the surface of the Earth, there are some general patterns that “emerge” by looking at a very large scale. You do not see them at the small scale, you see them looking at the large scale. Here “emergence” is related to the scale at which something is described. Many concepts we use in science emerge at some scale. They have no meaning at smaller scales.

TH: But this kind of scale emergence is a function solely of an outside conscious observer, in time, making an observation (in time) after contemplating new data. So aren’t we still confronted with the problem of explaining how time emerges in time?

CR: There is no external observer in the universe, but there are internal observers that interact with one another. In the course of this interaction, the temporal structure that they ascribe to the rest may differ. I think that you are constantly misunderstanding the argument of my book, because you are not paying attention to the main point: the book does not deny the reality of change: it simply confronts the fact that the full complexity of the time of our experience does not extend to the entire reality. Read the book again!

TH: I agree that common sense can be a faulty guide to the nature of reality but isn’t there also a risk of unmooring ourselves from empiricism when we allow largely mathematical arguments to dictate our views on the nature of reality?

CR: It is not “largely mathematical arguments” that tell us that our common sense idea of time is wrong. It is simple brute facts. Just separate two accurate clocks and bring them back together and this shows that our intuition about time is wrong. When the GPS global positioning system was first mounted, some people doubted the “delicate mathematical arguments” indicating that time on the GPS satellites runs faster than at sea level: the result was that the GPS did not work [when it was first set up]. A brute fact. We have direct facts of evidence against the common-sense notion of time.

Empiricism does not mean taking what we see with the naked eye as the ultimate reality. If it was so, we would not believe that there are atoms or galaxies, or the planet Uranus. Empiricism is to take seriously the delicate experience we gather with accurate instruments. The idea that we risk unmooring “ourselves from empiricism when we allow largely mathematical arguments to dictate our views on the nature of reality” is the same argument used against Galileo when we observed with the telescope, or used by Mach to argue against the real existence of atoms. Empiricism is to base our knowledge of reality on experience, and experience includes looking into a telescope, looking into an electronic microscope, where we actually can see the atoms, and reading accurate clocks. That is, using instruments.

TH: I’m using “empiricism” a little differently than you are here; I’m using the term to refer to all methods of data gathering, whether directly with our senses or indirectly with instruments (but still mediated through our senses because ultimately all data comes through our human senses). So what I’m getting at is that human direct experience, and the constant passage of time in our experience, is as much data as are data from experiments like the 1971 Hafele-Keating experiment using clocks traveling opposite directions on airplanes circling the globe. And we cannot discount either category of experience. Does this clarification of “empiricism” change your response at all?

CR: We do not discount any category of experience. There is no contradiction between the complex structure of time and our simple human experience of it. The contradiction appears only if we extrapolate our experience and assume it captures a universal aspect of reality. In our daily experience, the Earth is flat and we take it to be flat when we build a house or plan a city; there is no contradiction between this and the round Earth. The contradiction comes if we extrapolate our common-sense view of the flat Earth beyond the small region where it works well. So, we are not discounting our daily experience of time, we are just understanding that it is an approximation to a more complicated reality.

TH: There have been, since Lorentz developed his version of relativity, which Einstein adapted into his Special Theory of Relativity in 1905, interpretations of relativity that don’t render time an illusion. Isn’t the Lorentz interpretation still valid since it’s empirically equivalent to Special Relativity?

CR: I think you refer here to the so called neo-Lorentzian interpretations of Special Relativity. There is a similar case in the history of science: after Copernicus developed his systems in which all planets turn around the Sun and the Earth moves, there were objections similar to those you mention: “the delicate mathematical arguments” of Copernicus cannot weight as much as our direct experience that the Earth does not move.

So, Tycho Brahe developed his own system, where the Earth is at the center of the universe and does not move, the Sun goes around the Earth and all the other planets rotate around the Sun. Nice, but totally useless for science and for understanding the world: a contorted and useless attempt to save the common sense-view of a motionless Earth, in the face of overwhelming opposite evidence.

If Tycho had his way, science would not have developed. The neo-Lorentzian interpretations of Special Relativity do the same. They hang on to the wrong extrapolation of a piece of common sense.

There is an even better example: the Moon and the Sun in the sky are clearly small. When in antiquity astronomers like Aristarchus come out with an estimate of the size of the Moon and the Sun, it was a surprise, because it turned out that the Moon is big and the Sun even bigger than the Earth itself. This was definitely the result of “largely mathematical arguments.” Indeed it was a delicate calculation using geometry, based on angles under which we see these objects. Would you say that the fact that the Sun is larger than the Earth should not be believed because it is based on a “largely mathematical argument“ and contradicts our direct experience?

TH: But in terms of alternative interpretations of the Lorentz transformations, shouldn’t we view these alternatives, if they’re empirically equivalent as they are, in the same light as the various different interpretations of quantum theory (Copenhagen, Many Worlds, Bohmian, etc.)? All physics theories have two elements: 1) the mathematical formalisms; 2) an interpretive structure that maps those formalisms onto the real world. In the case of alternatives to Special Relativity, some have argued that we don’t need to adopt the Einstein interpretation of the formalisms (the Lorentz transformations) in order to use those formalisms. And since Lorentz’s version of relativity and Einstein’s Special Relativity are thought to be empirically equivalent, doesn’t a choice between these interpretations come down to a question of aesthetics and other considerations like explanatory power?

CR: It is not just a question of aesthetics, because science is not static, it is dynamic. Science is not just models. It is a true continuous process of better understanding reality. A better version of a theory is fertile: it takes us ahead; a bad version takes no part. The Lorentzian interpretation of special relativity assumes the existence of entities that are unobservable and undetectable (a preferred frame). It is contorted, implausible, and in fact it has been very sterile.

On the other hand, realizing that the geometrical structure of spacetime is altered has led to general relativity, to the prediction of black holes, gravitational waves, the expansion of the universe. Science is not just mathematical models and numerical predictions: it is developing increasingly effective conceptual tools for making sense and better understanding the world. When Copernicus, Galileo and Newton realized that the Earth is a celestial body like the ones we see in the sky, they did not just give us a better mathematical model for more accurate predictions: they understood that man can walk on the moon. And man did.

TH: But doesn’t the “inertial frame” that is the core of Einstein’s Special Relativity (instead of Lorentz’s preferred frame) constitute worse “sins”? As Einstein himself states in his 1938 book The Evolution of Physics, inertial frames don’t actually exist because there are always interfering forces; moreover, inertial frames are defined tautologically (p. 221). Einstein’s solution, once he accepted these issues, was to create the general theory of relativity and avoid focusing on fictional inertial frames. We also have the cosmic frame formed by the Cosmic Microwave Background that is a very good candidate for a universal preferred frame now, which wasn’t known in Einstein’s time. When we add the numerous difficulties that the Einstein view of time results in (stemming from special not general relativity), the problems in explaining the human experience of time, etc., might it be the case that the sins of Lorentzian relativity are outweighed by Special Relativity’s sins?

CR: I do not know what you are talking about. Special Relativity works perfectly well, is very heavily empirically supported, there are no contradictions with it in its domain of validity, and has no internal inconsistency whatsoever. If you cannot digest it, you should simply study more physics.

TH: You argue that “the temporal structure of the world is not that of presentism,” (p. 145) but isn’t there still substantial space in the scientific and philosophical debate for “presentism,” given different possible interpretations of the relevant data?

CR: There is a tiny minority of thinkers who try to hold on to presentism, in the contemporary debate about time. I myself think that presentism is de facto dead.

TH: I’m surprised you state this degree of certainty here when in your book you acknowledge that the nature of time is one of physics’ last remaining large questions. Andrew Jaffe, in a review of your book for Nature, writes that the issues you discuss “are very much alive in modern physics.”

CR: The debate on the nature of time is very much alive, but it is not a single debate about a single issue, it is a constellation of different issues, and presentism is just a rather small side of it. Examples are the question of the source of the low initial entropy, the source of our sense of flow, the relation between causality and entropy. The non-viability of presentism is accepted by almost all relativists.

TH: Physicist Lee Smolin (another loop quantum gravity theorist, as you know) argued views quite different than yours, in his book, Time Reborn, for example. In an interview with Smolin I did in 2013, he stated that “the experience we have of time flowing from moment into moment is not an illusion but one of the deepest clues we have as to the nature of reality.” Is Smolin part of the tiny minority you refer to?

CR: Yes, he is. Lee Smolin is a dear friend for me. We have collaborated repeatedly in the past. He is a very creative scientists and I have much respect of his ideas. But we disagree on this. And he is definitely in the minority on this issue.

TH: I’ve also been influenced by Nobel Prize winner Ilya Prigogine’s work and particularly his 1997 book, The End of Certainty: Time, Chaos and the New Laws of Nature, which opposes the eternalist view of time as well as reversibility in physics. Prigogine states in his book that reversible physics and the notion of time as an illusion are “impossible for me to accept” He argues that whereas many theories of modern physics include a reversible t term, this is an empirical mistake because in reality the vast majority of physical processes are irreversible. How do you respond to Prigogine and his colleagues’ arguments that physics theories should be modified to include irreversibility?

CR: That he is wrong, if this is what he writes. There is no contradiction between the reversibility of the laws that we have and the irreversibility of the phenomena. All phenomena we see follow the laws we have, as far as we can see. The surprise is that these laws allow also other phenomena that we do not see. So, something may be missing in our understanding --and I discuss this at length in my book-- but something missing does not mean something wrong.

I do not share the common “block universe” eternalist view of time either. What I argue in the book is that the presentist versus eternalist alternative is a fake alternative. The universe is neither evolving in a single time, nor static without change. Temporality is just more complex than either of these naïve alternatives.

TH: You argue that “the world is made of events, not things” in part II of your book. Alfred North Whitehead also made events a fundamental feature of his ontology, and I’m partial to his “process philosophy.” If events—happenings in time—are the fundamental “atoms” of spacetime (as Whitehead argues), shouldn’t this accentuate the importance of the passage of time in our ontology, rather than downgrade it as you seem to otherwise suggest?

CR: “Time” is a stratified notion. The existence of change, by itself, does not imply that there is a unique global time in the universe. Happenings reveal change, and change is ubiquitous, but nothing states that this change should be organized along the single universal uniform flow that we commonly call time. The question of the nature of time cannot be reduced to a simple “time is real”, “time is not real.” It is the effort of understanding the many different layers giving rise to the complex phenomenon that we call the passage of time.

294 comments:

  1. hi:

    i don't understand the use of the word "roast" in the title. according to the urban dictionary, and common english usage, roast is defined as "To humorously mock or humiliate someone with a well-timed joke, diss or comeback."

    naive theorist

    ReplyDelete
  2. Hardly a roast. I did appreciate his noting that the CMB amounts to a "preferred frame". I think this is right tho the CMB is not the frame itself but a proxy for it. Then again I am a Smolin fan..

    ReplyDelete
  3. Merriam-Webster: "roast: to subject to severe criticism or ridicule"

    I didn't know about the colloquial use, sorry. Is the revision any better? If not, please make a suggestion.

    ReplyDelete
  4. bee:

    'grill' is perfect.

    naive theorist

    ReplyDelete
  5. When the GPS global positioning system was first mounted, some people doubted the “delicate mathematical arguments” indicating that time on the GPS satellites runs faster than at sea level: the result was that the GPS did not work [when it was first set up]

    Is that true? I find it very surprising, considering GR was established science for decades at that time. A quick look at the GPS entry in Wikipedia didn't help.

    ReplyDelete
  6. Hello Dr. H. This interview neither severe or ridiculing (colloquially taken to be in good natured fun)

    So nothing wrong with your definition.

    Be well.

    ReplyDelete
  7. Yes, grill is perfect. Implies interrogation!!

    ReplyDelete
  8. I was surprised by CR's characterization that "The Lorentzian interpretation of special relativity assumes the existence of entities that are unobservable and undetectable (a preferred frame)." Lorentz's approach in a field theory context can be presented as no more than that the differential equations of the theory must be locally Lorentz invariant --- as Maxwell's equations are, for prime example. Einstein differs only in taking an empirical principle approach instead of a model building approach, but both discussions derive from the symmetry group of Maxwell's equations.
    Given that commonality, how one specifies the initial and/or boundary conditions or other constraints is not different for Einstein than it is for Lorentz.

    ReplyDelete
  9. A Foucault pendulum swings inside a tall Einstein elevator located at 45° latitude, containing an observer. The suspensory filament's ceiling anchor is fixed, does not rotate. What is the observer's model of space and time?

    ReplyDelete
  10. They should make this thread required reading before you watch Interstellar (the movie). It would definitely give you more room to suspend disbelief.

    Also I especially enjoy the analogies Rovelli uses, such as the 3 colors out of the spectrum for human eyes not being the whole color story. It makes elegant sense when replacing "color" with "time".

    Fascinating stuff. Thanks!

    ReplyDelete
  11. This isn’t so much a grilling as it is a persistent questioning by someone who has only a superficial grasp of the material w/ frustrated responses by an expert. CR is very clear in his book and his answers here; the interviewer does not seem to understand the deeper point. It reminds me of the TV news interview of Jordan Peterson where the interviewer continued to make naive generalizations about Peterson’s arguments that were absurd if you understood what he was saying.

    ReplyDelete
  12. "A big ball of wibbly wobbly, timey wimey stuff." The definition of "time" given by the tenth Doctor Who couldn't be more precise...

    ReplyDelete
  13. I have a very hard time separating the idea of change from time. Seems change arises as a consequence of time and can't be discussed without reference to two points in time. Unless one posits that state-0 and state-1 are separated by something other than time.

    But then I believe (as an article of faith) that time and space are fundamental, not in any way emergent. To me, thermodynamics arises as a consequence of particle behavior in time, not the other way around.

    ReplyDelete
  14. Given phenomena like the relativistic length contraction that occurs at large differences in velocity between observers, I sometimes wonder why physicists who deny time in some sense, because it is relative, do not also deny distance in a similar sense. Isn't it just as relative?

    ReplyDelete
  15. It seems that in the conversation, Rovelli kept pointing out the importance of context when evaluating the meaning of time -- i.e. local vs. global/universal, micro vs. macro, detailed reality vs. experiential approximation (what Rovelli, in his Order of Time, repeatedly refers to how we as biological organisms "blur" the actual details of our observed world in order to move on from moment to moment), etc. Yet, the interviewer didn't latch onto that context differentiation, rather kept asserting a more general context. So, at times it seemed the two were discussing different things under the same labels, though I felt Rovelli knew that and kept correcting the basis of the current topic.

    I was a bit surprised that the notion of entropy as a driver of change, and thus an enabler of the perception of time, didn't come up sooner, and was never explicitly mentioned except by examples.

    In any case, a good debate indeed. Thanks for posting it!

    ReplyDelete
  16. Maybe I'm missing something—but isn't this just a standard interview? "Grill" implies that there were uncomfortable or difficult questions the subject had trouble answering, e.g. "the reporters grilled the politician at the press conference about his affairs" or "the defense grilled the witness on cross-examination."

    Were there things you think Carlo got wildly wrong?

    ReplyDelete
  17. The main difficulty with this interview is that Mr. Hunt is not a working scientist and sometimes he gets a wrong notion how the discovery process works. Which would not be a problem by itself but unfortunately as an interviewer he is pushing his own opinions/interpretations too much - and Dr. Rovelli gets irritated by it so they both come across a bit more pompous and prickly than they probably are in daily life.

    For a very instructive example how to interview a physicist, I would recommend Natalie Wolchover (although it is not an entirely fair comparison: she is a physicist who just did not finish her thesis). A great example how to give an illuminating interview about theoretical research would be probably anything from Feynman.

    https://www.quantamagazine.org/edward-witten-ponders-the-nature-of-reality-20171128/

    ReplyDelete
  18. Bee

    do these claims on time require LQG to be true, and is it shared in other QG such as strings ?

    ReplyDelete
  19. Time is just like the rotation of the earth. We judge time by how many times the earth has rotated on its axis. Events happen that we can mark by how many rotations. Time is no more mysterious then this. As far as time rate changing, yes it does. Gravity and velocity just change the rate of our measurement tools.

    ReplyDelete
  20. Didn't John Wheeler once say something like "physics can explain everything but the 'now' "...?

    ReplyDelete
  21. Neo-Lorentzian wiki: https://en.m.wikipedia.org/wiki/Lorentz_ether_theory

    ReplyDelete
  22. As someone who shares the dissenting views of Louis de Broglie, John Bell, Ilya Prigogine, and Lee Smolin on the nature time, its wonderful to see a post on this topic! I'm not surprised Rovelli apparently hasn't bothered reading Prigogine. As Upton Sinclair said, "It's difficult to get a man to understand something when his salary depends upon his not understanding it."

    There are two sides to Rovelli. He had the courage to criticize string theory at the peak of its dominance over theoretical physics. But I think he's been no less guilty of similar sins in hyping his own work. For ages he claimed not only that time is an illusion, but future work would do away with the need for time entirely. That brings to mind Frank Wilczek's comment on string theory, "The theory is still promising...and promising."

    ReplyDelete
  23. I had also heard somewhere, long prior to this, that GPS (first established for military purposes) initially did not include General Relativity corrections, and that they had to be added for accuracy. However, the first couple Google hits I read now say that the existing system satellites went into orbit with relativity corrections built in. Perhaps there was an earlier test using a different satellite which established the need, when the system was being developed, or perhaps the story is apocryphal. I would bet on the story being true in spirit because most engineering development proceeds by trial and error, but it may well be exaggerated. (Many it was simply that the first paper or computer calculation did not include the relativity effects and someone noticed this.)

    "Grilling" is or used to be used to describe police interrogations of a suspect. (Also known as "sweating" the suspect, perhaps because a small room and a bright light were used; heat seems to have been a factor, and a grill is also the word for hot metal used to cook meat.) It seems very appropriate because the interview seemed to have an adversarial tone. Philosophers seem to get emotional quickly when someone else has a different view.

    ReplyDelete
  24. The interview stay at the level of general speaking so that no thoughts about the nature of time has been set under an analysis... :(

    ReplyDelete
  25. I took it the use of 'roast' and 'grill' were made with some humor in mind. 'interview' is correct but bland. I think Dr. H. might get the impression we are a humorless bunch!

    ReplyDelete
  26. Carlo Rovelli is a sage and he will be viewed as such in the future. I have no doubt that his view of time presented here will win out as will his interpretation of QM.

    ReplyDelete
  27. I read Dr. Rovelli's discussion of time after reading his "Reality Is Not What It Seems"

    I was in hope that the discussion would shed further light of his discussion of Quantum Gravity. Sad to say I found no illumination on that theory.

    The philosophical content was interesting and prompted me to pass it on to an itinerant philosophy investigator.

    ReplyDelete
  28. "Carlo Rovelli is director of the quantum gravity group at Marseille University in France and author of several popular science books."

    True, but you probable meant that he is the author of several popular-science books. Or perhaps several popular popular-science books. :-)

    ReplyDelete
  29. Somehow I understood GR replaces space and time with spacetime. These kinds of discussions about the nature of time, distinct from space, implicitly premise the falsity of GR, That seems empirically dubious so far as I know, but it appears to be the universal judgment of physicists. On the one hand, this is just the old irreconcilability of GR and QM appearing in a specific form. But on the other the the old problem of precisely how quantum processes, in which there are not exactly entities, things, observables, separate from the measurement process, but we don't really understand what measurement is? What Rovelli is arguing is that it's not a real problem at all, that it's just a misunderstanding to think that we should worry about such questions? This sounds a lot like the Copenhagen interpretation to me, there's quantum, there's everyday, and science isn't about reality, it's about predicting experimental results.

    ReplyDelete
  30. "I think that you are constantly misunderstanding the argument of my book, because you are not paying attention to the main point: the book does not deny the reality of change: it simply confronts the fact that the full complexity of the time of our experience does not extend to the entire reality. Read the book again!"

    I agree with Carlo here!

    "I do not know what you are talking about. Special Relativity works perfectly well, is very heavily empirically supported, there are no contradictions with it in its domain of validity, and has no internal inconsistency whatsoever. If you cannot digest it, you should simply study more physics."

    I agree with Carlo here!

    "How do you respond to Prigogine and his colleagues’ arguments that physics theories should be modified to include irreversibility?

    CR: That he is wrong"

    Carlo is right again!


    ReplyDelete
  31. I, too, was interested by the remark about GPS and the predictions of relativity. I ultimately found http://www.leapsecond.com/history/1978-PTTI-v9-NTS-2.pdf which may be as close as we'll get to the origin of this story. NTS-2 was launched with the provision for selecting a frequency that took into account the effect of GR+SR. Initially, it was brought up in the mode which did NOT account for GR+SR, which yielded a systematic frequency offset of +442.5 pp10^12 which was comparable to the prediction of +445.0 pp10^12.

    A modern author summarizes further at http://www.leapsecond.com/history/Ashby-Relativity.htm : "At the time of launch of the first NTS-2 satellite (June 1977), which contained the first Cesium clock to be placed in orbit, there were some who doubted that relativistic effects were real. A frequency synthesizer was built into the satellite clock system so that after launch, if in fact the rate of the clock in its final orbit was that predicted by GR, then the synthesizer could be turned on bringing the clock to the coordinate rate necessary for operation. The atomic clock was first operated for about 20 days to measure its clock rate before turning on the synthesizer. The frequency measured during that interval was +442.5 parts in 10^12 faster than clocks on the ground; if left uncorrected this would have resulted in timing errors of about 38,000 nanoseconds per day. The difference between predicted and measured values of the frequency shift was only 3.97 parts in 10^12, well within the accuracy capabilities of the orbiting clock. This then gave about a 1% validation of the combined motional and gravitational shifts for a clock at 4.2 earth radii."

    ReplyDelete
  32. "So, Tycho Brahe developed his own system, where the Earth is at the center of the universe and does not move, the Sun goes around the Earth and all the other planets rotate around the Sun. Nice, but totally useless for science and for understanding the world: a contorted and useless attempt to save the common sense-view of a motionless Earth, in the face of overwhelming opposite evidence."

    I have to disagree with Carlo here, at least somewhat. Tycho's model was not an attempt to save the appearances by a hidebound reactionary who couldn't accept that the Earth moved, but the result of the fact that he didn't know about wave optics. (Considering that Tycho never married his girlfriend and mother of his children, and that in the 16th century, I don't think that he was too set in his ways to accept something new. He certainly did accept many new things, some of which he discovered, such as the nature of comets, the fact of change in the heavens (novae), etc.)

    Wave optics? Stars have an angular size of about a minute of arc. Tycho reasoned that this means that they are about 30 times as far away as the Sun, which has an angular size of about 30 minutes of arc. In that case, he should see aberration. He didn't. Thus, his model. What was the problem? The apparent angular size of a star is not its true angular size, but is basically caused by diffraction on the pupil. (Of course, assuming that stars are 30 times as far away as the Sun means assuming that they are much fainter than the Sun in absolute terms. More correct would have been to assume that they are about as bright as the Sun but much more than 30 times as far away. However, in such a case they would have to be much, much, much larger than the Sun in order to explain their apparent angular size, so I don't think that we can fault Tycho for this.)
    javascript:void(0)
    Note: I have read most of Carlo's popular books (in English translation) and recommend them all.

    ReplyDelete
  33. Change does not need time.
    A variable may change with time, or it may change with position (space), or both.
    The 4 dimensional energy-momentum vector captures the near equivalence of time and space.
    (Minkowsky took it a little too far, but only a little.)

    Emergence does not need time.
    Take numbers, and the whole of mathematics emerges.
    The 'axis' along which this happens is time nor space, it is logic.

    Most physicists believe -I believe- that the whole of physics ultimately somehow emerges in a very similar way from, well, pure inevitable logic.
    Ergo, both time and space are probably emergent phenomena.

    Time is in essence causality, which is close to logic, so time might be more fundamental than space.

    ReplyDelete
  34. I just saw this; things have been a bit crazy lately. CR's comments about time and clocks need a "grilling", defects in measuring devices do not undermine the concept of time. More later.

    sean s.

    ReplyDelete
  35. Oops! I just discovered that English isn't Dutch. Of course I meant to say
    "neither time nor space".

    ReplyDelete
  36. Tycho reasoned that this means that they are about 30 times as far away as the Sun, which has an angular size of about 30 minutes of arc. In that case, he should see aberration.

    I think you meant parallax rather than aberration. Tycho was looking for parallax. Over 150 years later Bradley was still looking in vain for parallax, when he discovered aberration instead. (Stellar parallax was finally detected in 1839.)

    ReplyDelete
  37. I was a bit disappointed in Rovelli's interview here. The so called mystery of time really involves why it is general relativity for any arbitrary spacetime defines in ADM formalism energy as just a constraint NH = 0, there is the momentum constraint N^iH_i = 0 as well. Here N is a lapse function and N^i a shift function. The Wheeler DeWitt equation HΨ[g] = 0 has not reference to time, and we can represent this as a type of Schrödinger equation

    HΨ[g] = i∂Ψ[g]/∂t = 0.

    In general this means there is no Killing vector K_t = k∂/∂t that defines a Nötherian conservation law. In effect spacetimes in general have no natural bounding region one can define a Gaussian surface to define mass-energy. It also means we have difficulty understanding where time comes into the picture.

    The emergence of time may then be seen quantum mechanically. The Page-Wooters theorem is that for two Hilbert spaces H_1 and H_2 an entanglement of their states is then given by the product states of these Hamiltonians H_1⊗I_2 + I_1⊗H_2. A state then of the form

    |Ψ> = sum_{ij}c_{ij}(|ψ_i>|φ_j> + |ψ_j>|φ_i>),

    |ψ_i> ∈ H_1 and |φ_j> ∈ H_2 is an entanglement of state from these two Hamilton|Ψ>ians. Now we take an arbitrary state of the for |χ> = sum_ka_i|ψ_i> ∈ H_1 and project onto |Ψ> with

    = sum_{ijk}a*_k c_{ij}(|φ_j> + |φ_i>),

    = sum_{ijk}a*_k c_{ij}(δ_{ik}|φ_j> + δ_{jk}|φ_i>)

    = sum_{ij}(a*_i c_{ij}|φ_j> + a*_jc_{ij}|φ_i>).

    Since c_{ij} = a*_ia_j this is then written as

    = sum_{ij}(|a_i|^2a_j|φ_j> + |a_j|^2a_i|φ_i>) = 2sum_{ij}|a_i|^2a_j|φ_j>

    The matrix element c_{ij} = a*_ia_j is a relative phase term c_{ij} = exp(iθ_i)exp(-iθ'_j) and the difference in this relative phase θ_i - iθ'_j = ωt. This projection does here then is a way of measuring the phase of one system relative to another. Above the phase with θ'_j is chosen.

    This is a start as something, but a more general gauge-like theory is needed to make this work in quantum gravity. The Raamsdonk argument and other have illustrated how spacetime may be an emergent phenomena from entanglement, which would then in conjunction with Wooters-Page mean there is some global/local structure for states and entanglements that give rise to the local appearance of time.

    ReplyDelete
  38. To Jeff Epler: thanks for the excellent, informative comment!

    The truth, that some technical people involved did not believe in relativistic time effects in 1977, is stranger than I imagined. I can vouch for the fact that they were taught in physics classes in the 1960's (and a staple of science-fiction stories then, such as the story Poul Anderson's "Tau Zero" was based on).

    As Dr. Rovelli mentioned, precise clocks which are synchronized and then undergo different relativistic histories will measure different elapsed times, and the GPS satellites prove this.

    ReplyDelete
  39. An illuminating interview. I have seen too many physics articles mention how time is an illusion, and then talk about SR, rivers, arrow of time etc. Yet the articles I read never seem to me to actually show that time does not exist. The one exception was Smolin's description of Julian Barbour's idea of time. I don't count the block Universe as that just seems to me a particular view point.

    This interview may explain my confusion, I thought time and change were more or less synonymous. It appears that physicists think that when people talk about time they are explicitly arguing for a one dimensional ordering where time always progresses at the same rate, while change is something that happens because of time. I thought that stuff just happened and that time was simply the result of everything we see changing in its own "time".

    ReplyDelete
  40. "I think you meant parallax rather than aberration."

    Right. (However, in practice aberration is the larger effect.)

    As usual, the story is more complicated, and Tycho did have other reasons for believing in a stationary Earth, though the weight he put on different arguments changed with time. He was an innovative guy who shouldn't be remembered for not going all the way with Copernicanism, especially if he had at least some good reasons. Galileo got some stuff wrong too (like the tides), but is generally considered to be a good scientist.

    ReplyDelete
  41. I wholeheartly agree that Rovelli is wrong on Brahe, as detailed here, but a little correction to the correction is warranted:

    Considering that Tycho never married his girlfriend and mother of his children, and that in the 16th century, I don't think that he was too set in his ways to accept something new.

    This is in fact false since Tycho did marry Kirsten Jørgensdatter, just not in a way we are used to: the fact that he was a nobleman from one of the most prestigious houses of Denmark and she was a commoner was an obstacle for a regular blessing, but they indeed were married with a morganatic union.

    ReplyDelete
  42. @David Bonda, obviously I disagree with your take here. I think Carlo's positions are fatally flawed, based on the pretty simple logical critique I raise in my questioning about emergence. How does time emerge in time? How does structural emergence emerge outside of time (as Carlo raised in response to my follow up question)?

    The obvious answer in both cases is that they don't. So this is the flaw at the heart of any notion of the emergence of time from a deeper reality that exists outside of time. We are pretty much stuck with the passage of time as the fundamental container of existence, so all physical theories should overtly acknowledge that. We have instead, in the last few hundred years, fallen prey to the romantic and empirically-ungrounded notion that fundamental reality is outside of time.

    Check out Prigogine's book The End of Certainty. Or Max Jammer's book Concepts of Simultaneity (the last chapter in particular, on the conventionality of simultaneity debate). They're both quite enlightening.

    ReplyDelete
  43. @Philip Helbig, Carlo and I were missing each other a bit here, yes. I was suggesting (as I just described more overtly to Bonda) in my questions to Carlo that his notion of the emergence of time in time is incoherent. He says I didn't understand his point but I don't think he understood my implied critique of his notion of emergence. Again: how does time emerge in time? How does structural emergence occur outside of time? We need to confront these key points and accept that time (as change) is the fundamental container of our existence.

    Summing up on this point: Carlo is saying time as change is real at the human level but not real at the fundamental level. I was suggesting that this solution is incoherent. He disagreed or misunderstood my point about incoherence. Hence the impasse.

    Regarding Prigogine, have you read his book? He makes a rather compelling case. And Carlo's response on this is rather telling in that he acknowledges that empirically we never see t reverse, but he still asserts that the equations are correct and may reveal new features of reality. So this is truly putting the cart before the horse and is a nice example of what I wrote about in terms of falling prey to "largely mathematical arguments." This is the case because if the equations we develop to be able to predict phenomena in the natural world go beyond what we observe in the natural world, and then are used to say something like "see, nature is like this because of these equations," we have then forgotten that the equations were not actually modeling the natural world. They were, instead, going beyond the realities of the natural world. And that's why modern physics has gotten into trouble with respect to the nature of time as well as many other things (as Bee writes about in her recent book).

    ReplyDelete
  44. @zaalkort, change does need time. For example, any change in position of an object in the physical world must occur in time. Emergence also requires time, as I suggest to Carlo. Structural emergence is only an observational/epistemic kind of emergence. And any observational/epistemic act of course requires time. The logical emergence you refer to with respect to mathematics is an observational/epistemic act done by human observers in time. We agree on your point about time being more fundamental than space.

    ReplyDelete
  45. @Lawrence Crowell, how do we get past the initial objections about the emergence of time in time? These are foundational questions that have to be addressed before we go any further. And Carlo didn't and can't address them b/c the idea of emergence of time in time is incoherent.

    ReplyDelete
  46. Phillip Helbig said Tycho did have other reasons for believing in a stationary Earth… he shouldn't be remembered for not going all the way with Copernicanism, especially if he had at least some good reasons.

    B'Rat said I wholeheartly agree that Rovelli is wrong on Brahe…

    I don’t think it detracts from Tycho’s reputation as a great observational astronomer to cite his wacky cosmological model as an example (similar to neo-Lorentzianism) of how people try to retain old conceptual frameworks and beliefs for as long as they can – and sometimes longer. One of the implausible aspects of Tycho’s model was the non-rotating earth, which implied that the Sun and all the planets orbiting the sun (and all the stars) are all revolving in lock-step unison around the earth once per day. This is especially crazy since Tycho himself realized that (according to his model) the orbit of Mars intersects the Sun’s orbit, so he knew the old idea of spinning crystal spheres was untenable. The planets and comets etc. are all disconnected objects moving individually through space, and yet the whole formation (including the stars) is in rigid lock-step daily revolution around the earth. Really?

    Tycho’s reason for thinking the earth doesn’t rotate (in addition to biblical scripture) was that – in modern language – he denied the principle of inertia. For example, he argued that – contrary to popular belief (and common-place experience!) a ball thrown directly upward inside a uniformly moving ship would not rise and fall vertically. He implied that he knew this for a fact, and he used this “fact” to argue that the Earth’s surface can’t be moving from west to east, because if we throw a ball vertically upward it drops straight back down (more or less). Gassendi performed simple experiments that proved Tycho wrong (for anyone who didn’t already know it from their own experience). Even one of Tycho’s assistants (Longomontanus) soon proposed a semi-Tychonic model that at least admitted the rotation of the earth.

    Sure, the apparent optical diameter of stars was misleading, but it’s much easier to think of a physically plausible explanation for that than to rationalize Tycho’s model. (What is the apparent size of a candle’s flame from a mile away?) The importance of Copernicus was in pointing the way toward the principle of inertia and realizing that motions are most clearly described in terms of inertial coordinates, which enabled the laws of Newtonian physics to be recognized. Tycho’s model, like Ptolemy’s, was not based on inertia. It would never have been possible to discern the laws of celestial mechanics as long as the planetary positions were described in terms of an earth-centered system of coordinates. I think that’s what Rovelli meant when he said “If Tycho had his way, science would not have developed. The neo-Lorentzian interpretations of Special Relativity do the same.” The interviewer was obviously pushing a neo-Lorentzian approach, so I think Rovelli’s comment was on target.

    ReplyDelete
  47. It is important not to confuse “time” and “change.” We tend to confuse these two important notions because in our experience we can merge them: we can order all the change we experience along a universal one-dimensional oriented line that we call “time."

    ...change is ubiquitous but if we try to order change by labeling happenings with a time variable, we find that, contrary to intuition, we can do this only locally, not globally.

    “Time” is a stratified notion. The existence of change, by itself, does not imply that there is a unique global time in the universe. Happenings reveal change, and change is ubiquitous, but nothing states that this change should be organized along the single universal uniform flow that we commonly call time.


    It would appear from these comments that CR does not wholly subscribe to the expanding universe model and its singular 13.8Gy timeline. Too bad the interviewer didn't press the point.

    ReplyDelete
  48. @Phillip Helbig and others

    "
    "I do not know what you are talking about. Special Relativity works perfectly well, is very heavily empirically supported, there are no contradictions with it in its domain of validity, and has no internal inconsistency whatsoever. If you cannot digest it, you should simply study more physics."

    I agree with Carlo here!

    "


    >The situation on the validity of SR is more nuanced, as the experimental facts of the 1972 Hafele-Keating experiment on SR show :

    http://hyperphysics.phy-astr.gsu.edu/hbase/Relativ/airtim.html

    Concerning the verification of the existence of a locally preferred frame, this is an important test, aside from testing the sheer quantitative validity of the gravitational and kinetic time dilatation factors. Nobody could disagree on the latter, but the former - as a preferred frame test - does give surprising results :

    * C = control tower clock at earth surface
    * A = clock A flying east at speed v
    * B = clock B flying west at speed v
    * E = earth centered inertial frame located at the center of the earth

    •Using sheer SR:

    Viewed from C, A and B should indicate the exact same time on the clocks (same relative velocity used), thus differing both with the exact same amount from C.
    The experiment clearly shows this is not the case : 1 clock gained seconds, the other lost seconds compared to C.

    • Using the ECI as a locally preferred frame, you add up the speed of A to C, and you substract the speed of B from C, yielding thus the differing times on the clocks as experimentally established.

    An undisputable assessment for the preferred frame approach.

    Now, one might argue that SR is not suitable for rotating frame setups, but then of course you disqualify SR for almost all earth bound experiments because there's always a velocity vector parallel to the equator.

    One cannot ignore these results.


    Now, one might wonder why particle accelerators do not feature this problem, and give correct predictions concerning kinetic time dilatation, or the related relativistic mass increase predictions.

    Consider a particle accelerator on the equator shooting particles east and west, to get a nice parallel to the HK-setup:
    This is of course due to the relativistic speeds in contrast with the every day speeds of the airplane clocks. Meaning that the prediction discrepancy is extremely small because c plus earth orbit speed or c minus earth orbit speed is practically speaking the same value. But if you shoot particles a tlower and slower speeds, you will get increasingly stronger discrepancies with SR predictions, as shown by the HK experimental facts.

    Further, and very important, one usually says that GR is needed for rotating frames. Clearly the very existence and validity of the HK experiment contradicts that statement. Clearly GR is not needed. Only the radius from the earth center, the speed and direction of the airplane, and the Lorentz factor. They work with what could be called the principle of a merry go round to get higly accurate results, more accurate than sr used for particle accelerators for example.

    These are the facts, not my facts, but solidly proven facts found by physicists. And one should not try to counter facts with some part of a theory, progress in physics does not work that way.

    Accept the facts, extrapolate consequences, then improve the theory, not the other way around.

    Best, Koenraad

    ReplyDelete
  49. My impression of the interview agreed with that of Greg Malone's (much earlier) comment. That is, just from the interview material (not having read the background sources such as Dr. Rovelli's books) it seemed that Dr. Rovelli's point was that time is local, not absolute; that it exists, but not in a unique form (similar to curvature).

    On the issue of time-reversible equations, which is sometimes stated as "there is no arrow of time in the equations of physics", my idiosyncratic notion is that there is.

    Take Newton's Law d(mv)/dt = F. Hidden in that, as I see it, is the fact that in order to predict what will happen with that law using a numerical computation, the forward-form of the time derivative must be used: d[f(t)]/dt = [f(t+dt)-f(t)]/dt (which we use to calculate f(t+dt) from the data at previous times). In that form, there is an arrow of time, which the more common form ( [f(t+dt)-f(t-dt)]/(2dt) ) does not have. Only after an experiment or event has concluded do we have the data we need for the second form. There is no need to rewrite Newton's Law, we just need to acknowledge that a time derivative can have either a forward, backward, or neutral direction.

    ReplyDelete
  50. The Page-Wooters theory involves differential phases between states. It is similar in a way to gauge theory that involves local phase interference of states. The difference in phase is what defines time. In this way time can be seen to emerge in a local setting in the contact manifold defined by the ADM Hamiltonian NH = 0 and in the quantum form the Wheeler-DeWitt equation HΨ[g] = 0.

    If you have an EPR pair of states and you remain outside a black hole while your partner falls into the black hole the entanglement phase of this EPR pair is transferred to the black hole. The entanglement is shifted to the black hole instead of what you can recognize as your partner. Your partner from your perspective is a nearly infinitely red shifted form on the stretched horizon and FAPP indistinguishable from the black hole. Suppose you want to reconnect with your partner in the black hole to reestablish this entanglement. You would have to time your journey into the black hole, while your partner would have to be on a geodesic with a long time, that would require precise timing. A clock required to make this timing work would have a mass on the order of the black hole itself. Of course that would perturb the system so as to demolish your timing!

    In this way entanglement is also connected to the emergence of space and time or spacetime. With the Page-Wooter theory and this argument there appears to be a sort of dualism between local gauge principles and nonlocal entanglements.

    ReplyDelete
  51. @Tam Hunt "We are pretty much stuck with the passage of time as the fundamental container of existence, so all physical theories should overtly acknowledge that."

    This statement gets to the heart of the problem. What it does, is reify the human concept of time and insist that this concept has a causal relationship with reality. There is, however, no empirical evidence supporting such a reification. Neither you nor anyone else can demonstrate the existence of time independent of the observed physical processes from which the concept was derived in the first place.

    The concept of time (and space, and/or spacetime) is like the concept of temperature. Temperature is not a thing in and of itself. No one believes that something called temperature causes things to get hot or cold. Temperature is a descriptive term that blankets the underlying, well understood thermodynamical behavior of matter-energy systems. It is a useful descriptive concept, but a concept that has no causal relationship to physical reality.

    Time is just such a human concept derived from the observation of physical processes. There is no scientific evidence to support the reification of that concept into a physically interacting component of reality. The idea of "the passage of time as the fundamental container of existence" is simply a metaphysical speculation that lacks any justification for its inclusion in a scientifically realistic framework. Note also that time and temperature are not illusions. They refer to very real, underlying processes, but they do not cause those processes.

    ReplyDelete
  52. @bud rap, viewing the passage of time as fundamental is not reification. It's basic empiricism. Everywhere we look, at the human scale, at the micro scale, at the cosmic scale, we see the passage of time. It's literally universal. The "passage of time" is simply change (to be clear on my terms). You seem to be thinking that I am arguing for a Newtonian absolute time. That's not the case. I agree with Carlo that time should be defined simply as change. And that's why I've taken issue with his arguments that the passage of time (ie change) is an illusion at the fundamental level. There is indeed change at every level of reality, so change does not emerge. Ergo: the passage of time does not emerge.

    ReplyDelete
  53. PS. Here's a nice article from Maudlin also arguing that the passage of time (change) is fundamental: https://philocosmology.rutgers.edu/images/uploads/TimDavidClass/05-maudlin-chap04.pdf. Page 4 states: "I believe that it is a fundamental, irreducible fact about the spatio-temporal structure of the world that time passes."

    ReplyDelete
  54. "This is in fact false since Tycho did marry Kirsten Jørgensdatter, just not in a way we are used to: the fact that he was a nobleman from one of the most prestigious houses of Denmark and she was a commoner was an obstacle for a regular blessing, but they indeed were married with a morganatic union."

    True, but the point remains: he wasn't afraid to flaunt convention.

    ReplyDelete
  55. @Koenraad Van Spaendonck

    Is the following a correct summary of your comment?

    This experiment demonstrates that the huge majority of scientists don't know how to do relativistic calculations?

    ReplyDelete
  56. My comments above are directed to Tam Hunt.

    I want to second Bud Rap's comments. Time and temperature are in ways intimately related with the generator of unitary evolution -iEt/ħ and the log of a thermodynamic partition function -E/kT in a Wick rotation are equivalent. We then have the Euclidean time τ = it = ħ/kT. Temperature is something that parameterizes entropy with energy so T = ∂E/∂S, but entropy is the logarithm of the volume of a macrostate defined in a subjective manner. The log function does mean that variations in such definitions lead to tiny errors, so the “slop” is not bad. However, temperature is just a measure for how energy is shuffled around in a macrostate according to 1/T = ∂S/∂E. As Bud Rap puts it, temperature is not a causal factor, but the parameterization of other observables.

    Time and by extension space are similarly qualitative measures for how entanglements are coarse grained into similar configurations similar to a macrostate. Time is then an epiphenomenology, in some ways a manifestation of consciousness, that we use to parameterize a sequence of events. We then really do not have the case often said that “time flows like a river.” That sort of implies a rate of time with time, which does not make direct sense. Time it is not something that is itself a causal form.

    ReplyDelete
  57. "Again: how does time emerge in time?" This question struck me as similar to asking, "How does water emerge in water?" That is, it seems to assume its desired conclusion, that there is nothing more fundamental that time could emerge from, as water emerges, not from itself, but from the collective behavior of dihydrogen monoxide molecules, which individually do not flow or have surface tension.

    "It would appear from these comments that CR does not wholly subscribe to the expanding universe model and its singular 13.8Gy timeline." This comment from Bud Rap (whose next comment impressed me greatly) confused me. Suppose an early space-faring civilization occurred at, say, ten billion years from the Big Bang on our time-line, and sent out a colonizing space ship which reached a speed of 0.3c and is still in progress. On that ship, its clocks would not be indicating 13.8Gy since the Big Bang at this time--or so I thought.

    Wikipedia says this about the 1972 Hafele-Keating experiment: "When reunited, the three sets of clocks were found to disagree with one another, and their differences were consistent with the predictions of special and general relativity. ... Because the Hafele–Keating experiment has been reproduced by increasingly accurate methods, there has been a consensus among physicists since at least the 1970s that the relativistic predictions of gravitational and kinematic effects on time have been conclusively verified. Criticisms of the experiment did not address the subsequent verification of the result by more accurate methods, and have been shown to be in error."

    All of which is to say, sometimes other people's comments confuse me, but maybe that is my problem, and in any case I am grateful for the thoughts they provoke.

    ReplyDelete
  58. Carlo Rovelli says :

    " What indicates that our intuition of time is wrong is not microscopes or telescopes. It’s clocks. Just take two identical clocks indicating the same time and move them around. When they meet again, if they are sufficiently precise, they do not indicate the same time anymore. This demolishes a piece of our intuition of time: time does not pass at the same “rate” for all the clocks. Other aspects of our common-sense intuition of time are demolished by other physics observations


    This refers to experimental evidence with atomic clocks and comparable systems.

    But the question is, what is the definition of a clock ? A time keeping device ? The question is very important to try and understand what time in Relativity consists of.

    Because hour glasses and pendulums also keep time, but they behave opposite to atomic clocks as to their ticking rate.

    One cannot generalize the experimental effects to all time keeping devices.

    So time in Relativity must remain exclusively associated to the oscillation rate of electron energy levels and directly comparable systems.

    So the nature of time here should invoke the need to investigate the causes of (mechanisms behind) these changes of ticking rate. We should not stay satisfied with the mere well known formulas involving speed, or distance from a gravitational well.

    We know from the proven formulas that distances, ticking rates, lenghts, mass and energy, are all linked to the Lorentz factor and its gravitational counterpart. On top of that include frequencies of light.

    That's the starting point of a unification, where the deeper causes of the ticking rates should be found in comparisons of energy density, relativity of local energy densities.

    This demystifies time in relativity, and opens the road to a scientific explanation rather than stick with abstractisms.

    I've done a great deal of work to model these energy densities within a unifying picture.


    Best, Koenraad

    ReplyDelete
  59. I do not agree with the argument that a “change” in scope or perspective is a change that can occur without time. I don’t consider such as a “change” but a mere difference in ones description. Indeed, the only way to accomplish such a difference in the first place requires time. Saying otherwise ignores physical reality.

    Nor do I think time’s behavior under relativity speaks to its lack of fundamental reality. I don’t see any necessary connection between time being relative and it being therefore emergent or unreal. That it can never be ignored in any way speaks to its fundamental reality. (As xkcd once out it, ‘If you have to be constantly dragged along a dimension against your will, thank heavens it’s not one of the spatial ones!’ Which actually seems an argument that time isn’t really a dimension, either.)

    Coincidentally watched a video about time by Sean Carroll the other day. “What is Time?” He spoke about presentism and eternalism and his own views. Good video.

    ReplyDelete
  60. @Amos, I find the combination of the details you get right, those you get wrong and what is conspicuously missing puzzling. But your conclusions are gravely misguided.


    "I don’t think it detracts from Tycho’s reputation as a great observational astronomer to cite his wacky cosmological model as an example (similar to neo-Lorentzianism) of how people try to retain old conceptual frameworks and beliefs for as long as they can – and sometimes longer."
    Except it was not "wacky" at all, and in fact it went to dominate the scene for decades after the Ptolemaic model was disproved: given Brahe's role as an innovator making him such an example is rather uncalled for. I'll have to write a long rebuttal, however, for getting something wrong is an easy matter and getting it right requires a far greater deal of work.

    ReplyDelete
  61. "One of the implausible aspects of Tycho’s model was the non-rotating earth, which implied that the Sun and all the planets orbiting the sun (and all the stars) are all revolving in lock-step unison around the earth once per day. This is especially crazy since Tycho himself realized that (according to his model) the orbit of Mars intersects the Sun’s orbit, so he knew the old idea of spinning crystal spheres was untenable. The planets and comets etc. are all disconnected objects moving individually through space, and yet the whole formation (including the stars) is in rigid lock-step daily revolution around the earth. Really?"

    1) When doing history thou must consider what was known at the time, hindsight is 20/20: you seem to imply a void space between planets, which was highly doubtful. Brahe famously observed that comets passed through the "crystal spheres", which was far more convincing proof than his model: actually a fluid space was already extensively discussed, and its currents could have explained the common motion (someone said "vortex theory"?).


    "Tycho’s reason for thinking the earth doesn’t rotate (in addition to biblical scripture) was that – in modern language – he denied the principle of inertia. For example, he argued that – contrary to popular belief (and common-place experience!) a ball thrown directly upward inside a uniformly moving ship would not rise and fall vertically. He implied that he knew this for a fact"

    2) Brahe was far from being overly reliant on the Bible, like most of the other astronomers of his time: the Catholic Church held that the literal meaning of the Scriptures was the least important, and while the Protestant ones were more into it they were also kinda free to interpret it as they wished to. In fact, when during their astronomical controversy over heliocentrism Rothmann read one of his remarks as an accusation of impiety, Tycho took umbrage and denied the intention. At most his "religious" arguments were quasi-theological, an expression of the 'The Universe cannot be that weird' feeling that we sometimes see even now (only translated as the 'God cannot have made the Universe full of stars absurdly larger than the Sun' the Copernican model implied).
    (While I can't quickly find an online source, see "The Lord of Uraniborg" ch. 9)

    3) The exact quotation from Tycho is "[...] things actually happen quite differently. In fact, the faster the motion of the ship, the more difference will be found". If anything, it suggests that he observed himself on not too dense objects and implies that Brahe was even too much of an empiricist, since it's exactly what one would see. Since inertia was not established until Newton, if he didn't think of air resistance it makes sense.

    4) Brahe was not even that definitive in his belief in a non-rotating Earth. From "The Lord of Uraniborg": "Already in his letter to Brucaeus in 1584, Tycho professed to be unable to decide whether the diurnal motion belonged to the heavens or to the earth. But all he said about the issue was that despite the opinions of such distinguished ancients as the Pythagoreans and the followers of the "divine Plato," he was inclined to think it was the heavens that moved.". Also, "in his letter to Rothmann on 20 January 1587, Tycho stated that the possibility that the Earth might have a natural perpetual motion around its own center was not as problematical as most people imagined - certainly not as ridiculous as the Ptolemaic assumption that planets could circle equably around a noncentral equant point."

    ReplyDelete
  62. "Even one of Tycho’s assistants (Longomontanus) soon proposed a semi-Tychonic model that at least admitted the rotation of the earth."

    5) In fact, unlike the revolution, the rotation was regarded by the astronomers as a minor difficulty, at least since the inconclusive analysis by Buridan and d'Oresme based on the pre-inertial impetus theory in the XIV Century, and what was known for decades as the Tychonic model was often with a rotating Earth like for the one proposed by Ursus, bitter rival of Brahe.


    "Sure, the apparent optical diameter of stars was misleading, but it’s much easier to think of a physically plausible explanation for that than to rationalize Tycho’s model. (What is the apparent size of a candle’s flame from a mile away?)"

    6) You can't simply handwave away this central point. Yes, we know better, but they didn't.
    Galileo himself, extremely invested in refuting Brahe's conclusions, experimented with exactly candles, vision and glasses, yet concluded that while the naked eye was deceptive the telescope was reliable and did let one see the actual body of the stars. Upon suggestion from his supported, he thus performed the inverse of Tycho's investigation: by estimating the distances of the Mizar pair of stars from their apparent angular radius and the assumption that they were as big as the Sun he concluded that their relative parallax should have been about ten times their angular distance, more than easily observable. Alas, this experiment failed spectacularly, so Galileo did they honest, rational thing to do: he kept it a secret to his grave and wrote in the Dialogo that a similar setup would have been perfect to demonstrate Copernicus.
    Simon Marius and Riccioli instead simply updated the Tychonic argument.

    7) Ironically, the Copernicans' answers to Brahe's conclusions were not 'maybe your measures are wrong', but 'God did it!'.


    "The importance of Copernicus was in pointing the way toward the principle of inertia and realizing that motions are most clearly described in terms of inertial coordinates, which enabled the laws of Newtonian physics to be recognized. Tycho’s model, like Ptolemy’s, was not based on inertia."

    8) As I said in 5), actually Buridan and d'Oresme pointed toward inertia.

    9) As to what Copernicus would have realized about "inertial coordinates", I have no idea what you are talking about. Earth's surface is not an inertial frame of reference, and some of the objections to heliocentrism was that it didn't show.


    " It would never have been possible to discern the laws of celestial mechanics as long as the planetary positions were described in terms of an earth-centered system of coordinates."

    10) Again, you can't fault past people for not knowing which would have been the winning strategy in the long run.

    ReplyDelete
  63. @Amos
    In conclusion, the problem with trying to learn from history as Rovelli does is that one has to, well, get history right. Neither him nor you seem to have any accurate grasp of it.
    (The former for example has a tendency to decry the conversion of the Roman Empire by citing the "destruction of all the texts not in accordance with Christian ideas", something that has not happened, and to talk total nonsense about Lucretius's influence on science or his works' persecution by the Church, i.e. his "Council of Trent ban" being a mere hoax. Apparently he follows Greenblatt.)

    A little knowledge is a dangerous thing indeed.

    ReplyDelete
  64. Time is a fundamental property of our universe.
    Change is only possible because time exists of its own right.
    Change is how we detect time; time permits change.

    Time cannot “emerge’ from anything because “emergence” is a process or a sequence of events that happen “under or within time”. Time may be “implied” by things; but implication does not create or cause, it merely relates things.

    It is probable that physics can describe time but not account for its existence. Time can only be caused by existence itself. Our universe is vast; its contents are infinitesimal and dynamic. Without time, that last one is not possible.

    Certainly the “rate” of time varies from place to place; but that describes the behavior of changes occurring within time; it does not put time in doubt.

    Time is related to consciousness only in the way time is related to any process. Consciousness of time—our perception of time—is distinct from time itself.

    sean s.

    ReplyDelete
  65. "
    Because hour glasses and pendulums also keep time, but they behave opposite to atomic clocks as to their ticking rate."


    Reference, please.

    ReplyDelete
  66. B'Rat said...
    1) Brahe famously observed that comets passed through the "crystal spheres"… actually a fluid space was already extensively discussed, and its currents could have explained the common motion..

    Intersecting currents of a fluid at many different radii all remaining in rigid lock-step formation is self-evidently absurd, even from commonplace observations of fluids.

    2) Brahe was far from being overly reliant on the Bible…

    I didn’t say he was “overly reliant” on the Bible, I said his reasoning was based on failure to grasp how ordinary inertia works, and mentioned only parenthetically that he had also cited scripture.

    3) Since inertia was not established until Newton, if he didn't think of air resistance it makes sense.

    He talked about “from inside the ship”, so he knew his argument against a rotating earth was not valid when the air is calm at the earth’s surface like the air inside the ship.

    4) Brahe was not even that definitive in his belief in a non-rotating Earth.

    Right, because even he could see that it was wacky.

    5) …what was known for decades as the Tychonic model was often with a rotating Earth…

    That’s what I said, i.e., even those who espoused the “Tychonic model” soon rejected Tycho’s non-rotating earth with the implicit lock-step daily revolution of the entire celestial formation. (And yet you say this was seen as only a minor problem.)

    6) You can't simply handwave away this central point [about apparent optical diameter of starts]. Yes, we know better, but they didn't.

    I didn’t hand-wave it away, I said it’s far easier to imagine a plausible explanation for the apparent optical diameter of stars (people were already on the right track studying distant candles, etc.) than to rationalize Tycho’s wacky model.

    7) The Copernicans' answers to Brahe's conclusions were not 'maybe your measures are wrong', but 'God did it!'.

    We’re all “Copernicans” in the sense that we understand the stars are much further away than Tycho thought, and the scientific explanations for all the detailed phenomena where found by Copernicans because they were looking for the right things in the right ways, not by Tychonians. That’s the point.

    8) As I said in 5), actually Buridan and d'Oresme pointed toward inertia.

    Again, the importance of the Copernican model was in placing the description of celestial phenomena on a more nearly inertial basis, without which it would never have been possible to discern the laws of celestial mechanics. It enabled the phenomena of astronomy to be understood in terms of the inertial physics of Buridan, et al.

    9) As to what Copernicus would have realized about "inertial coordinates", I have no idea what you are talking about.

    I know. Again, the Copernican model describes the phenomena in terms of a more nearly inertial coordinate system. That’s why it was (and is) superior to other coordinate systems for purposes of understanding celestial mechanics.

    10) You can't fault past people for not knowing which would have been the winning strategy in the long run.

    The whole point for a scientist is to figure out (anticipate) the “winning strategy”, and Tycho’s model was very definitely not it. Many of his contemporaries could see clearly the superiority of describing the phenomena of the solar system in terms of a Sun-centered (or nearly Sun-centered) system of reference. As Rovelli said, Tycho’s model and neo-Lorentzian are both examples of attempts to maintain outmoded conceptual frameworks.

    ReplyDelete
  67. @ Tam Hunt,

    TH: ...viewing the passage of time as fundamental is not reification. It's basic empiricism.

    Empirical: based on, concerned with, or verifiable by observation or experience rather than theory or pure logic.

    Reify: make (something abstract) more concrete or real.

    TH: The "passage of time" is simply change (to be clear on my terms).

    You have therein defined something that cannot be observed (or detected) directly (time) in terms of something that can be directly observed (change). Change is an empirical fact, time is just a human concept derived from the observation of change(s). Conflating the two is an act of reification.

    TH: And that's why I've taken issue with his arguments that the passage of time (ie change) is an illusion at the fundamental level.

    If you define time in terms of something else then it cannot be fundamental.

    ReplyDelete
  68. @Jim V,

    Your example of a hypothetical spaceship moving at .3c gets to the heart of the problem. Relative to what reference frame is the ship moving? If your answer is the universal reference frame in which the universe's proper time is 13.8Gy, then the ship's occupants should be able to perform GR corrections to arrive at the correct universal proper time.

    If such a correction can be made, however, then there must exist a universal simultaneity (a universal now) at T=13.8Gy, a violation of Relativity Theory in which simultaneities can only be relative. On the other hand, if such a correction cannot be made then the claim that there exists a universal proper time is falsified, as is the standard model.

    At any rate, CR's comments, particularly this one: ...change is ubiquitous, but nothing states that this change should be organized along the single universal uniform flow that we commonly call time, seems to suggest a disagreement with the standard model of cosmology. However, since the question was not pursued, that may in fact, not be the case. I'm not trying to attribute to CR views that are not his. Just wish there had been an elaboration.

    ReplyDelete
  69. B’Rat to JimV: “Relative to what reference frame is the ship moving?

    A “universal reference frame” could be defined based on the mean CMB (mCMB); it would be analogous to our practice of measuring altitude from mean sea level. In JimV’s hypo, the colonizing space ship could reach a speed of 0.3c measured against the mCMB.

    There is no universal, absolute reference frame, but a standard reference frame is not impossible or without value.

    sean s.

    ReplyDelete
  70. Chris Sonnack: “Nor do I think time’s behavior under relativity speaks to its lack of fundamental reality. I don’t see any necessary connection between time being relative and it being therefore emergent or unreal.

    I completely agree.

    sean s.

    ReplyDelete
  71. bud rap; the passage of time is experienced constantly; that means it is observed. It is confusion to equate time with change. Change is how we observe the passage of time. The leaves on the trees outside my window are shaking in the breeze; that is one way I observe wind.

    If you define time in terms of something else then it cannot be fundamental.

    The passage of time is a “behavior” of time; time is fundamental, time is what is “passing”.

    sean s.

    ReplyDelete
  72. Bud Rap, thanks very much for the reply. As might have been predicted, it still leaves me confused.

    Yes, people on the postulated spaceship could calculate the time that we experience in our frame as we can calculate the time that they experience. But what makes our frame more fundamental than theirs (or vice-versa)? It seems to me that we could pick any frame and count "universal time" by it, but, as SR claims, that choice would be arbitrary.

    Separately, I would ask those who claim that our notion of time is fundamental and cannot be the result of anything more fundamental, how do they know they are not in the same position as those philosophers who disagreed with Democritus and his atomic postulate, which also seemed to violate the intuition of the time? I myself do not know whether time is fundamental or not, but I feel respect for those who are capable of imagining other possibilities, as Democritus did.

    ReplyDelete
  73. JimV;

    Separately, I would ask those who claim that our notion of time is fundamental and cannot be the result of anything more fundamental, how do they know they are not in the same position as those philosophers who disagreed with Democritus and his atomic postulate, which also seemed to violate the intuition of the time? I myself do not know whether time is fundamental or not, but I feel respect for those who are capable of imagining other possibilities, as Democritus did.

    This legitimate question can be summarized as “how do you know you’re not wrong?” Usually that is something no one can know, but in reply I have to ask why would I think I was wrong? (... about time. I’ve been and will be wrong about other things, for sure.)

    I believe that time is fundamental for two reasons I can cite at this moment: 1) because I cannot identify a likely candidate for time to “emerge” from or be caused by, and 2) I also cannot make sense out of how time can be created by a process/effect that operates within time. I cannot fathom how there could be a situation in which there is,
    a) no time, and then
    b) some time passes or some things happen, and then
    c) time appears.

    I respect those who are capable of imagining other possibilities; but I can’t buy into those other possibilities without a good reason; and so far, none of these visionaries (sincerely said) have given a persuasive argument for believing time is either imaginary, emergent, or otherwise not fundamental.

    sean s.

    ReplyDelete
  74. @Jim V,

    But what makes our frame more fundamental than theirs (or vice-versa)?

    Nothing makes our frame fundamental - in relativity theory all reference frames are local. The statement that the universe is 13.8Gy old imposes a universal frame on our conception of the cosmos that is antithetical to relativity theory.

    If relativity theory is more fundamentally correct about the nature of physical reality (and I believe it is) the the concept of a universal time frame is wrong and the 13.8Gy claim is meaningless.

    Just to be clear, this issue is distinct from the question of whether time is substantival or merely a relational concept.

    ReplyDelete
  75. Here is another interesting (?) point which might be relevant to our understanding of time (or current physics or both): according to the time-dilation factor of Relativity, a photon traveling at the speed of light experiences no time; even those of the CMB, which, according to our reference frame, occurred billions of years ago. Another, more familiar one is that one observer can experience event A occurring before event B and another observer in a different frame can experience B before A. (Which is why I never argue with basketball referees on block/charge calls.)

    (Some joke has it that "time is what keeps everything from happening at once"; then according to SR, for a photon, everything happens at once.)

    My main point on the issue of time is that calling Dr. Rovelli and others illogical ("logically incoherent") for suggesting that time might not be fundamental is itself illogical unless one has a logical proof (not a circular/restricted-choice one, such as "time cannot emerge from time") rather than just a belief. Without proof, people are entitled to believe (provisionally) what feels right to them and others to believe otherwise, without anyone necessarily being illogical. (In fact, I suspect a lot of such arguments are based on semantics, such as what it means to be fundamental.)

    ReplyDelete
  76. @sean s,I believe that time is fundamental for two reasons I can cite at this moment: 1) because I cannot identify a likely candidate for time to “emerge” from or be caused by, and 2) I also cannot make sense out of how time can be created by a process/effect that operates within time.

    This is a circular argument - the existence of time is in your premise.

    The passage of time is a “behavior” of time; time is fundamental, time is what is “passing”.

    For that to be true you would need to demonstrate the existence of non-passing time. I don't doubt that you sincerely believe in the fundamental nature of time, but I don't see any substantive evidence supporting that belief.

    ReplyDelete
  77. @JimV: “I would ask those who claim that our notion of time is fundamental and cannot be the result of anything more fundamental, how do they know they are not in the same position as those philosophers who disagreed with Democritus and his atomic postulate, which also seemed to violate the intuition of the time?”

    I would agree with every word @sean s wrote. Certainly the issue is unresolved, so it it ultimately boils down to which signs you follow to a conclusion. I’m not (I hope!) close-minded to alternate views, but I do need to see something compelling to switch horses, and I don’t find the idea of emergent time compelling in the least. To me, it seems incoherent that time emerges from change. Horse before the cart there, I think.

    I’m even puzzled by how physicists assert there is no time in physics. Isn’t physics often the study of dynamic systems? How is that possible without time being fundamental? Isn’t “action” a key concept in physics? What about Hamiltonians and Lagrangians? Time isn’t a part of those?

    ReplyDelete
  78. Chris Sonnack (thanks for the reply), if you are one of "those who claim that our notion of time is fundamental and cannot be the result of anything more fundamental", in an absolute sense (you're right as a matter of logic about this and could not possibly be wrong, e.g., what seems to be Tam Hunt's position), then my comment was aimed at you (that was what some philosophers, I think including Plato and Aristotle, thought about Democritus's idea); if you only have a provisional belief because you yourself cannot think of any other possibility, then it was not aimed at you.

    As I understand it (vaguely) some credentialed scientists have the idea that time and space evolve from quantum entanglements. I am not qualified to assess that idea, but by the same token I cannot dismiss it. I am not sure if this was what Dr. Rovelli meant though. He could be simply pointing out that time is a relative quantity, not something fixed. TM kept arguing against time-emergence in the interview, but CR did not seem to be defending such a position.

    Bud Rap, I guess we were furiously agreeing with each other all along, except not very furiously. I somehow got the feeling that your 13.8Gy reference was suggesting that an absolute time coordinate or universal time existed. I think now you were complaining that cosmology seems to pretend that. I apologize for my misunderstanding.

    ReplyDelete
  79. @bud rap, you're confusing a definition that includes something else as part of the definition, and a definition that is a simple identity. I've defined the passage of time as change -- an identity definition. Therefore the definition I've posed is simply a different word or phrase and therefore it is not reification. So, we agree that change is real. Therefore, the passage of time, defined as change, is also real. Therefore, any arguments or theories that suggest that change somehow emerges (in time) from a deeper changeless reality, are self-contradictory. The structural emergence arguments also fail for reasons I've already mentioned (any structural emergence is epistemological and therefore requires an observer, in time, by necessity).

    ReplyDelete
  80. Carlo Rovelli’s “book does not deny the reality of change”.

    The delta in the equations that represent the laws of nature is an assumption that change occurs, but these equations say nothing about why change occurs. Here “change” means “change of number”.

    Energy does not cause change: the equations show that the energy variable does not cause change and neither does momentum or mass. The equations can only represent relationships between variables; the equations cannot cause numbers to change. More correctly the equations, which represent laws of nature, show that the laws of nature cannot cause numbers to change.

    The only definite cause of number change in the universe is (something about) quantum events: something about quantum events drives number change, i.e. something about quantum events is driving the universe.

    ReplyDelete
  81. @JimV: I think I might be in superposition between your options. Is my belief that FTL is impossible because I cannot think of a way to accomplish it, or is it that SR is such a compelling argument? With regard to emergent time, I find myself in a like position. The arguments for fundamental time seem compelling and logical, while counter-claims (so far) don't.

    I'm not quite sure from the interview if Rovelli does think time is non-fundamental and emergent or just really complex beyond our experience of it. He doesn't offer a clue to its parts or what it might emerge from, and the meaning of his discussion of change isn't clear to me.

    I agree there are forms of change that don't include the notion of time. Change and time are different concepts, so they don't always overlap. But I think in physics, change implies two states separated in time. This is quite different than the abstract change implied by timelessly shifting perspective from a locally flat Earth to a globe as seen from space.

    To actually accomplish that shift in any physical way implies time. First, you're here. Then, you're there.

    It's also not clear to me the meaning of Rovelli's comments about SR. Absolutely, time is relative. But is he just pointing out old news for the interviewer, or does he mean this demonstrates time is not fundamental. I disagree with the latter. A thing about SR for me is that causality isn't violated. There is always a proper time from event to cause. Observers can disagree about simultaneous events, but they cannot disagree on cause and effect. (Time dilation just means you can run, not walk, into the future. It's not that Special.)

    Rovelli does suggest time is not singular, but again I'm not sure what he implies. Different observers have different time senses of what they observe. But their personal clocks tick identically. Proper time is always, well, proper, so to speak.

    If all Rovelli is saying is that time is more complex than our experience can ever tell us, I agree.

    I disagree time is not a fundamental property of reality, in part, on grounds that time emerging from something "timeless" does seem logically incoherent to me. The very notion of "arising" or "emerging" implies time. Other grounds include a constantly experienced proper time existing for everything and that all must agree on cause and effect.

    I do have a question that might be relevant enough to ask: Photons see their flight path as foreshortened to zero length, and therefore see no time between source and destination. Does this remain true when traveling through a medium for which the speed of light is slower than c? If not, does the photon suddenly experience length and time passing?

    ReplyDelete
  82. @JimV, Carlo does indeed argue for the emergence of time (as change) from a more fundamental reality that transcends time (based primarily on reasoning from the relativity of simultaneity and special relativity, which is what I was probing in my questions about alternative interpretations of the Lorentz transformations). He argues this emergence position both in his book and in our interview.

    ReplyDelete
  83. @bud rap, the 13.8Gy calculated age of the universe is an example of the use of a "cosmic frame" in cosmology, as I suggest in the interview is commonly done. So from the perspective of this cosmic frame (the CMB or other largest-scale structures of the observed universe), we calculate, based on general relativity, an accelerating expanding universe, and a few other assumptions, we calculate the age from the initial point of expansion (the singularity). So, if the cosmic frame is often used in practice, and is in practice the "preferred frame" used in Lorentz's interpretation of relativity, using the same equations as Einstein used (the Lorentz transformations), why wouldn't we seriously consider the Lorentz interpretation as a valid alternative to the flawed Einstein interpretation? When we also consider the easy consonance of the Bohm interpretation of quantum mechanics with Lorentzian relativity, and inconsonance with special relativity (as Bohm highlighted in his work), we get a number of pretty compelling arguments for the Lorentz interpretation.

    ReplyDelete
  84. "I've defined the passage of time as change."

    And yet, as previously mentioned, a lot of change has occurred since the CMB was emitted (all the galaxies we see formed, for one thing), but the CMB photons have experienced no passage of time during that change. This makes the definition seem inadequate, to me. That is, adequate for our daily affairs, but a better definition seems to be required to encompass all the physics we know.

    I think some will draw a distinction between photons and conscious observers, but I don't attach any magic to the process of consciousness, myself. (It seems somewhat analogous to me to the operating systems of digital computers.)

    "Photons see their flight path as foreshortened to zero length, and therefore see no time between source and destination. Does this remain true when traveling through a medium for which the speed of light is slower than c? If not, does the photon suddenly experience length and time passing?"

    I have wondered about this myself, so I will hazard my speculation, which is that the apparent change (observed by us) in velocity of photons in dense media might be due to the same cause that makes my walking pedometer vary from day to day along the same road which a car's odometer measures consistently: I don't walk in a straight line, due to obstacles I encounter on the way. Similarly, the photon's path may be warped by the EM fields it encounters in a dense media. If so, that is some more change the photon experiences (bouncing around due to particles in or near its way), without any passage of time.

    ReplyDelete
  85. Taking things a bit further (then I'll shut up for a while), might we not say that time is an emergent property of mass? Without mass, everything would move at the speed of light and there would be no passage of time.

    More seriously (although that was semi-serious), it still seems to me the issue is at least partly and perhaps mostly semantic. Dr. Rovelli might simply be defining fundamental as that which is fixed and absolute in any coordinate system. Emergent might just be used to denote something which depends on something else (e.g., mass) for its existence. Language is often ambiguous, as I have seen in many events here. (I almost typed "seen many times here".)

    ReplyDelete
  86. bud rap; “This is a circular argument - the existence of time is in your premise.

    No, in this conversation it’s been more of a given. The discussion here has been principally about the nature of time’s existence (illusiory, emergent, or fundamental); time’s existence has been treated as a given

    The conclusion of my argument (in answer to JimV’s question) is that time is fundamental; not that “time exists” so my argument is not circular.

    You could, of course, argue now that time does not exist. The last paragraph of my answer to JimV addresses that: I cannot buy into the “non-existence of time” without good reason. I am not aware of any good reason to consider that a credible possibility.

    For that to be true you would need to demonstrate the existence of non-passing time.

    There is no logical requirement to do such a thing; none that I am aware of.

    sean s.

    ReplyDelete
  87. JimV;

    ... a lot of change has occurred since the CMB was emitted (all the galaxies we see formed, for one thing), but the CMB photons have experienced no passage of time during that change.

    Sure they have: they’ve moved an enormous distance; many have been absorbed by objects along the way; or their direction has been changed by intervening special curves. Photons don’t have many properties that can change, but what they have has changed for the CMB photons.

    Any change experienced is an experience of the “passage of time”.

    Similarly, the photon's path may be warped by the EM fields it encounters in a dense media. If so, that is some more change the photon experiences (bouncing around due to particles in or near its way), without any passage of time.

    Photons “experience the passage of time” with every change you’ve mentioned, and with every wavelength of their existence.

    sean s.

    ReplyDelete
  88. @JimV, there's no passage of time for photons only if we assume the validity of the Special Relativity interpretation. In Lorentzian Relativity (using the same Lorentz transformations as in SR), time dilation is a coordinate effect only, a mathematical artifact (to use Lorentz's language), so not a real physical phenomenon. Accordingly, the CMB or other large-scale structure of the universe works just fine as a cosmic preferred frame. A great overview of these issues is the book Einstein's Clocks, Poincare's Maps, by Peter Galison. And for a shorter treatment see the last chapter in Max Jammer's book Concepts of Simultaneity.

    ReplyDelete
  89. The question is: “why are the numbers changing?”. We can’t merely accept that change just happens like Lee Smolin does: “We live in a universe that is always changing, full of matter that is always moving.” [1]. Clearly, physics’ time variable, and our sense of time, both derive from change.

    The equations that represent the laws of nature don’t change, but the numbers that represent aspects of fundamental-level reality do change.

    The only obvious and definite “moving part” in the universe is something to do with quantum events. Something about quantum events drives number change, and gives us our sense of time.

    1. https://leesmolin.com/

    ReplyDelete
  90. @Tam Hunt

    You make it look like your argument depends on Einstein being wrong, when you talk about his "flawed" interpretation of Special Relativity. When you talk about the consonance of the Lorentzian interpretation with quantum mechanics, can you make the argument mathematically precise? If you don't, then take this advice from another interested layman: you need to be very very aware that don't fully understand the arguments for and against this or that theory. Reading popular science books isn't nearly enough; the devil is in the (mathematical) details.

    And it seems you do not need to rely on alternative interpretations of relativity at all. In both special and in general relativity, there is no universal time, but there is still (local) time everywhere. Why not stick to this much simpler argument?

    ReplyDelete
  91. @Tam Hunt,

    I've defined the passage of time as change -- an identity definition.

    I'm sorry but I do not accept that rather arbitrary definition. The passage of time is not a directly observable thing. Time cannot be empirically demonstrated to exist except by reference to those things (changes of position or state of observables) that it is a measure of - it is a relational concept, like temperature. You cannot demonstrate the existence of time other than as a relational concept. Your attempt to define the concept of time into existence by identifying it with change is a reification, plain and simple.

    Therefore, any arguments or theories that suggest that change somehow emerges (in time) from a deeper changeless reality, are self-contradictory.

    This is a metaphysical straw man argument. I'm not generally interested in metaphysics and I have not made any metaphysical arguments like the one you are attempting to refute with that statement.

    ReplyDelete
  92. @doctor Boktor, I'm not exactly a layman in this area. Bee took some license in her tagline for my piece. I've written, or am writing, a number of books and papers in the last decade in which the nature of time is central, so I've read through much of the literature on the philosophy of time and related physical theories in the last decade. So, yes, I am suggesting that Special Relativity is highly problematic and the Lorentz (or neo-Lorentz) interpretation should now be considered superior for a number of empirical and philosophical reasons. Here's my in-progress technical paper addressing the empirical issues with SR: https://www.authorea.com/users/61793/articles/190801-reconsidering-the-interpretation-of-the-lorentz-transformations.

    And as I mention in the interview, we don't even need to challenge the mathematical formalisms if the issue with a theory is the interpretation of those formalisms, which is the case here. I'm not challenging the utility of the Lorentz transformations -- just their interpretation, i.e., their mapping on to the real world. Lorentz argued that time dilation is a mathematical artifact, a coordinate effect just like your time "changes" when you change time zones. So it's not a physical effect, just a coordinate effect.

    ReplyDelete
  93. @bud rap, at this point we'll have to agree to disagree. I've made clear what I'm talking about when I'm talking about time, and you're saying you're talking about something else. So let's leave it at that.

    ReplyDelete
  94. Tam Hunt said :

    " I'm not challenging the utility of the Lorentz transformations -- just their interpretation, i.e., their mapping on to the real world. Lorentz argued that time dilation is a mathematical artifact, a coordinate effect just like your time "changes" when you change time zones. So it's not a physical effect, just a coordinate effect."

    I agree and disagree.
    Take the train and the station, clock experiment.

    Time dilatation for the train clock is real, proven by experimental result:seconds lost on train clock compared to the other.

    Time dilatation for station clock is coordinate effect, proven by the fact that no seconds are lost on the clock compared to the other.

    If both were just coordinate effects, then both clocks would indicate the same time after the experiment.

    If both were real effects, again both clocks would indicate the same time.

    But it doesn't end there, and SR doesn't take that extra context into account: Both clocks will show seconds lost when compared to a clock on the sun. There is a cascading relativity at play here, which involves consecutive locally preferred frames.
    More accurately, describe it as a barycentric relativity, given the fact that the clocks at the barycenters are the preferred frames. Moon-earth-sun-black-hole,..

    Further, there is no symmetry for the situation of the train clock and the station clock. That is if you interpret symmetry as 'the same physical effect taking place'. You can call it a symmetry, but that's due to an artifical input of an equivalent, as coordinate speed, a mathematical extra input to complete the 'symmetry'

    And if it is not a real symmetry (same effect from both sides), then the Noether theorem no longer holds: no conservation of energy can come from, on one hand a real effect, and on the other hand a coordinate effect (a coordinate effect has zero meaning for, zero impact on reality).

    One should profoundly realize that with these SR experiments, there is only one experimental result: one clock shows less seconds on the clock then the other, there is no vice versa, fact.

    And it doesn't matter if some observer can't distinguish during the experiment. The facts and only the facts matter. He'll distinguish upon observing both clock after the experiment.

    Best, Koenraad

    ReplyDelete
  95. @Tam Hunt

    Ok, back to a question that Carlo already asked you in the interview, about GPS. As Carlo said, the time corrections needed for GPS are direct evidence for general relativity. I am confused by your response.

    If I understand you correctly you are saying that time dilation is a mathematical artifact, but not a real physical effect. Am I misinterpreting your words now? If not, how do you explain the time dilation that we observe in GPS? Those observations are made with very high confidence.

    I also had a look at your article. The argument of CMB being a special frame of reference struck me as weak - I don't see how it could be an argument against Relativity or its interpretation. It just means that the observable universe had an approximate common rest frame at the time of the big bang. I say approximate because there are statistical fluctuations in the CMB and its frame of reference is an average at best; it is not a fundamental constant. And just because there is an average common frame of reference doesn't mean its needs to have a special role in a theory of physics - even I can tell that, and you're not trying to convince me, but actual scientists. ;-)

    Discussions about the (in)compatibility of GR and QM are way above my head, so no further comments. I am sorry if I have been blunt in my previous response. Good luck with your article.

    ReplyDelete
  96. According to any clocks they have, the twin who stays on Earth and the one who travels at an average speed (as seen by the stay-home twin) of 0.2c to Alpha Centauri and back will be reunited at different ages. Whether this is just a coordinate transformation or the first twin will have gray hair and the second won't, I guess we will see when and if this experiment is completed. (I am betting on Einstein but not with absolute certainty.) (If I win this bet, then in fact photons do not experience the passage of time; I am surprised that anyone who has had Special Relativity in college is certain of the opposite. The arguments for it seemed very conclusive in my EM course.)

    Regardless of that, after reading the interview again I have a simpler explanation of what might be the issue.

    Infinity and infitesimality are concepts which can be explored mathematically but not really comprehended naturally. Based on this, Zeno (and/or his teachers) disagreed with the notion of continuous motion. Lucretius and Democritus, influenced by Zeno, applied the concept to the continuity of matter, and came up with the atomic theory. Some of us think this could apply to time also. That is, finite, discrete increments (very small) which combine to seem continuous to us, at our scale. Quantum Mechanics tells us that this definitely applies to energy, and suggests that at the Planck scale, things become at least ambiguous.

    In the interview, Dr. Rovelli uses the example of the Earth's curvature more than once. He tells us that it only "emerges" at a certain scale. It seems to me he could be saying the same thing about time. If it is composed of discrete increments, then it only seems to "flow" at some point (well below us) above the Plank scale. Below that, it jumps, discontinuously. It has the "illusion" of flowing. (But, he says in the interview, time itself is not an illusion.)

    Reading the interview with that concept in mind, everything he said makes sense to me. I don't know if that is actually what he had in mind (I would need to read the book, as he urged, which I probably won't do), but it seems consistent with what he was saying. (Also, separately, he agrees with the conventional notion of Special Relativity, as do I, provisionally.)

    ReplyDelete
  97. @Tam Hunt (and others),

    What exactly is the “change” that Carlo Rovelli is talking about, and why is this “change” happening?

    I would say that “change” means change in the numbers that represent fundamental-level variables (e.g. momentum, relative spatial location). Do you agree?

    Physicist Lee Smolin says: “We live in a universe that is always changing, full of matter that is always moving.” [1]. But why are the numbers “changing”?

    Lee Smolin seems to imply that the numbers change because that is just the way that reality is. Do you agree? Or is Time an entity that has the power to cause numbers to change? Or does something else cause numbers to change?

    1. https://leesmolin.com/

    ReplyDelete
  98. Tam Hunter;

    ... the CMB or other large-scale structure of the universe works just fine as a cosmic preferred frame.

    I agree completely; a mean-CMB is a perfectly valid standard or preferred frame.

    sean s.

    ReplyDelete
  99. bud rap;

    The passage of time is not a directly observable thing.

    Of course it is, we all experience this effect every day.

    Time cannot be empirically demonstrated to exist except by reference to those things (changes of position or state of observables) that it is a measure of ...

    That doesn’t matter; the Higgs Boson cannot be directly observed. It’s existence was empirically demonstrated by reference to the debris of its decay.

    “[TH’s] attempt to define the concept of time into existence by identifying it with change is a reification, plain and simple.

    Regardless of what one thinks of TH’s definition; it cannot “define the concept of time into existence” because that concept is far, far older than TH or any one of us.

    Time exists; we all experience it. The main question is “what is time?”

    sean s.

    ReplyDelete
  100. doctor Boktor;

    The argument of CMB being a special frame of reference struck me as weak - I don't see how it could be an argument against Relativity or its interpretation. It just means that the observable universe had an approximate common rest frame at the time of the big bang. I say approximate because there are statistical fluctuations in the CMB and its frame of reference is an average at best; it is not a fundamental constant.

    I think your take on the CMB as a “special frame of reference” is correct. The statistical fluctuations would argue for a “mean” CMB as the preferred standard, like mean sea level is.

    And just because there is an average common frame of reference doesn't mean its needs to have a special role in a theory of physics...

    Also correct, I think.

    sean s.

    ReplyDelete
  101. JimV;

    If I win this bet, then in fact photons do not experience the passage of time; ...

    Photons experience the passage of time; they experience changes in direction, in wavelength, and are absorbed/reflected by things they encounter. To the extent a photon has wave properties, every cycle of its existence is a change: another “tick of its clock”.

    Photons probably don’t “experience” the passage of time in the sense of being aware of time’s passage; but awareness is not a requirement of time’s passage.

    “[Rovelli] tells us that [the Earth’s curvature] only ‘emerges’ at a certain scale. It seems to me he could be saying the same thing about time.

    The “emergence” of the Earth’s curvature at some certain scale is an example of “emergence” without change which is why it can appear to be a “timeless emergence”. The shape of the Earth does not change as we gain a new perspective on it; it’s round and always was.

    The Earth’s curvature does not “emerge” at any scale; perception of its true shape is what “emerges”; and that “emergence” happens over a period of time.

    If [time] is composed of discrete increments, then it only seems to ‘flow’ at some point (well below us) above the Plank scale. Below that, it jumps, discontinuously. It has the ‘illusion’ of flowing.

    That may be correct; which goes to the behavior of time, not whether it is fundamental.

    sean s.

    ReplyDelete
  102. Lorraine Ford;

    I would say that “change” means change in the numbers that represent fundamental-level variables (e.g. momentum, relative spatial location). Do you agree?

    One should be careful here; those numbers are nothing more than how we measure things; they are not the things themselves. “Change” refers to changes in or to the things that constitute our universe.

    Physicist Lee Smolin says: ‘We live in a universe that is always changing, full of matter that is always moving.’

    That does seem to be the case.

    But why are the numbers ‘changing’?

    Again, those numbers are nothing more than how we measure things; they are not the things themselves. The numbers change because the things they measure change.

    Think of the numbers like shadows; shadows move because the object casting them moved, or the light source moved.

    So your question should be “why are things always changing?”

    That’s a good question; but should the universe be static instead of dynamic?

    It may well be the answer you seek will never be better than “That’s just the way it is.” Better answers should be sought, but better answers might never be found.

    sean s.

    ReplyDelete
  103. @Tam Hunt: "Lorentz argued that time dilation is a mathematical artifact, a coordinate effect just like your time "changes" when you change time zones."

    What does that mean in terms of muons from cosmic ray showers reaching the ground? Or in terms of the apparent lifetimes of particles in high-speed colliders? How does the mathematical artifact become physical reality?

    @Koenraad Van Spaendonck: "But it doesn't end there,..."

    Even SR isn't the end of it! Because of GR, the Earth's center is younger than its surface. The center of the sun is all the more younger than its surface. We change our relative time every so slightly by merely going down a mine shaft or up a mountain. GPS is the physical reality of this.

    SR and GR make the spacetime fabric as twisted as smoke in air. (But causality still trumps all. Light cones don't lie!)

    FWIW:, philosophically speaking Kant thought space and time were intuitions -- immediate and singular -- rather than concepts. This is because concepts are made of other concepts, while intuitions are indivisible. The concept {desk} has many parts, but "my desk" is a singular concept.

    Likewise, space and time, Kant said, are immediate and singular. They are not comprised by other parts, and they are everywhere self identical. The space over here is the same sort of stuff as the space over there (for any given here and there). The minutes I experience are the same sort of stuff the dinosaurs experienced. Proper time passes the same way in all frames within those frames.

    How this philosophical view translates to physical reality is a question, but this does have a lot to do with my own view on space and time being fundamental aspects of whatever-the-heck reality actually is.

    ReplyDelete
  104. I'd like to ask again: Why the common assertion 'there actually is no time in physics'?

    From Maxwell's equations to Einstein's SR and GR to Schrödinger's wave. In the definition of an action, of a Lagrangian, of a Hamiltonian. There are little ts scattered everywhere. Even Feynman diagrams have a "before" and "after."

    What point of view in physics makes time go away?

    And, FWIW, I don't see SR and GR arguments as very strong against the idea of fundamental time and space. The monkey wrench for me is entanglement and what non-locality suggests about time and space. Reconciling that with realism is tricky.

    ReplyDelete
  105. I did an edit/search (Ctrl-F) on this thread for "fundamental". Tam Hunt uses it many times in his questions, and Dr. Rovelli seems to disagree with those interpretations in most cases, using "fundamental" only twice himself, as follows:

    "What I say is not “we live in a world without time.” What I say is “we live in a world without time at the *fundamental* level.” There is no time in the basic laws of physics. This does not imply that there is no time in our daily life. There are no cats in the *fundamental* equations of the world, but there are cats in my neighborhood. Nice ones. The mistake is not using the notion of time [at our human scale]. It is to assume that this notion is universal, that it is a basic structure of reality. There are no micro-cats at the Planck scale, and there is no time at the Planck scale."

    It seems clear that by fundamental/basic, Dr. Rovelli means events which take place at the Plank scale, where QM tells us it is impossible to measure anything since the act of measurement will perturb the system strongly enough to destroy all the data to be measured. At that scale, in my hypothesis, (and perhaps at somewhat larger scales depending on the minimum increment) time is not a continuous parameter. It jumps discontinuously, and between jumps there is no time. If the jump goes from one Planck-time to the next or one several Planck-times later, there never is any time within a Planck-time. (Try not to get hung up on the semantics, he cautioned himself, this just means the Planck-time is a fictitious time-increment which is less than the actual minimum time increment.)

    That may or may not be true, but there seems to me to be nothing illogical about it, and if so, much of the discussion here has been chasing red herrings (to mix metaphors egregiously). That is, a few people have chosen their own worst-case interpretations of what Dr. Rovelli is saying and prosecuted him for them in absentia; and/or decided to argue with the consensus of known physics on things like time-dilation.

    In the interview as I read it, Dr. Rovelli is saying that the common-sense view of time (as fixed and universal, for example) is not supported by physics as we know it (Special Relativity and Quantum Mechanics). The examples he gives are the same kinds that are given in any college courses on these topics (except perhaps for some, but not all, "Bible colleges"). People arguing with those examples are *fundamentally* arguing with college texts and with Einstein and other Nobel Prize winners. As is their right, and perhaps they will win their own Nobel Prizes someday.

    ReplyDelete
  106. In Lorentzian Relativity, time dilation is a coordinate effect only, a mathematical artifact (to use Lorentz's language), so not a real physical phenomenon.

    That's not true. The only empirically viable "Lorentzian relativity" is just an interpretation of standard special relativity, not a distinct theory, so it has all the same "real physical phenomena". You're misrepresenting "Lorentz' language", because he was describing his earlier failure to realize the full physical significance of the t' coordinate (that he had originally introduced merely as a mathematical artifice), and the "remarkable reciprocity" (to use Lorentz's phrase) pointed out by Einstein between t' and t. Time dilation, length contraction, the inertia of energy, etc., are all observable phenomena, not merely "coordinate effects", regardless of which interpretation you espouse. For example, two identically constructed clocks following different paths between two events will show different elapsed times. This is not a coordinate effect, and Lorentz was well aware of it.

    I am suggesting that Special Relativity is highly problematic...

    In what way? Obviously it doesn't encompass gravity, so it is understood to apply only locally, and hence today special relativity is really just the principle of local Lorentz invariance. This is just about the least problematic thing in physics. Strenuous efforts have been made to detect the slightest violation of local Lorentz invariance, but no such violation have ever been found. In what way do you consider it to be "highly problematic"?

    ReplyDelete
  107. @sean s,

    Of course it is, we all experience this effect [time] every day.

    Your answer to @Lorraine Ford re numbers is directly applicable here: ...numbers are nothing more than how we measure things; they are not the things themselves. Likewise with time - it is not a thing itself. It is nothing more than a human way of thinking about and measuring underlying physical phenomena. All of your statements to the contrary seem, to me, subjective and/or metaphysical in nature.

    ...the Higgs Boson cannot be directly observed. It’s existence was empirically demonstrated by reference to the debris of its decay.

    Those are contradictory statements. To be directly observed is to be empirically demonstrated. You are confusing empirical evidence of the Higgs, which, as you say, does not exist, with model dependent inferential evidence. Empirical evidence is conclusive, inferential evidence is not. That the standard model is suffused with particles that are only inferred to exist within the context of the model, is a whole other can of metaphysical worms.

    Regardless of what one thinks of TH’s definition; it cannot “define the concept of time into existence” because that concept is far, far older than TH or any one of us.

    Let me clarify my statement. TH's definition simply reifies the concept of time by identifying it with something that does exist. If I say, the beating of angel's wings causes change, can I justify that statement by then defining the beating of angel's wings as change?

    Time exists; we all experience it. The main question is “what is time?”

    Temperature exists; we all experience it. The analogy is precise. Do you experience any existential qualms about the nature of temperature? Is temperature somehow 'fundamental'? If not, why not? What distinction is to be made between the natures of time and temperature in this regard?

    ReplyDelete
  108. @Amos

    "

    'I am suggesting that Special Relativity is highly problematic...'

    In what way?.. "


    Perhaps the following could interest you.

    In my post nr 49 above, I clearly indicated how SR predictions can deviate strongly from the facts, accompanied with plenty of context to be found there : extract :

    "Consider a particle accelerator on the equator shooting particles east and west, to get a nice parallel to the HK-setup:
    This is of course due to the relativistic speeds in contrast with the every day speeds of the airplane clocks. Meaning that the prediction discrepancy is extremely small because c plus earth orbit speed or c minus earth orbit speed is practically speaking the same value. But if you shoot particles a tlower and slower speeds, you will get increasingly stronger discrepancies with SR predictions, as shown by the HK experimental facts."
    Best, Koenraad

    ReplyDelete
  109. Great Blog here, just discovered :)
    I have to say also couple of words.

    First of all Carlo Rovelli is brilliant here. I have feeling that sometimes if not often philosophy is dangerous for the science (with all respect to philosophy itself).
    We should be careful not to fall into playing with words. We can fall easily in traps of words.
    One can see this problem between scientist who tries to stick with reality of nature and the philosopher who seems to fall into traps of words, concepts, ideas etc.

    I think in science we should try to somehow empathy with nature when we try to understand how it works. Math is all fine and very useful, but at the end it is still a tool in service of science and humanity.
    It's nonsensical to try to match the nature to math and the results we get from complex calculations on paper. It should be other way around. If that does not work for some phenomena, then may be the math is not the right tool, may be we need something more advanced.
    Anyway, back to the time.
    I think time does not exist on its own. It's what we name it. It shows the distance between the events. We can measure time again with help of events.
    A popular event is the Eath's rotation. Our nanosecond is a fracture of this rotation. We name it nanosecond, you can further go downwards in scale.
    With these seconds and hours we actually don't measure time. We measure events. That precise clock in my smartphone ticks....and what it shows me is how many ticks was it there.

    We have the Universe, the matter, galaxies, stars, planets, here on Earth it went so far that we have biological systems: Virus, bacteria, more complex organisms. Everything is interacting with each other....obeying rules of causality. Egg needs to be fertilized before it starts to divide. Most of the events are sequences: Sun appears on the horizon...again and again...you can count how many heart beats you have between the Sun's appearance and call it a day, then day is this many heart beats long.
    Regardless of time, my heart beats with more or less constant frequency on the Earth.
    Time does not exist....it's the Universe and events (from macro to nano scale) which infers what we need to call time.
    Yet, it's very interesting and strange what Einstein says: If I travel fast enough and come back to Earth, I age less than others! How is that, I don't mean the clock I took with me ticks slower.....is it illusion? What changes physically? Does my Heart beats slower? My cells divide slower or less? My body needs to obey some rules so that I can survive...What does it mean I age less? Is it a illusion or is it real? If it's real....my biology must change so that I age less...but this seems not possible. I've found....it must be so that as I travel with high speed...My time must flow faster than those on Earth, yet my Heart beat is still the same...and when I'm back on earth, I say I traveled 1 year...my friends on Earth will say: No you were 10 years away. Hah! Still strange....Ok, I eliminated my Heart frequency and the cells division pace....what left: CLock? I took a clock with high precision crystal oscillator....does the frequency change on the flight? Is that possible? What changes physically? Einstein! God damn man....this Universe is so strange! Reverence.

    ReplyDelete
  110. @Tam Hunt,

    ...at this point we'll have to agree to disagree. I've made clear what I'm talking about when I'm talking about time, and you're saying you're talking about something else. So let's leave it at that.

    Sorry, we have no agreement to disagree. That assumes we merely have differences of subjective opinion. That is not the case here.

    You are simply unwilling to debate the question of whether time is a substantival or relational concept, unless you can enshrine your substantivalist view in the terms of the debate by redefining time as change.

    I'll debate you on this issue anytime you are willing. I will not insist that you conceed my position at the outset.

    The fact that you now want to agree to disagree because I won't conceed your central point at the outset, however, speaks to the weakness of your arguments. Take your marbles and go home if you can't defend your position, but we have no agreement to disagree.

    ReplyDelete
  111. Chris Sonnack;

    Kant thought space and time were intuitions -- immediate and singular -- rather than concepts.”

    I doubt the distinction between intuitions and concepts is meaningful here; and with respect to Kant, he never knew about GR and the evidence of how time operates under it.

    sean s.

    ReplyDelete
  112. JimV;

    (Quoting TH): “... and there is no time at the Planck scale.

    If there are events at the Planck scale, then there is a “before” and an “after”; and therefore time. If there’s no time at the Planck Scale; then nothing can happen at that scale.

    At the Planck Scale, time may be “quantitized”; or perhaps merely events are quatitized while time is continuous. Again, change is not time; change is how we observe time. Certainly our common-sense view of time is not right; but our common-sense view of many things is not right; and yet those things exist.

    sean s.

    ReplyDelete
  113. bud rap;

    “[Time] is not a thing itself. It is nothing more than a human way of thinking about and measuring underlying physical phenomena.

    Unfortunately, unless time does exist, the things we are thinking about cannot occur. Unless time exists, thinking cannot occur. Change cannot occur unless time exists. We cannot measure phenomena except by acting within time; and we all our thoughts occur within time; so time cannot be the product of either thinking or measuring.

    A topic this thread has taken up is the nature of time. If that’s a metaphysical topic, then metaphysical arguments are appropriate. Your claim that time does not exist is just as “metaphysical” as mine that time is fundamental.

    To be directly observed is to be empirically demonstrated.

    True, but the converse does not hold. To be empirically demonstrated does NOT require direct observation. Ex: Higgs Boson.

    You are confusing empirical evidence of the Higgs, which, as you say, does not exist, ...

    The Higgs boson does exist; I’ve never said otherwise. Are you saying it does not exist? Do you reject claims that the Higgs Boson was discovered/confirmed by the LHC?

    ... with model dependent inferential evidence.

    My statements about time are in no way based on “model dependent inferential evidence”.

    Let me clarify my statement. TH's definition simply reifies the concept of time by identifying it with something that does exist.

    Since time exists, TH’s definition cannot reify it.

    Temperature exists; we all experience it. The analogy is precise.

    Humans do not experience temperature; human experience heat (or its relative absence: cold) and changes in heat (heating, cooling, or wind chill). Your analogy is poor.

    Consider a windless, overcast, 28-degree day (Fahrenheit); one would probably just need a light jacket. Now compare that to being immersed in 28-degree (F) sea water. One would likely go into shock within 2 minutes and drown. Even with a life-jacket, survival expectation drops to mere minutes. Yet in both situations, exposure was to exactly the same temperature! Why the dramatic, grave difference?

    Because the heat-transfer occurring in the two situations is dramatically different; the cold water will simply suck heat from a body faster that it can generate it; resulting in hypothermia, shock, and death. All this at a temperature where air is benign.

    Do you experience any existential qualms about the nature of temperature? Is temperature somehow 'fundamental'? If not, why not? What distinction is to be made between the natures of time and temperature in this regard?

    Time and heat both exist and can be experienced; so do many other things. Similarities or some shared attributes do not mean all attributes are shared. Cats and tardigrades share some attributes, but they are in no way the same.

    You can generate heat; just rub your hands together. We cannot generate time. We can move heat around. Time: not so much. Etc.

    Time is fundamental; heat might be fundamental, maybe; but temperature is not.

    sean s.

    ReplyDelete
  114. @Koenraad

    The HK experiment is not at all in conflict with SR. Some people try to argue that it does, but they are ignoring two basic facts:

    1) The surface of the Earth is not an inertial frame. It rotates. You have to choose an inertial frame before you start your relativistic calculations, otherwise you get the wrong answers. In the case of an airplane this is especially important, because it moves slowly with respect to the speed of rotation.

    2) You need to correct for effects of general relativity.

    If you take both of these into account, the results of the HK experiment are in perfect agreement with special relativity. The original experimenters were fully aware of that and never tried to argue against SR.

    Unfortunately, some pseudo-scientists later acted as if the surface of the earth is an inertial frame, concluded that SR must be wrong and spread those ideas on the internet.

    ReplyDelete
  115. @doctor boktor,

    You asked about time dilation and GPS. This is where the debate gets very interesting. In the technical paper I gave you a link for I included an appendix looking at three of the most cited papers concerning time dilation, the last of which addresses a well-known 1997 paper on the GPS system, using GPS data to test the anisotropy of the speed of light. Here's the summary of my analysis of that paper (see my paper for the full discussion):

    The [Wolf and Petit] paper states: “δc is the deviation from c of the
    observed velocity of a light signal traveling one way along a particular
    spatial direction with the measuring clocks synchronized using slow
    clock transport.” Slow clock transport is by definition equivalent to
    Einstein synchronization in the same inertial frame. And under Einstein
    synchronization the constant speed of light, regardless of the motion of
    the observer, is assumed. This is an operational assumption made in
    order to provide a simple and reliable way to synchronize distant
    clocks. It is important to note also that ongoing re-synchronization
    cannot, of course, be done using slow clock transport; Einstein
    synchronization (using light signals) must be used. Einstein states in
    his well-known book on SR and GR (p. 27 of the 1952 edition, emphasis in
    the original):

    "There is only one demand to be made of the definition of
    simultaneity, namely, that in every real case it must supply us with an
    empirical decision as to whether or not the conception that has to be
    defined is fulfilled. That my definition satisfies this demand is
    indisputable. That light requires the same time to traverse [a given
    path] is in reality neither a supposition nor a hypothesis
    about the physical nature of light, but a stipulation which I can
    make of my own free will in order to arrive at a definition of
    simultaneity."

    This technique does provide a concrete method for defining simultaneity
    and thus for synchronizing distant clocks, but we must be careful to not
    use this technique and then forget that we have from the outset assumed
    an isotropic c in order to achieve synchronization.

    Unfortunately, Wolf & Petit overlooked this issue in their methodology.
    The 1997 paper is often cited (over 100 citations) as strong support for
    relativistic effects. While finding the methodological tautology in this
    paper is not readily apparent to the casual reader, it is surprising
    that no other physicists or philosophers have noticed this fatal flaw in
    this well-known paper.

    In terms of CMB as a preferred frame, another point you raise, you're misunderstanding my point here. The point I was making is that contrary to SR's assumption that there is no preferred frame in the universe, there is in fact a pretty obvious preferred frame in the CMB (with its anisotropies and its literally universal scope), that is in fact used all the time in astronomical work such as calculating the age of the universe or calculating z scores for distant stars. So this may reasonably be considered evidence for falsification of SR. It's not a slam dunk case, as I acknowledge in my paper, on this count alone, but it's a pretty big black eye for the universality of SR, and combined with the the various other considerations in my paper I conclude that the weight of the evidence is in favor of the Lorentz view.

    ReplyDelete
  116. @Amos, I cover the empirical and philosophical issues with SR in this in-progress paper here: https://www.authorea.com/users/61793/articles/190801-reconsidering-the-interpretation-of-the-lorentz-transformations.

    ReplyDelete
  117. @Amos, you write:

    "That's not true. The only empirically viable "Lorentzian relativity" is just an interpretation of standard special relativity, not a distinct theory, so it has all the same "real physical phenomena". You're misrepresenting "Lorentz' language", because he was describing his earlier failure to realize the full physical significance of the t' coordinate (that he had originally introduced merely as a mathematical artifice), and the "remarkable reciprocity" (to use Lorentz's phrase) pointed out by Einstein between t' and t. Time dilation, length contraction, the inertia of energy, etc., are all observable phenomena, not merely "coordinate effects", regardless of which interpretation you espouse. For example, two identically constructed clocks following different paths between two events will show different elapsed times. This is not a coordinate effect, and Lorentz was well aware of it."

    Your statements here aren't accurate. Check out Peter Galison's excellent book Einstein's Clocks, Poincare's Maps, for a review of this history. Lorentz actually came up with the Lorentz transformations (hence the name) and Einstein adapted them for SR, so Lorentz Relativity (LR) is not a variant of SR, it's vice versa. It is the case, of course, that SR is by far the more dominant approach today and LR all but forgotten. But I'm suggesting that this state of affairs should change at this juncture because our empirical and philosophical knowledge have changed and we now have abundant reasons to opt for the Lorentzian or a neo-Lorentzian approach.

    In terms of relativistic effects, length contraction is part of both LR and SR, but for different reasons. Whereas there is a real physical mechanism for length contraction in LR (interaction with the ether in a similar manner as a ponderable body interacting with a fluid would experience drag), the mechanism for length contraction in SR is simply interaction with spacetime. And this interaction with spacetime flows directly and only from the assumed constancy of the speed of light for all observers. That assumed constancy thus renders time and/or space malleable (b/c for c to remain constant time and/or space must shift).

    Time dilation for Lorentz was, however, a mathematical fiction/artifact, and not a real physical effect, despite the equations in LR as well as SR showing "time dilation." Lorentz was explicit in his work that he saw "local time" (time dilation clock readings) as equivalent to changing time zones, that is, a coordinate effect only, a result of one's perspective, not a real physical change in the flow of time. In SR, again, time dilation, which is purported to be physically real, results simply from the postulated constancy of c (plus the principle of special relativity). Again, Galison discusses this history quite well.

    And if you're still not convinced check out Jammer's book Concepts of Simultaneity, particularly the last chapter on the conventionality of simultaneity debate.

    As for observed changes in clocks traveling at different speeds, the famous Hafele-Keating experiment was in fact heavily massaged, as a paper published in 2000, using their original data, made clear. I discuss this paper and the experiment in the appendix to my in-progress paper here: https://www.authorea.com/users/61793/articles/190801-reconsidering-the-interpretation-of-the-lorentz-transformations.

    There's a lot to debate here, but in my review of a number of papers purporting to find physical time dilation, all of what seem to be pretty serious problems. I have not done any exhaustive review at this point but it's pretty telling that all of the ones I have reviewed so far have been far from problem-free.

    ReplyDelete
  118. I’ve only been half-following this intertwining of several discussions, through notifications since I commented, so I’ll use a very small voice:
    Can we not take time to be a coordinatization of change?
    We typically establish timings by reference to a number of repeats of the same state of a changing process, that the sun is in (almost) the same place as yesterday, but it moved in between, and that is the naive idea of a day's length (or, technologically, we have to wind up a clock, for example, otherwise the second law of thermodynamics will quite soon make the clock stop). On this view, one can’t speak of time without change, but the a posteriori fact that some processes of change do repeat FAPP makes time quite as fundamental as the a posteriori fact that we, embedded in everything else, do notice changes in everything else relative to our own internal processes. We notice the sameness of repetitions almost as much as we notice the differences of change, and this is the way of the world.

    ReplyDelete
  119. @Doktor boktor

    I appreciate your effort to clear things up.

    But you can't get the right results without using the ECI as a locally preferred frame. The clock difference between A and B airplain clocks, results from comparing first A to eci and b to eci.

    Secondly, if you exclude the earth surface because it rotates, then you couldn't use sr for particle accelerators for instance. We do that anyway, and the results are ok, but would greatly differ from sr predictions if you shoot particles at much slower speeds, as I indicated, because then the rotation speed of the earth becomes more and more significant for the result. This becomes very clear if a linear accelerator were placed parallel to the equator, but even without that, there's almost always a velocity vektor parallel to the equator.

    I am aware of GR corrections due to heights.

    I'm not fooled by pseudostuff on the internet, I look at the facts, but you seem to want to counter them with theory.

    And frankly I don't care at all about this entire SR debunking agitation, I'm primarily interested in understanding the larger context, and the HK experiment reveals something about that, unlike straight forward sr experiments.

    Best, Koenraad

    ReplyDelete
  120. @sean s,

    A topic this thread has taken up is the nature of time. If that’s a metaphysical topic, then metaphysical arguments are appropriate. Your claim that time does not exist is just as “metaphysical” as mine that time is fundamental.

    Apparently this is where our disagreements are sourced. I am not the least bit interested in metaphysical arguments as such. However, when metaphysical arguments are used, as you and Tam Hunt appear to be doing,to assert the physical existence of something for which there is no scientific evidence, I'm going to object - on scientific grounds. Science is the study of observable (broadly inclusive of our various mechanical extensions of our senses), physical reality.

    There is absolutely no empirical evidence for the existence of time as a physical entity that causes change. You seem to be claiming the opposite and that is the crux of our disagreement. In fact this just an ancient metaphysical dispute (Is time a substantival entity or a relational concept?) which is readily resolved by science but insoluble in the realm of metaphysics.

    Science rests on empiricism - it is the study of the phenomenological world. In the scientific realm no evidence for the existence of substantive time can be found; time as a relational concept is, however, both acceptable and useful to science. It is that distinction that you seem to have difficulty understanding.

    The Higgs boson does exist; I’ve never said otherwise. Are you saying it does not exist? Do you reject claims that the Higgs Boson was discovered/confirmed by the LHC?

    I am saying that there is no direct empirical evidence for the Higgs, which is indeed what you said: ...the Higgs Boson cannot be directly observed.

    The attempt to conflate empirical evidence (direct observation) with a model-based inference derived from the empirical observation of other phenomena, is as disingenuous as it is common. The two forms of evidence are not the same and therefore, should not be weighted the same. Direct empirical evidence is far stronger than inferential evidence. So, my position on the Higgs is that there is no compelling evidence for its existence.

    Since time exists, TH’s definition cannot reify it.

    Circular arguments like this are logical errors. Also, you cannot partially quote an argument, change the terms of that argument to suit your own purposes, and then claim to have refuted the original argument:

    Humans do not experience temperature; human experience heat (or its relative absence: cold) and changes in heat (heating, cooling, or wind chill). Your analogy is poor.

    Doing that sort of thing speaks to the weakness of your own arguments.

    ReplyDelete
  121. sean s,

    Re “time permits change” (4:08 PM, August 29, 2018 ) and “Change cannot occur unless time exists” (9:30 PM, September 04, 2018):

    Except for the abrupt change of particle interactions and other quantum events, “change” means the reality underlying (what we would represent as) the smooth change of number for a variable. Considering that physics represents reality as equations and numbers, you’ve only got 2 choices for “change”: change of number for a variable, and change of law of nature equation. (And presumably, especially with living things, you’ve got change of what can only be represented algorithmically). No matter what the situation, at a fundamental level, change usually means change of number.

    The abrupt change of particle interactions and other quantum events cannot be thought of as change in time; seemingly only smooth change of number can potentially be thought of as change in time. In any case, when looked at closely, smooth change of number is seemingly not smooth: it is abrupt/instantaneous i.e. not occurring in time. I.e. all change of number for a variable must seemingly be able to be traced back to the abrupt change of quantum events, and the law of nature relationships between these variables.

    ReplyDelete
  122. sean s,

    Re “change is how we observe time” (9:23 PM, September 04, 2018):

    This is the wrong way around: it should be “time is how we observe change”. I.e. our sense of time derives from knowledge of (what we would represent as) number change.

    Note that the physics’ time variable does not “emerge”. The physics’ time variable can seemingly only be derived algorithmically, because the only way to determine that numbers have changed is via an algorithm. Considering that life seems to use the equivalent of algorithmic information, is it surprising that the universe might be using algorithmic information?

    ReplyDelete
  123. Koenraad Van Spaendonck said... In my post nr 49 above, I clearly indicated how SR predictions can deviate strongly from the facts…

    Aside from the effects of gravity, the results of Hafele-Keating agree perfectly with the predictions of special relativity. What you described in that post was a common misunderstanding related to rotating coordinate systems and the Sagnac effect.

    Using the ECI as a locally preferred frame, you add up the speed of A to C, and you subtract the speed of B from C, yielding thus the differing times on the clocks as experimentally established.

    The ECI is not used as a “preferred frame”, it is simply one of infinitely many inertial coordinate systems, all of which give exactly the same results. Of course, we would get the same results using a non-inertial coordinate system too, but in that case the equations of physics take a more complicated form. The class of inertial coordinate systems is distinguished in this sense, but it does not constitute or single out any “preferred frame”.

    Now, one might argue that SR is not suitable for rotating frame setups, but then of course you disqualify SR for almost all earth bound experiments because there's always a velocity vector parallel to the equator.

    The fact that the Earth is rotating does not disqualify the use of special relativity for Earth bound experiments. In fact, one of the earliest Sagnac devices was built specifically to show that it could detect the rotation of the earth – thereby corroborating the prediction of special relativity. Hafele-Keating is another good example. Of course, in situations of limited extent and duration, such as in a lab for a short time, the lab is near enough to being inertial so that the effects of the Earth’s rotation are negligible, but if extremely high precision is required we need to take the Earth’s rotation into account. Again, this doesn’t invalidate special relativity at all.

    An undisputable assessment for the preferred frame approach.

    Not at all. It’s true that the equations of physics take a particularly simple form in terms of a special class of coordinate systems, called inertial coordinates, but this does not represent a “preferred frame”. Any inertial coordinate system is equally suitable. You don’t have to use the ECI to analyze Hafele-Keating; you will get exactly the same answer if you analyze it in terms of any inertial coordinate system, such as the inertial coordinates in which the center of mass of the solar system is at rest (which would show slight corrections due to the Earth’s non-inertial path around the Sun), or even the inertial coordinates in which the ground based clock is initially at rest… but of course in the latter case the ground based clock will soon no longer be stationary in those coordinates, so you must account for that. In practice we choose whichever inertial coordinate system is most convenient, and of course it only needs to be sufficiently close to perfectly inertial to support the required accuracy.

    if you shoot particles at slower and slower speeds, you will get increasingly stronger discrepancies with SR predictions, as shown by the HK experimental facts."

    No, aside from the gravitational effects, there are no discrepancies in the Hafele-Keating results with the predictions of special relativity.

    ReplyDelete
  124. Tam Hunt wrote: Lorentz actually came up with the Lorentz transformations (hence the name) and Einstein adapted them for SR, so Lorentz Relativity (LR) is not a variant of SR, it's vice versa.

    Lorentz's theory didn’t achieve complete empirical viability (including the correct form of the current density) until after Einstein (and Poincare) corrected him in 1905/6, based on a straightforward application of the principle of relativity. From that point forward, Lorentz’s fully corrected and amended theory is not a variant of special relativity, it is just a different metaphysical interpretation of the same theory, with all the same physical phenomena, contrary to your claim that Lorentzian relativity has different physical phenomena.

    Whereas there is a real physical mechanism for length contraction in LR (interaction with the ether in a similar manner as a ponderable body interacting with a fluid would experience drag), the mechanism for length contraction in SR is simply interaction with spacetime.

    Not true. Active length contraction is a consequence of Maxwell’s equations, and was already computed by Heaviside back in the 1880s, which is why his friend Fitzgerald was able to point it out as the explanation for the Michelson-Morley result, years before Lorentz. The properties of the electromagnetic field (and every other field, we now know) and the inertia of energy are the “real physical mechanism” for length contraction. By the way, Lorentz knew that the idea of ether “drag” causing length contraction was nonsense. His “ether” had no mechanical properties at all (other than immobility), which is why Poincare pointed out it violated conservation of momentum.

    Time dilation for Lorentz was, however, a mathematical fiction/artifact, and not a real physical effect…

    Not true. Speaking about his mature views, post 1909, his understanding of the physical phenomena of time dilation was identical to Einstein’s, and he clearly understood that time dilation is not a mathematical fiction. Again, two identically constructed ideal clocks following different paths between two events will show different elapsed times. This is true in any theory in which inertial coordinate systems are related by Lorentz transformations. Lorentz was well aware of this, and it obviously is not a mathematical fiction.

    Lorentz was explicit in his work that he saw "local time" (time dilation clock readings) as equivalent to changing time zones, that is, a coordinate effect only, a result of one's perspective, not a real physical change in the flow of time.

    That’s completely untrue. Again, Lorentz stressed that he had previously failed to appreciate the true physical significance of what he had first regarded as just a mathematical artifice. You need to learn about the difference between active and passive transformations.

    As for observed changes in clocks traveling at different speeds, the famous Hafele-Keating experiment was in fact heavily massaged, as a paper published in 2000, using their original data, made clear.

    Of course the data was massaged, as the effect was just at the limits of capability at the time. This is often the case for the first experimental tests of something. But the results were legitimate, and later repetitions with much more precise clocks have confrimed the results beyond all doubt, and of course we now have GPS clocks and so on.

    In my review of a number of papers purporting to find physical time dilation, all of what seem to be pretty serious problems.

    But your position (denial of all empirical evidence) makes no sense, because Lorentzian relativity predicts exactly the same results for time dilation (due to motion) as does ordinary special relativity. So if you deny things like time dilation then you are denying Lorentzian relativity as well.

    ReplyDelete
  125. @Amos

    Thank you for your explanations.

    Looking at the crucial elements, as given in your 2 last paragraphs, I have a question :

    (KVS)"if you shoot particles at slower and slower speeds, you will get increasingly stronger discrepancies with SR predictions, as shown by the HK experimental facts."

    (A) No, aside from the gravitational effects, there are no discrepancies in the Hafele-Keating results with the predictions of special relativity."

    The Special Theory of Relativity uses a relative speed v, ok. Shooting slow moving particles east and west in a linear accelerator along the equator, SR will give wrong predictions because it requires to use the same speed for both east and west shot particles, relative to the lab. And the goal was a comparison to the lab clock.

    The use of the eci is needed, along with the rotation speed of the earth, to get the correct predictions.The comparison to the lab clock can thus only be done correctly, after the use of the eci.

    >Am I to understand, according to your explanations above, that the latter practice is considered a part of SR then ?
    Because this is not included at all in the original theory, it's not 'relativity' as it was intended.

    Best, Koenraad

    ReplyDelete
  126. The Special Theory of Relativity uses a relative speed v, ok.

    I don’t know what you mean by that. Special relativity deals with speeds in terms of well-defined coordinate systems. It says the equations for the basic laws of physics take the same form when expressed in terms of any system of inertial coordinates, and that such coordinate systems are related by Lorentz transformations. Hence all physical phenomena are (locally) Lorentz invariant.

    Shooting slow moving particles east and west in a linear accelerator along the equator, SR will give wrong predictions because it requires to use the same speed for both east and west shot particles, relative to the lab.

    No, special relativity does not give wrong predictions (aside from the effects of gravity). For the brief duration of time required for a particle to move from one side of a lab to the other, the lab is nearly at rest in an inertial coordinate system moving tangent to the Earth’s surface. In terms of that inertial coordinate system, two identical particles emitted from rest with equal force in opposite directions will indeed have equal speeds, but of course they will not have equal speeds in terms of some other inertial coordinate system, in which they were not initially at rest. The predictions of special relativity have always been confirmed by experiment (again, neglecting gravity).

    The use of the eci is needed, along with the rotation speed of the earth, to get the correct predictions.

    No, we get exactly the same predictions in terms of any system of inertial coordinates, not just the ECI. It so happens that the ECI is a convenient inertial coordinate system for us here on Earth, but we could also use any other inertial coordinate system and get exactly the same results. For example, if you work out the Hafele-Keating elapsed times in terms of the inertial coordinates in which the Sun (or center of mass of the solar system) is currently at rest you get the same results, with a very small correction due to the fact that the ECI really isn’t exactly inertial due to the Earth’s revolving around the Sun. You can arbitrarily select any other inertial coordinate system and get the same results.

    Am I to understand, according to your explanations above, that the latter practice is considered a part of SR then ? Because this is not included at all in the original theory, it's not 'relativity' as it was intended.

    Again, it’s not necessary to use the ECI. Any system of inertial coordinates is equally suitable, so there is no “preferred frame”. Of course the rotation of the Earth must be taken into account for an experiment like Hafele-Keating, but I don’t know why you think this is “not relativity as it was intended”. It sounds like you are confusing relativity with Leibnizian relationism. Those are two very different things. As Einstein said, special relativity, no less than Newtonian mechanics, is based on the principle of inertia: The equations for the basic laws of physics take the same simple form when expressed in terms of any system of inertial coordinates.

    Neither Newtonian physics nor special relativity ever claimed that the equations of physical laws take the same simple form in terms of rotating coordinates (they obviously don’t, due to centrifugal and Coriolis terms, etc). For example, the speed of light is c in terms of any system of inertial coordinates, but obviously not in terms of rotating coordinates. Think about the speed of distant galaxies in terms of the Earth's rotating coordinates.

    ReplyDelete
  127. Lorraine Ford;

    I think you’re placing way too much focus on “the numbers”, obscuring the underlying reality. The changing numbers are a mere reflection of the changes in the underlying phenomena.

    Further, even the most abrupt changes occur within time. There is the period of time before the change and the period of time after the change. Time without change is ‘stasis’, change without time is absurd.

    I wrote (and you cited), “change is how we observe time”.

    You replied, “This is the wrong way around: it should be ‘time is how we observe change’. I.e. our sense of time derives from knowledge of (what we would represent as) number change.” (emphasis added.)

    If “our sense of time derives from knowledge of … [any] change” that means change is how we observe time!

    Whatever actual “sense of time” we have is due to some internal “clock” (for lack of a better word) which tracks time through changes. Again, this means change is how we observe / sense time.

    sean s.

    ReplyDelete
  128. bud rap;

    Science is the study of observable (broadly inclusive of our various mechanical extensions of our senses), physical reality. ... Science rests on empiricism - it is the study of the phenomenological world.

    Science does rest on empiricism, but not only on empiricism. The scientific method includes observation, experiment, and reason; it includes deductive and inductive methods. As long as evidence and reasoning are treated with proper care, there’s no reason to draw the line where you want us to.

    Your idiosyncratic definition of science would put a lot of legitimate scientific topics outside science, for no apparent reason. The four forces cannot be directly observed, and nearly nothing in QM can be directly observed.

    You are free, of course, to insist on your peculiar and personal definition of science, and the rest of us are free, of course, to reject it. Which for the most part, we do.

    The attempt to conflate empirical evidence (direct observation) with a model-based inference derived from the empirical observation of other phenomena, is as disingenuous as it is common.”

    It’s common because the practice is accepted in the community of scientists and rational people as valid—because it is valid. As long as the nature of the evidence is well understood and properly treated, there’s no problem. Relatively weaker evidence is not “no evidence”.

    The two forms of evidence are not the same and therefore, should not be weighted the same.

    Oh, we agree, and I’ve never said otherwise, but neither should any evidence be disregarded on a mere metaphysical basis (as you appear to).

    I wrote, “Since time exists, TH’s definition cannot reify it.

    You replied, “Circular arguments like this are logical errors.

    Your argument is that time does not exist, therefore TH’s definition reifies it.; this is no less “circular” than mine.

    Doing that sort of thing speaks to the weakness of your own arguments.

    Failing to respond meaningfully speaks to the weakness of your arguments. Humans do not experience “temperature”, they experience heat and changes in heat. (This is a tangential topic here, obviously, but you brought it up.)

    sean s.

    ReplyDelete
  129. @Amos

    Thank you for taking the time to explain further.

    'For the brief duration of time required for a particle to move from one side of a lab to the other, the lab is nearly at rest in an inertial coordinate system moving tangent to the Earth’s surface...

    Brief, nearly, these ate not neglectable details, and if you shoot the particles at everyday speeds, the prediction fails.

    Further, the fact remains that in order to get the difference with the lab clock correct, you need to pass by the eci, or another as you state. That's not Special Relativity, the fact remains that sr fails to give correct result when asked to compare the lab clock to clock A and B, it doesn't work without invoking another frame, that's not what SR originally stood for.

    Sure, you can expand the theory to include these effects you quoted, and still call it Special Relativity, but then you minimise the significance of these effects, they tell us something about the larger context. Which becomes important for a theory of quantum gravity, probing which microprocesses are at the basis of it. Physicist were successful in constructing the laws, now we want to find out why they take the form they do. Why, as in what mechanisms, for lack of a better word, hide behind them.

    Best Koenraad

    ReplyDelete
  130. @sean s: "I doubt the distinction between intuitions and concepts is meaningful here; and with respect to Kant, he never knew about GR and the evidence of how time operates under it."

    Yet that distinction is exactly what is being debated here. (And which distinction is unresolved since well before Kant.) It might be useful to know that Kant have very precise meanings for "intuition" and "concept," and the former especially isn't per the common usage.

    In any event, I find his analysis useful even in the context of GR. Specifically that time is immediate and singular. His a priori analysis gives emergent time a very tough row to hoe. (As I mentioned, it has a lot to do with why I find the idea incoherent.)

    As I've also mentioned, I see the connection between time and GR (and SR) as decidedly secondary. Causality is always respected, and that, to me, is the primary consideration. No matter how time flows in different frames, causality is never violated.

    It's entanglement that strikes me as Hey, wait a minute...

    ReplyDelete
  131. @bud rap, you're not understanding my definition of time. You state above in response to Sean S. that I define time "as a physical entity that causes change." That's not at all my definition. As I stated clearly to you: I define time as change. It's an identity, that is, they're identical and the terms are interchangeable. There is no reification, there is simply your misunderstanding. Change is real, we've both agreed, and must be explained if we hope to explain the world in a coherent way.

    ReplyDelete
  132. @sean s,

    Your last reply is a master class in straw man argumentation:

    [ss] Science does rest on empiricism, but not only on empiricism.

    I did not say it only rested on empiricism.

    [ss] Your idiosyncratic definition of science would put a lot of legitimate scientific topics outside science, for no apparent reason. The four forces cannot be directly observed, and nearly nothing in QM can be directly observed.

    You are free, of course, to insist on your peculiar and personal definition of science, and the rest of us are free, of course, to reject it. Which for the most part, we do.


    I did not offer a definition of science. (BTW, is that an imperial "we" or do you have a tapeworm?)

    [ss] Relatively weaker evidence is not “no evidence”.

    I did not say it was. What I said, very clearly with regard to the Higgs was: Direct empirical evidence is far stronger than inferential evidence. So, my position on the Higgs is that there is no compelling evidence for its existence. No compelling evidence is not the same as no evidence.

    [ss] Your argument is that time does not exist, therefore TH’s definition reifies it.; this is no less “circular” than mine.

    I did not make that argument. My argument is as follows:

    1) There is no empirical evidence for the existence of a substantival "time" that causes physical events.

    2)The evidence only supports the existence of time as a relational concept.

    3)Therefore when you or TH appear to assert that time is "real" in the substantival sense you are reifying a relational concept.

    [ss] Failing to respond meaningfully speaks to the weakness of your arguments. Humans do not experience “temperature”, they experience heat and changes in heat. (This is a tangential topic here, obviously, but you brought it up.)

    1) I did not bring up the temperature vs heat distinction, you did. I said that temperature is a relational concept - like time.

    2) I do not respond to nonsensical, semantic diversions.

    Lastly, it would be helpful if you could say something indicating that you at least comprehend the substantival vs relational distinction that is at the core of our disagreement. To this point, your responses have been unclear on the matter.

    ReplyDelete
  133. ”For the brief duration of time required for a particle to move from one side of a lab to the other, the lab is nearly at rest in an inertial coordinate system moving tangent to the Earth’s surface...”

    Brief, nearly, these are not neglectable details…


    Well, the effects either are or are not negligible, but in neither case is there any violation of special relativity. We know how fast the earth is rotating, and we know how long it takes a particle to move from one end of the lab to the other, and if the effect of the Earth’s rotation is negligible, then it is negligible. Otherwise we must account for it. Again, there is no violation of special relativity.

    and if you shoot the particles at everyday speeds, the prediction fails.

    What prediction fails? Aside from the effects of gravity, no prediction of special relativity is known to fail. I think you simply misunderstand what special relativity predicts.

    Further, the fact remains that in order to get the difference with the lab clock correct, you need to pass by the eci, or another as you state. That's not Special Relativity…

    It is special relativity. The first principle in the first section of the first paper Einstein wrote on special relativity is that the equations of physics take the same simple form in terms of any system of inertial coordinates. This was carried over from Galilean relativity over 300 years ago. Again, you seem to be fundamentally confusing relativity with relativism. Despite the similarity of the words, those are two very different things.

    the fact remains that sr fails to give correct result when asked to compare the lab clock to clock A and B, it doesn't work without invoking another frame, that's not what SR originally stood for.

    No, none of what you have stated is a fact. Aside from the effects of gravity, special relativity gives exactly the correct elapsed times on all the clocks. The problem here is that you have a mistaken idea of “what SR originally stood for”. It never stood for what you thought it did. Relativity is not relationism. Velocity is relative but acceleration is absolute. This is and always was “what SR stood for”. Of course, after 1915 and general relativity it's understood that special relativity doesn't account for the effects of gravity, so inertial coordinate systems can in general be defined only locally, but that doesn't seem to be your complaint.

    Sure, you can expand the theory to include these effects you quoted, and still call it Special Relativity…

    I’m not expanding special relativity. What I’m describing is precisely what special relativity has always meant. It never claimed that rotating or accelerating coordinate systems are indistinguishable from inertial coordinate systems. That would be absurd. Granted, some pop-science accounts might give that impression, but that's why people shouldn't rely on those.

    ReplyDelete
  134. sean s,

    I disagree with your way of looking at time and change. You seem to agree with Lee Smolin’s book Time Reborn [1]. Five years ago I thought I agreed with Lee Smolin’s book Time Reborn, but now I think Lee Smolin is barking up the wrong tree.

    Time is not necessary at a fundamental level: what is necessary is the capacity to change, or create new, (what we would represent as) numbers, mathematical relationships and algorithms. This necessary capacity to change, or create the new, does not require time for permission to exist or operate. (This change/creation of the new is only seen in quantum events.) Time is superfluous at a fundamental level because time does not cause or create change.

    Of course, time is close-to-fundamental. But time is different to a truly fundamental variable because time can’t be seen as (what we would represent as) a mathematical relationship between other such variables. Time can only be seen as (what we would represent as) an algorithmic relationship that determines whether or not existing variable numbers have changed. The time variable represents close-to-fundamental level information: the fact that (what we would represent as) a number for a variable has changed.

    Lee Smolin’s view is also wrong because he seems to somehow equate time to perception/subjective experience [1]. But what living things experience is like algorithmically-derived and collated, high-level information about their situation in the universe, derived from more fundamental-level information (where the fundamental-level information is representable as numbers for one or more variables, and as relationships between these variables). Subjective experience is information, including time/change information.

    In summary, Lee Smolin seems to be saying that: 1) time is somehow a container which allows otherwise unexplained, miraculous change to happen; and 2) time somehow equates to subjective experience. Lee Smolin seems to give time powerful characteristics which it can’t possibly have.

    ........

    1. Physicist Lee Smolin says in the book: “I believe…Time will turn out to be the only aspect of our everyday experience that is fundamental. The fact that it is always some moment in our perception, and that we experience that moment as one of a flow of moments, is not an illusion. It is the best clue we have to fundamental reality."(p xxxi) and “Human beings perceive time as change” (p28).

    ReplyDelete
  135. @Amos

    ' The problem here is that you have a mistaken idea of “what SR originally stood for”. '

    You are correct,I understood the effects and mechanisms and experimental results at hand in all of the above context, but was wrong about which items actually belong or don't belong within SR.

    You have cleared up a very long quarrel I had with interpreting SR in therms of its actual relevance.

    Your explanations were to the point and actually adressed the content of issues raised. In many other cases explanations remain to general.

    My thanks.

    Best, Koenraad

    ReplyDelete
  136. Time=change does not seem satisfactory to me due to two problems that I perceive with it:

    1) At velocity c, all changes happen instantaneously, so there is change without time.

    2) Suppose that there is a completely isolated system containing some radioactive atoms. Nothing happens, until suddenly one of the atoms emits a beta particle. Then nothing, until a second atom emits a beta particle. Meanwhile, in the rest of the universe, things have been happening, unknown in the isolated system. Has no time elapsed in the isolated system except when the radioactive decays occurred?

    Time=change (including clock-ticks) is sufficient for our daily lives, but to be a good scientific definition it should encompass all possibilities, it seems to me.

    So what is time? Well, if this universe were a simulation being run on the unimaginably-powerful computer of some higher universe, time would be the clock cycles of that computer; or maybe this universe is somehow simulating itself; in short, I don't know.

    Time is how we measure the rate of change? (Relatively, that is the rate of some change relative to the rate of some other change.) (A happens N times while B happens M times, so the rate of A relative to B is N/M.)

    ReplyDelete
  137. @Tam Hunt,

    OK, I misunderstood you to the extent that I interpreted statements such as, "...the passage of time as the fundamental container of existence" and "...viewing the passage of time as fundamental is not reification. It's basic empiricism.", as implying some level of causality. If you are saying that time is not a substantival entity that causes things to happen, but merely a relational concept, then we are in agreement.

    However, I cannot see any justification for conflating or identifying the concept of change with the concept of time. While both are relational concepts, change describes a broad range of manifestly unrelated, but observable, phenomena. You can change your socks, you can change your weight by dieting, you can change the light level in a room by flipping a switch, you can increase the temperature in that room.

    All of those changes are measured in units unique to each type of event, none of which are units of time. When the temperature in a room changes from 60 degrees F to 70 degrees F, the rate of change is only a secondary, (time) derivative of the change in temperature.

    If you say something changed over the course of two hours but do not specify what changed, you've said nothing physically meaningful. Therefore, time and change are not the same thing.

    ReplyDelete
  138. Carlo Rovelli says:

    But change is far more general than time. We can have “change,” namely “happenings,” without any possibility of ordering sequences of these happenings along a single time variable” and “There is no time in the basic laws of physics”.

    But Rovelli is wrong to imply that “change” can have no ordering sequence or direction: it all depends on what “change” is. So it is important to more clearly define what “change” is: the word “happenings” doesn’t tell you anything useful.

    Leaving aside the abrupt change that occurs in quantum events, physics represents reality as equations and numbers, so you’ve only got 2 choices for “change”: change of number for a variable, and change of law of nature equation. So no matter what the fundamental-level situation, “change” will usually mean change of number for a variable.

    So one might ask: “why are the numbers changing?”, i.e. “what is causing (what we would represent as) number change in the universe?”. And the answer is obvious: quantum events cause number change; and quantum events have only one direction. Seemingly, when traced back, quantum events are the only possible driver of all number change in the universe; and quantum events are the reason that number change has only one direction.

    ReplyDelete
  139. @Amos, my sources for understanding Lorentz and the history of his, as well as Einstein's, theorizing about relativity are primarily Galison's book, Einstein's Clocks, Poincare's Maps, as well as Lorentz's papers, and Einstein's own books, papers and Isaacson's excellent biography of Einstein (as well as many other sources that don't need to be listed). All of these sources clearly describe the evolution of relativity from Lorentz's original 1895 explanation of the apparent null result of the Michelson-Morley experiment through explanation of these phenomena by use of the Lorentz transformations and an ether that interacts with matter and thus causes length contraction, as velocity of the matter at issue increases in relation to the ether.

    Lorentz was clear that his "local time" that resulted from the apparent time dilation in the Lorentz transformations was a coordinate effect only, a mathematical fiction.

    So, yes, both Einstein and Lorentz include the same formalisms with respect to length contraction and time dilation but as I've already suggested they interpreted the formalisms rather differently. And Lorentz continued to argue for the ether concept until his death, eventually getting Einstein to concede that some kind of ether was in fact necessary to explain the phenomena (see Lukwik Kotro's book Einstein and the Ether for this particular history).

    I'm puzzled by your suggestion that interaction with the electromagnetic field was the physical basis for length contraction in Einstein's SR. See Callender 2007 for explicit discussion of this issue, stating that it was "spacetime structure" responsible for length contraction in SR and GR. Callender writes: "On this view, Lorentz invariance is not a spacetime symmetry but a dynamical symmetry, and the special relativistic effects of dilation and contraction are not purely kinematical. The background spacetime is Newtonian or neo-Newtonian, not Minkowskian." https://books.google.com/books?hl=en&lr=&id=gDC52yTjI6sC&oi=fnd&pg=PA50&dq=callender+real+time&ots=xjhHAoRcYZ&sig=JWEtYerwJnGeURI8qKC_tdtOdpA#v=onepage&q=callender%20real%20time&f=false

    You seem to state that Lorentzian relativity compares two inertial frames, and thus is equivalent to SR, but this is not the case because of course Lorentz viewed relativistic effects as being caused by interaction with the ether, expressly not an inertial frame. Have you read Mansouri and Sexl 1976?

    No, physical time dilation was not apparent in Lorentz's era as no experiments that I'm aware of before his death purported to find different clock readings resulting from different motions of clocks (experimental evidence for time dilation and contraction came much later). Lorentz suggested that his equations may lead to an apparent difference in clock readings (which would be tested in much later experiments), but that any difference in clock readings would result from a difference in reference frame only, a different vantage point for taking such measurements, and would not be a real physical time dilation effect.

    Lorentz viewed time dilation as a “mathematical fiction” or “coordinate effect,” not a real physical effect like length contraction. A coordinate effect is, for example, like changing time zones when traveling. When we change time zones there is no real loss or gain of time. Time passes continuously no matter what time zone we’re in and we don’t literally gain or lose an hour as we change time zones. Rather, each time zone is just a different convention for keeping track of the same shared passage of time. Just so with the time dilation of the Lorentz transformations: the “local time” of each frame of reference is a way to keep track of time between different frames of reference, but global time proceeds independently of the conventions used for measuring the local time. Galison writes:

    (cont. below)

    ReplyDelete
  140. (cont.)

    In SR, however, there is no global time and the apparent passage of time itself is rendered an illusion. This is the case because if time is malleable and the speed of light absolute, then there is no privileged time and no universal “now.” We can slice the universe into an infinite number of possible “nows” depending on the speed at which we move in relation to the distribution of matter and energy in our universe.

    Lorentz was of course explaining the apparent null result of the M-M experiment by invoking interaction with the ether, a very different explanation than Einstein offered with his postulated invariant c (and thus malleability of time and space to allow for invariance of c).

    In terms of comparing proximate clocks that have gone through different acceleration, as in Hafele-Keating, the original experiment was apparently fatally flawed, as discussed in detail by Kelly 2000 and in my in-progress paper that I've already linked to. The data was heavily massaged so its conclusions are not valid. I'd be happy to look over other similar experiments if you send me a citation.

    In terms of comparing distant clocks and purported relativistic time dilation, as in GPS, for example, also see my appendix in my in-progress paper for a discussion of Wolf & Petit 1997, a classic paper looking at the invariance of c using GPS clock data. My analysis suggests that they end up with a tautological methodology (stemming from Einstein's synchronization technique that assumes invariant c), despite trying to be careful to avoid such tautology. Would appreciate your feedback on my analysis.

    You state: "But your position (denial of all empirical evidence) makes no sense, because Lorentzian relativity predicts exactly the same results for time dilation (due to motion) as does ordinary special relativity. So if you deny things like time dilation then you are denying Lorentzian relativity as well."

    You seem to be missing the key point of Lorentz's view that time dilation was a fiction/artifact that made the equations simpler, but was not a real physical effect. My point in my paper is that where distant clocks seem to show time dilation it seems to result either from methodological issues or tautological assumptions, and thus time dilation is a fiction. And where clocks starting together and accelerated seem to show time dilation it's been due to inappropriate massaging of the data. Check out my paper. I'd appreciate feedback. This issue really comes down to the opposite postulates of LR vs SR: invariant c only with respect to the stationary ether and ether interaction as the basis for relativistic effects for LR (rendering predicted length contraction real but time dilation not); and a postulated invariant c (and no ether) as the basis for such effects in SR (rendering predicted length contraction and time dilation both real).

    (cont. below)

    ReplyDelete
  141. (cont.)

    A couple more good quotes on this debate:

    J. S. Bell, the colorful Irish physicist who formulated the Bell inequalities, which formed the theoretical basis for Aspect’s non-locality experiments, was a supporter of the Lorentzian view. He stated in a 1986 interview with physicist Paul Davies (Davies 1986):

    [T]he pre-Einstein position of Lorentz and Poincaré, Larmor and Fitzgerald was perfectly coherent, and is not inconsistent with relativity theory. The idea that there is an aether … is a perfectly coherent point of view. The reason I want to go back to the idea of an aether here is because … the suggestion that [in nonlocality experiments] behind the scenes something is going faster than light. Now if all Lorentz frames are equivalent, that also means that things can go backward in time … [This] introduces great problems, paradoxes of causality, and so on. And so it is precisely to avoid these that I want to say there is a real causal sequence which is defined in the aether.

    Yuri Balashov, a philosopher at the University of Georgia, stated in Balashov 2000:

    [T]he idea of restoring absolute simultaneity [which is the basis for the Lorentzian interpretation of relativity theory] no longer has a distinctively pseudo-scientific flavor it has had until very recently. It is a well-known fact that one could accept all the empirical consequences of SR (including length contraction, time dilation, and so on) and yet insist that there is a privileged inertial reference frame, in which meter sticks really have the length they have and time intervals between events refer to the real time.

    ReplyDelete
  142. @bud rap, it is you who are now reifying time by your statement that time can only be measured in units you're familiar with. Time can be measured by whatever physical phenomenon you want to use, including changing of socks, as in your example. Again, I have defined time as simply change. Period. I'm trying to explain change/time, which may be viewed as the same thing. Check out Popper's book World of Parmenides for some good background on these issues.

    ReplyDelete
  143. @JimV, you state two problems you see with the notion of time as change:

    1) At velocity c, all changes happen instantaneously, so there is change without time.

    2) Suppose that there is a completely isolated system containing some radioactive atoms. Nothing happens, until suddenly one of the atoms emits a beta particle. Then nothing, until a second atom emits a beta particle. Meanwhile, in the rest of the universe, things have been happening, unknown in the isolated system. Has no time elapsed in the isolated system except when the radioactive decays occurred?

    Addressing your first point: this is only the case if we assume invariant c regardless of the speed of the observer, which Lorentz didn't agree with and which I am suggesting is problematic for a variety of reasons. For Lorentz and others c was invariant only with respect to the stationary ether, rendering it variant for observers moving in relation to the ether, just as all other velocities are variant with respect to moving observers (speed of sound, speed of cars, speed of bullets, etc.). So this approach takes away the strangely special/weird nature of the speed of light and harmonizes it with the behavior of other physical phenomena.

    Addressing your second point: there are no completely isolated systems in nature so the hypothetical isn't physically realistic. In the real world there is always change at all levels. However, if we acknowledge (as I would) that there is a smallest granularity of change/time, then we could argue that below that granularity the passage of time/change effectively stops, until the next tick of an implicit universal clock.

    And that brings me to a more global comment: even though I've argued so far that time = change, because that's a simpler argument for conveying my key points, at a deeper level I would argue for a clock cycle/absolute notion of time as an implicit physical process that proceeds independent of any relational physical changes. Under this approach, the universe is re-creating itself in each moment, with each tick of the most granular chronon. This idea has strong precedent in Whitehead's process philosophy, even though there is significant debate in the process philosophy field as to whether there is an absolute time or only a relational notion of time is better. See David Ray Griffin's edited volume Physics and the Ultimate Significance of Time for more on this.

    ReplyDelete
  144. @Tam Hunt

    Back to your answer, concerning GPS experiments:
    This technique does provide a concrete method for defining simultaneity
    and thus for synchronizing distant clocks, but we must be careful to not
    use this technique and then forget that we have from the outset assumed
    an isotropic c in order to achieve synchronization.


    By isotropic c, do you mean the proposition that C is constant, regardless of your own velocity? You also seem to doubt this in your answer to JimV:

    this is only the case if we assume invariant c regardless of the speed of the observer

    But the speed of light has been measured with very high precision (up to a single m/s) in experiments with lasers. If any experiment could have demonstrated that C depends on the speed of the observer, it would have been major news, worthy of a Nobel prize.

    But that didn't happen. Experiments have confirmed, up to a high precision, that the speed of light in a vacuum is constant and independent of the speed of the observer. I am confused why you call it an assumption. Am I misunderstanding your point?

    ReplyDelete
  145. Lorraine Ford said "But Rovelli is wrong to imply that “change” can have no ordering sequence or direction: it all depends on what “change” is."

    I liked the rest of the comment, but that part is I think unfair to Dr. Rovelli. Again, it is a question of semantics:

    1) "change can have no ordering sequence" = "there are cases where two observers in different reference frames will disagree on the ordering sequence", or
    2) "---" = "it is impossible to ever have any order or direction of events".

    It seems to me that Dr. Rovelli clearly meant (1), following Einstein, that is, Special Relativity does say that different observers could (not always, but could) order events differently.

    Of course, if you think SR is wrong, as apparently Tam Hunt does, and "time" only ever behaves as we experience it in our daily lives, then from your point of view Dr. Rovelli (and Einstein) are wrong.

    In my long-ago E&M course it was shown how Special Relativity could be derived from a few basic and very believable assumptions. The night after the professor introduced the concepts, I stayed up making my own derivation which I proudly showed the professor during his office hours the next day, while he ate a ham sandwich for a delayed lunch (having had many student visitors that day). He did not seem very impressed, and had probably seen a lot of similar efforts in his time, but found nothing wrong with it. Anyway, I have a long-standing respect for SR, myself.

    ReplyDelete
  146. All of these sources clearly describe the evolution of relativity from Lorentz's original 1895 explanation …an ether that interacts with matter and thus causes length contraction…

    Lorentz’s 1895 was unsatisfactory for reasons pointed out by Poincare (and others) and acknowledged by Lorentz, and among the problems was the (then) ad hoc and unexplained hypothesis of length contraction. By 1904 Lorentz had realized that (as Fitzgerald had already noted) if all forces, including those responsible for maintaining the structure of matter, have the same form as Maxwell’s equations of electromagnetic forces, then length contraction is the inevitable result. “If we suppose each particle of a solid body to be in equilibrium under the action of attractions and repulsions exerted by its neighbors, then if the velocity v is imparted to it, the body will of itself change… the translations will produce the deformation.” Then italics are Lorentz’s. There is no mention of any “interaction with the ether” is his discussion. He used it purely in his thinking only to give a definite meaning to “velocity v”, but he later realized (after Einstein explained it to him) that this was operationally superfluous, so the adherence to an absolute frame became purely metaphysical (as Lorentz himself said).

    Lorentz was clear that his "local time" that resulted from the apparent time dilation in the Lorentz transformations was a coordinate effect only, a mathematical fiction.

    This is simply not true at all. You must be thinking about 1895 Lorentz, not 1904 Lorentz, and certainly not 1909-15 Lorentz: “The chief cause of my failure was my clinging to the idea that the variable t only can be considered as the true time and that my local variable t’ must be regarded as no more than an auxiliary mathematical quantity… Attention must now be drawn to a remarkable reciproicity that has been pointed out by Einstein… it would be impossible to decide which of [two frames] moves or stands still with respect to the ether, and there would be no reason for preferring the times and lengths measured by one to those determined by the other, nor for saying that either of them is in possession of the “true” times or the “true” lengths.”

    Lorentz continued to argue for the ether concept until his death…

    Sure, but he acknowledged that it was a metaphysical preference, and his reasons (as you can see in private correspondence with Einstein) were partly that he thought Lorentz invariance might fail at some point (it hasn’t), and partly a quasi-religious notion about the sensorium of god (to borrow Newton’s phrase).

    ..eventually getting Einstein to concede that some kind of ether was in fact necessary…

    But the “some kind of ether” is just the spacetime of general relativity, with no mechanical properties at all, and to which no state of motion or rest can be assigned, so it is completely different than the 19th century mechanistic ether than you espouse. This does not support your beliefs at all.

    You seem to state that Lorentzian relativity compares two inertial frames...

    Yes, the Lorentz transformation is, by definition, the relationship between operationally-defined inertial coordinate systems, from which all the relativistic effects follow.

    I'd be happy to look over other similar experiments if you send me a citation.

    Check the Wikipedia page on Hafele-Keating, in the section titled “Repetitions”.

    My analysis suggests that they end up with a tautological methodology (stemming from Einstein's synchronization technique that assumes invariant c)…

    The theory of relativity (whether you espouse the Lorentzian or the Einsteinian interpretation) is not tautological, and the predicted physical effects have been thoroughly and conclusively verified to the achievable precision.

    ReplyDelete
  147. @doctor Boktor, invariant c regardless of the speed of the observer is a postulate of special relativity and cannot be proved (how would you prove it?). That was kind of Einstein's point: since we can't know the two-way speed of light, since we know of nothing faster by which we would measure the two-way speed of light, if we simply assume c is invariant then we can use it as the basis of a system of establishing the relative time of distant events, by using light signals for synchronizing or comparing different clocks. For this to work we have to assume that c is invariant.

    The price we pay for using light signals for synchronization, if we take that methodology to be ontologically significant, is the loss of any real notion of absolute simultaneity (a common now between distant observers), and all of the other weird and mind-bending consequences of special relativity that have been discussed for decades now.

    OR, a very big or, we can instead recognize the utility of Einstein synchronization but not take it to have ontological significance. Instead, we view interaction with a immobile ether as the basis for an invariant c (c is invariant only with respect to the ether), allowing us to preserve the empirical equivalence of SR and this Lorentzian relativity, but dispense with all of the counter-intuitive and problem-inducing consequences of viewing time dilation as ontologically real.

    This means that we can use light signals to synchronize distant clocks but we recognize that it's just a tool, not something that literally changes time physically. Time dilation is under this approach reduced to a coordinate effect like the similar tool of time zones: time doesn't literally change, it's just a different way of telling time.

    Now, there's a big complicating factor here with quantum nonlocality in that there may well be ways to use quantum nonlocality to establish the two-way speed of light in difference circumstances. This is why there's been talk for decades now about apparent empirical issues between nonlocality and SR. In fact, nonlocality is defined simply as something that appears to defy the speed of light limit. So if we could somehow manage to use nonlocal influences to measure the two-way speed of light, that would be a game changer. But the general view is that nonlocality can't be used for signaling or communication. My feeling is (without having delved deeply enough into this issue yet) that this conclusion mostly stems from the assumption that anything that contradicts SR must automatically be wrong and forbidden, and that's the position I've been arguing against here since we do in fact have alternatives to this standard interpretation of the Lorentz transformations.

    ReplyDelete
  148. @Tam Hunt

    ...it is you who are now reifying time by your statement that time can only be measured in units you're familiar with.

    I made no such statement. This disingenuous argumentative technique in which you have engaged, is simply a transparent attempt to divert attention from an argument, to which, you are ill-equipped to respond. It is a diversionary technique that indicates you don't understand your own position well enough to defend it properly. I will object, very specifically, every time you attribute to me a statement or argument I did not make. I'll do the same if I see you do it to anyone else. I suggest you refrain from all such behavior lest you acquire a reputation for mendacity.

    You wish to "define" time as identical to change. I'm simply pointing out to you the following argument that refutes the viability of your idiosyncratic definition:

    1) Change is a relational concept (like time). Change is not a thing that causes something to happen.

    2) The units of change vary with nature of the change. Changes in temperature are measured on a temperature scale, changes in mass are measured on a mass scale, changes in the distance between objects are measured on a length scale, etc.

    3) The local start/stop time of such events, or the rate of change, require time units to be added to the description, but the time units do not describe the change itself. The statement, the temperature in a room changed from 60F to 70F is meaningful in itself. The statement that some otherwise unspecified change in some unspecified state occurred between 3pm and 4pm is essentially meaningless.

    4) Therefore time and change are not the same thing, despite your personal desire to declare them so.

    ReplyDelete
  149. Tam Hunt,

    You mention Process Philosophy [1]. Process philosophy has very likely influenced the views of scientists like Lee Smolin and Carlo Rovelli with its notions of “we experience our world and ourselves as continuously changing” [2]. I don’t entirely disagree with the ideas of process philosophy; but the trouble with process philosophy is that it seems to assume that “change” is a primitive of the universe, a first principle which cannot be questioned or further analysed.

    1. 2:28 AM, September 09, 2018
    2. https://plato.stanford.edu/entries/process-philosophy/

    ReplyDelete
  150. JimV,

    I agree that that part of my comment might have been slightly unfair to Dr Rovelli. I agree that "there are cases where two observers in different reference frames will disagree on the ordering sequence". I have no disagreement with Special Relativity.

    But what I’m objecting to is the notion of “change” as a primitive of the universe, a primitive that cannot be further analysed. Irrespective of SR, I’m asking a more general question of why (what we would represent as) numbers change.

    You pointed out that “if this universe were a simulation being run on the unimaginably-powerful computer of some higher universe, time would be the clock cycles of that computer”[1]. In such a situation, the algorithmic computer program and the electricity powering the computer would provide a reason for the changing output numbers. But in our actual universe the numbers are changing, and clearly there must be a reason/cause for this change, and the law of nature relationships cannot cause number change.

    1. 11:31 AM, September 08, 2018

    ReplyDelete
  151. invariant c regardless of the speed of the observer is a postulate of special relativity and cannot be proved (how would you prove it?)

    By measuring the speed of light while moving relative to the source? Maybe this was debatable at the time Einstein formulated his theory, but by now the speed of light has been measured with great precision under many circumstances, including moving light sources.

    Now, you could argue that this does not prove that the speed of light is exactly constant for an observer at any possible speed, because you would need an infinite number of experiments.

    But if I understand you correctly, you are claiming that the speed of light is equal to C + v, where v is your speed relative to some ether. Again I wonder if this is really what you mean? Because that is something which has already been demonstrated very clearly to be false by measurements.

    The price we pay for using light signals for synchronization, if we take that methodology to be ontologically significant, is the loss of any real notion of absolute simultaneity (a common now between distant observers), and all of the other weird and mind-bending consequences of special relativity that have been discussed for decades now.
    Among physicists this is not controversial. Please note that while absolute simultaneity is lost, causal order and the arrow of time remain absolute (because past and future lightcones are invariant under Lorentz transformations).

    ReplyDelete
  152. Lorraine Ford, thanks for the reply. Other than that possible misunderstanding, I have followed your comments and found them meaningful.

    Would a Gnome Ann reference be appropriate here? Probably not, but here goes: like xkcd's Gnome Ann, you have boldly gone before where I have yet to go.

    ReplyDelete
  153. Tam Hunt wrote: The price we pay for using light signals for synchronization…

    The use of light signals is not essential; we get exactly the same synchronization using mechanical inertia or any other physical process. The equations of every physical law - not just electromagnetism but mechanics as well - take the same form in terms of any system of inertial coordinates. This was shown by the failure of the “ether wind” experiments, which were designed to detect a difference between the (presumed) Galilean relativity of mechanics and the Lorentzian relativity of electomagnetism and optics. The absence of any difference shows that mechanics and electromagnetism (and all other physical laws) are Lorentz invariant, not Galilean invariant, and this has hugely significant physical consequences. You can read about this, and the underlying reason for it, in any good book on relativity (using the true Scotsman definition of “good book”).

    Invariant c regardless of the speed of the observer is a postulate of special relativity and cannot be proved (how would you prove it?).

    It’s an empirical fact that the speed or light in vacuum is c in terms of any system of inertial coordinates, meaning coordinates in which the equations of Newtonian mechanics hold good in the low speed limit.

    That was kind of Einstein's point: since we can't know the two-way speed of light…

    That’s garbled. The two-way speed of light (with the distance measured by stationary rulers) can be measured with a single clock, and doesn’t involve any distant synchronization. But even after we’ve established that the two-way speed is always c, there remains the question of the one-way speed, since there are self-consistent coordinate systems in which the one-way speed varies even though the two-way speed is always c. However, in terms of inertial coordinate systems, operationally defined as above, the one-way speed of light is empirically c.

    …allowing us to preserve the empirical equivalence of SR and this Lorentzian relativity, but dispense with all of the counter-intuitive and problem-inducing consequences of viewing time dilation as ontologically real.

    Hold on, your whole position is based on the denial of the physical reality of time dilation, i.e., you claim that actual physical time dilation is a false prediction of special relativity but not of neo-Lorentzian relativity, and yet now you are invoking empirical equivalence between those two interpetations. You can’t have it both ways. Neo-Lorentzian relativity and special relativity are just two interpretations of the same theory, with all the same physical phenomena and predictions, so your claims are self-contradictory. Also, you keep alluding to “problem-inducing consequences” of special relativity, but you haven’t identified any. The physical phenomena of neo-LorentzIan relativity are identical to those of special relativity, so it is no more or less “weird and mind-bending”.

    Time dilation is under this approach reduced to a coordinate effect like the similar tool of time zones: time doesn't literally change, it's just a different way of telling time.

    Again, your beliefs are based on denial of the facts, e.g., the different elapsed proper time along different paths between two events… not to mention E=mc^2 and all the other consequences of the inertia of energy. Lorentz invariance is not a mathematical fiction nor an artifact of a peculiar choice of coordinate systems. It represents real physical phenomena, and it can be (and has been) thoroughly tested to high precision.

    ReplyDelete
  154. @Lorraine Ford, change is indeed at the heart of process philosophy (hence the name) but it is not the case that change is simply taken as a given and not further analyzed. Whitehead's magnum opus, the 1929 Process and Reality, describes in detail various aspects of his "philosophy of organism" (which is what we now call process philosophy as a more general rubric), including a genetic analysis of concrescence and the creative advance. His later books add further color. We don't need to delve deeply into process philosophy here, however, since it's not essential for the current discussion, but if you're curious and have a lot of time and patience you will enjoy the deep dive into Whitehead's work.

    ReplyDelete
  155. @doctor Boktor, there's a lot to say about measuring the speed of light, but here's the gist: measuring the 2-way speed of light rests on the assumption that the speed is the same out and back, which is begging the question in terms of the invariance/isotropic nature of c. No other physical phenomena display this puzzling feature so why would light? Well, it does in SR as a postulate simply b/c it's operationally useful to assume that this is the case and it ostensibly explained the apparent null result of the Michelson-Morley experiment. Measuring the one-way speed of light is similarly difficult b/c it requires synchronizing distant clocks. And how is this synchronization done? Well, usually with light signals, which must go out and back to achieve said synchronization, and the speed of light in this technique is assumed to be isotropic (same both ways), which again begs the question. Are you starting to see the problem? This is part of what's known as metrology today and when you dig deep you realize that any system of measuring the universe has to start with some postulate(s). Invariant c is one possible postulate for this brute fact but it leads to many problems, such as the ones I describe in my paper.

    The simple alternative is the Lorentzian approach, which assumes invariant c with respect to the preferred frame only (ether or whatever name you prefer).

    Here's a juicy and useful quote from Dr. Einstein himself in his 1955 popular book on special and general relativity:

    “There is only one demand to be made of the definition of simultaneity, namely, that in every real case it must supply us with an empirical decision as to whether or not the conception that has to be defined is fulfilled. That my definition satisfies this demand is indisputable. That light requires the same time to traverse [paths of equal distance, regardless of motion] is in reality neither a supposition nor a hypothesis about the physical nature of light, but a stipulation [postulate] which I can make of my own free will in order to arrive at a definition of simultaneity.”

    I'll add some more color to these issues in my response to Amos below.

    In terms of causal order being preserved even with the relativity of simultaneity, as you suggest, this is not the generally held view by today's relativists. See Carroll's new book The Big Picture for an argument that causality itself needs to be thrown out, in part due to SR and GR, which is in my view a rather perverse result and yet another reason to carefully consider alternatives to SR and GR.

    ReplyDelete
  156. @Amos, responding to your 2:49 Sept. 9 post.

    Thanks for highlighting the later Lorentz's change in views. However, both Lorentz and Poincare argued to their death that a background-dependent (ether-based) relativity was the superior interpretation, as Galison makes clear in this passage:

    By 1902, Lorentz had long been experimenting with fictional mathematical variations in the way the time variable t would be defined for an object moving in the ether. With Poincaré’s and other physicists’ further articulation, the notion of a fixed ether had gained ground. As we know, Poincaré had (first ignoring the ether altogether and then for systems explicitly moving through it), interpreted simultaneity through light-signal-coordinated clocks. Though his use of the ether changed, Poincaré never wavered from his conviction that the ether was enormously valuable as a tool for thinking, a condition for the application of productive intuition. He never equated “apparent time” (time measured in a moving frame) with “true time” (time measured at rest in the ether).

    Galison, Peter. Einstein's Clocks, Poincare's Maps: Empires of Time: Empires of Time (pp. 241-242). W. W. Norton & Company. Kindle Edition.

    As for Lorentz's acknowledgement to Einstein of the difficulty in establishing which frame was "at rest" and thus equivalent to the ether frame, my earlier points about using the CMB as a cosmic preferred frame resolve that issue readily. We simply use the CMB (as is already done in many ways in today's astronomy) as a universally-available preferred frame.

    I am not espousing a 19th C. mechanical notion of the ether, and nor was Lorentz. I am suggesting that we do in fact have good candidates for a cosmic preferred frame, as just mentioned, that function as the ether did for Lorentz. More generally, we do in fact have ubiquitous fields that are the more accurate analog of Lorentz's ether. So looking at the CMB as the anisotropic manifestation of the various fields of nature as a cosmic preferred frame is entirely congruent with modern field-based physics as well as Lorentz's views on relativity. It is also arguably most congruent with Einstein's own direction during the middle and end phases of his career, as the following summary from Max Jammer demonstrates:

    Max Jammer, in a Foreword to Kostro’s book (Einstein and the Ether), summarizes Einstein’s interesting and complicated history vis a vis the ether:

    "Einstein … acknowledged only three kinds of ether. But what Einstein called “ether” is no longer a rarified material medium that permeates all space, but rather the much more abstract geometrodynamic constituent of spacetime which determines the inertio-gravitational behavior of matter. … [B]efore Einstein, space and time had played the role of merely a passive background in which events take place, but Einstein’s theories transformed them into active participants in the dynamics of the cosmos. We should also recall that Einstein created not only the special and general theory of relativity: during the last decades of his life he was preoccupied, albeit without success, with establishing a third theory, a unified theory that unites the gravitational with the electromagnetic forces. With each of these three theories he associated an “ether” in the above-mentioned sense."

    It seems reasonable to me that if Einstein had lived to see the CMB data and other similar data re universal-scale baryonic matter structures that he would have agreed that these data represent an obvious way to distinguish a preferred cosmic frame.

    More responses to come; that's all I have time for tonight.

    ReplyDelete
  157. @Tam Hunt

    Thanks for clarifying. You did teach me something new regarding the one-way speed of light and how it leaves the door open to the Neo-Lorentzian interpretation.

    In terms of causal order being preserved even with the relativity of simultaneity, as you suggest, this is not the generally held view by today's relativists. See Carroll's new book The Big Picture for an argument that causality itself needs to be thrown out, in part due to SR and GR, which is in my view a rather perverse result and yet another reason to carefully consider alternatives to SR and GR.
    The point that Sean Carroll (and others) are really making is that the direction of causality that we experience in our everyday lives is dictated by the flow of entropy; the past is different than the future because entropy was lower in the past and increases with a probability of nearly one. The laws of physics remain the same if you reverse the arrow of time. (I recall reading there may be obscure exceptions in certain rare types of particle decay, but not sure.)

    Bits of clay and spilt coffee on the floor can spontaneously jump onto your table and form a cup of coffee, if the molecules all align their movements (from warmth) in exactly the right way.

    The probability is ridiculously small, but the laws of physics do not forbid it. Hence, ´cause and effect´, as we use the term in our everyday lives, arises from a low entropy in the past and statistical likelyhood. And that is what Sean Carroll tells us in his book.

    In particular, SR and GR do not have a preference for one time direction or another. @Tam Hunt

    Thanks for clarifying. You did teach me something new regarding the one-way speed of light and how it leaves the door open to the Neo-Lorentzian interpretation.

    In terms of causal order being preserved even with the relativity of simultaneity, as you suggest, this is not the generally held view by today's relativists. See Carroll's new book The Big Picture for an argument that causality itself needs to be thrown out, in part due to SR and GR, which is in my view a rather perverse result and yet another reason to carefully consider alternatives to SR and GR.
    The point that Sean Carroll (and others) are really making is that the direction of causality that we experience in our everyday lives is dictated by the flow of entropy; the past is different than the future because entropy was lower in the past and increases with a probability of nearly one. The laws of physics remain the same if you reverse the arrow of time. (I recall reading there may be obscure exceptions in certain rare types of particle decay, but not sure.)

    Bits of clay and spilt coffee on the floor can spontaneously jump onto your table and form a cup of coffee, if the molecules all align their movements (from warmth) in exactly the right way.

    The probability is ridiculously small, but the laws of physics do not forbid it. Hence, ´cause and effect´, as we use the term in our everyday lives, arises from a low entropy in the past and statistical likelyhood. And that is what Sean Carroll tells us in his book.

    In particular, SR and GR do not have a preference for one time direction or another. But in that respect they are no different from classical mechanics. As you will find in any textbook on physics, there still remains ´causality´, defined by lightcones, which tells us how information, mass or energy can or cannot propagate (either forwards or backwards). Sean Carroll acknowledges this and even complains that people tend to misunderstand what physicist mean by the word.

    How do you propose that the neo-Lorentzian interpretation could make a difference here? It is a different description of special relativity, making the same predictions and having the same time symmetry..

    ReplyDelete
  158. Lorraine Ford;

    Sorry for the delayed response; Life Happens. Anyway ...

    You seem to agree with Lee Smolin’s book Time Reborn”.

    I have never read Smolin’s book.

    Time is not necessary at a fundamental level: what is necessary is the capacity to change, or create new, (what we would represent as) numbers, mathematical relationships and algorithms.

    Time is absolutely necessary because change cannot occur without there being the period of time before the change and the period of time after the change. Time without change is ‘stasis’, change without time is absurd.

    This necessary capacity to change, or create the new, does not require time for permission to exist or operate.

    I don’t know anything about permission, but the capacity to change cannot exist without the prior existence of time. Time is necessary for change to be possible.

    Time is superfluous at a fundamental level because time does not cause or create change.”

    Time might not be sufficient for change, but it is necessary. Sufficiency and Necessity are not the same.

    In summary, Lee Smolin seems to be saying that: ...

    I am not Lee Smolin, and I have no stake in anything Smolin said or wrote.

    Elsewhere you wrote that “... physics represents reality as equations and numbers, so you’ve only got 2 choices for ‘change’: change of number for a variable, and change of law of nature equation.

    You are making a fundamental error.

    How physics “represents reality” does not control how reality operates. As the old-but-true saying goes: the map is not the territory. The representations of reality made by physicists only reflect what they know or believe about reality; reality does not care about those representations. In reality, all changes are embedded in time, there is always a period of time before the change and a period of time after the change. How representations change DOES NOT MATTER. Only how reality changes matters.

    Why are the numbers changing? Because reality is constantly changing.

    sean s.

    ReplyDelete
  159. bud rap;

    [ss] Science does rest on empiricism, but not only on empiricism.

    I did not say it only rested on empiricism.


    Good. We agree.

    What I said, very clearly with regard to the Higgs was: Direct empirical evidence is far stronger than inferential evidence. So, my position on the Higgs is that there is no compelling evidence for its existence. No compelling evidence is not the same as no evidence.

    What you seem to be saying is that only direct empirical evidence is compelling evidence; that’s not a compelling argument. Is logic not compelling?

    My argument is as follows:

    1) There is no empirical evidence for the existence of a substantival ‘time’ that causes physical events.

    2) The evidence only supports the existence of time as a relational concept.

    3) Therefore when you or TH appear to assert that time is "real" in the substantival sense you are reifying a relational concept.


    There is plenty of experiential evidence that time exists.
    There is NO evidence that time is only a relational concept.
    There is NO logical way to describe actual change in the real world without invoking time.

    An important point that I think needs to be addressed: it is not established what role spacetime plays in the “causing” physical events. But it is clear that time is necessary for change to be possible whether it is the cause or not. Without time, change cannot happen; there’d be no before or after. Without time, cause cannot precede effect. The very idea of causality requires time to exist.

    It would be good to work out the causal role of spacetime, but until there’s consensus that time exists fundamentally, the matter seems premature.

    I did not bring up the temperature vs heat distinction, you did.

    Correct, but you should have. You said humans experience temperature, which is an error. We experience heat and heat-changes; not temperature.

    Lastly, it would be helpful if you could say something indicating that you at least comprehend the substantival vs relational distinction that is at the core of our disagreement. To this point, your responses have been unclear on the matter.

    My response have been quite clear if not explicit. Substantivalism asserts that spacetime is fundamental. I have been EXCRUCIATINGLY CLEAR that I claim time is fundamental. You disagree. Your position is clear to me; I cannot fathom how you could be confused about mine.

    sean s.

    ReplyDelete
  160. JimV;

    At velocity c, all changes happen instantaneously, so there is change without time.

    Only photons can travel at c, and they are constantly changing location or direction. Their wave function is also never unchanging unless their frequency is zero.

    Other objects cannot travel at c, their time never fully stops, and within such objects their changes continue. No matter how close to c a living human gets, they will still breathe and have a pulse. To them it will seem normal.

    Suppose that there is a completely isolated system containing some radioactive atoms. Nothing happens, until suddenly one of the atoms emits a beta particle. Then nothing, until a second atom emits a beta particle. Meanwhile, in the rest of the universe, things have been happening, unknown in the isolated system. Has no time elapsed in the isolated system except when the radioactive decays occurred?

    When “nothing is happening”, time is still passing. The elapsed time of the isolated system could be determined with some precision by comparing the radioactive breakdown to the known half-life of those atoms.

    So what is time?

    I wish I had a good answer, but like many fundamental things, descriptions from our derivative experiences are often hard to apply. We make models, but the model is not the thing modeled.

    sean s.

    ReplyDelete
  161. Thank you JimV :-)

    Glad you mentioned Gnome Ann. The “facts” about Gnome Ann seem very (?) relevant to this discussion:

    Time and Tide wait for Gnome Ann” (proverb); “Time ripens all things; Gnome Ann is born wise” (The novel Don Quixote by Miguel De Cervantes).

    ReplyDelete
  162. Tam Hunt,

    Why are you reluctant to quantify and categorise “change”? Only by attempting to quantify and categorise can you determine whether or not elements of change remain which cannot be quantified and categorised. E.g. a quantum event itself can’t be quantified or categorised: only the outcome of a quantum event can be quantified and categorised e.g. as a number assigned to particular category of variable.

    But what is a quantum event? QBist physicists Christopher Fuchs and Blake Stacey say that reality is participatory:

    “In some way yet to be fully fleshed out, each quantum system seems to be a seat of active creativity and possibility, whose outward effect is as an “agent of change” for the parts of the world that come into contact with it.” [1]

    “noting how the Big Bang itself is a moment of creation with some resemblance to every individual quantum measurement, one starts to wonder whether even it “might be on the inside.” Certainly QBism has creation going on all the time and everywhere; quantum measurement is just about an agent hitching a ride and partaking in that ubiquitous process.” [1]

    1. QBism: Quantum Theory as a Hero's Handbook, Christopher A. Fuchs and Blake C. Stacey, https://arxiv.org/abs/1612.07308v2

    ReplyDelete
  163. Lorentz and Poincare argued to their death that a background-dependent (ether-based) relativity was the superior interpretation…

    It goes without saying that Lorentz espoused a Lorentzian interpretation. Everyone knows this, just as everyone knows that Poincare’s position was more nuanced (or ambiguous), since he famously said things like “Whether the ether exists or not matters little—let us leave that to the metaphysicians… some day, no doubt, the ether will be thrown aside as useless”. But the point is that the Lorentzian and Einsteinian interpretations of special relativity are empirically equivalent, so they both contradict your beliefs. Regardless of interpretation, inertial coordinate systems (operationally defined) are related by Lorentz transformations, from which it follows that (among other things) two ideal clocks show different elapsed times for different paths between the same events. This is an empirically verified fact, and your entire system of belief rests on the denial of this abundantly demonstrated fact.

    It seems reasonable to me that if Einstein had lived to see the CMB data… he would have agreed that these data represent an obvious way to distinguish a preferred cosmic frame.

    You completely misunderstand. It was Einstein who, in 1917, originated general relativistic cosmology by discussing a closed spherical universe with an obvious distinguished global time (at each location), long before any expansion or CMBR was detected, but neither this nor any subsequent developments undermine the validity of local Lorentz invariance. Please note that the perfect symmetry between local inertial coordinate systems is not a drawback or something to be overcome if only we were clever enough to think of some globally distinguished frame, it is a miraculously fortuitous circumstance that gives us insight into the locally Lorentz invariant laws of physics, and it has very significant physical consequences, such as the inertia of energy (E=mc^2). One minor example is the fact that, in negligible gravity, two ideal clocks taking different paths between two events show different elapsed times precisely in agreement with the equations of special relativity.

    In summary, I think your beliefs are self-contradictory (since you claim that Lorentzian and Einsteinian relativity are and are not empirically equivalent) and empirically falsified (since you deny different elapsed times for ideal clocks following different paths).

    ReplyDelete
  164. Lorraine Ford (to Tam Hunt):

    Why are you reluctant to quantify and categorise ‘change’?

    Perhaps because your request is too vague. “Change” is a very general term. There are many kinds of change even if we restrict our view to the quantum level. There are changes in location, velocity, frequency, etc.; when particles break down then there are additional varieties of end-products and velocities.

    Perhaps you could provide some examples of the kind of quantifying and categorizing you’re looking for; that might help demonstrate why what you ask matters.

    ... a quantum event itself can’t be quantified or categorised: only the outcome of a quantum event can be quantified and categorised

    That doesn’t seem right. The combination of the input to the event plus the outcome of the event should enable “categorizing” any event.

    sean s.

    ReplyDelete
  165. sean s,

    What you seem to be saying is that only direct empirical evidence is compelling evidence; that’s not a compelling argument. Is logic not compelling?

    Logical arguments are only as compelling as the premises they are based on. People have had logical arguments about the number of angels that can dance on the head of a pin. What I said is what I meant: Empirical evidence is stronger and therefore more compelling than model-based inferential evidence.

    [br] 1) There is no empirical evidence for the existence of a substantival ‘time’ that causes physical events.

    [ss]There is plenty of experiential evidence that time exists.

    Yes, there is, but that does not refute point #1 above.

    [br] 2) The evidence only supports the existence of time as a relational concept.

    [ss] There is NO evidence that time is only a relational concept.

    Yes there is. Negative empirical evidence is evidence, and the absence of empirical evidence supporting substantival time is strong evidence that time is only a relational concept.


    [br] 3) Therefore when you...assert that time is "real" in the substantival sense you are reifying a relational concept.

    There is NO logical way to describe actual change in the real world without invoking time.

    The room became brighter. That statement is complete in itself as a description of a physical change. It can be qualified by invoking the time and duration of the event in the local frame but that elaboration is not necessary. On the other hand, merely stating that something unspecified changed in a room over the course of a specific hour is fundamentally meaningless. You have to invoke a physical change to give the relational concept of time meaning.

    ReplyDelete
  166. Amos,

    you said, that 'Regardless of interpretation, inertial coordinate systems (operationally defined) are related by Lorentz transformations, from which it follows that (among other things) two ideal clocks show different elapsed times for different paths between the same events'.

    That's new for me. I thought that not the different paths, but the different velocities or different accelerations or different gravitational potentials make the difference.

    What do you think about the allegation against Hafele & Keating that their huge citated result is a wrong interpretation of their data?

    http://www.cartesio-episteme.net/h%26kpaper.htm

    "7. Conclusions

    The H & K tests [Hafele & Keating experiment] prove nothing. The accuracy of the clocks would need to be two orders of magnitude better to give confidence in the results. The actual test results, which were not published, were changed by H & K give the impression that they confirm the theory. Only one clock (447) had a failry steady performance over the whole test period; taking its results gives no difference for the Eastward and the Westward tests."

    ReplyDelete
  167. Hi Bud Rap,

    I am largely in agreement with you, but to advance the conversation perhaps you could give an example of something you believe fits the bill of substantival existence that is supported by empirical evidence. In other words, you say time is merely a relational concept and I agree, but I think all things are merely relational and lack substantial existence. Is there any physical concept that you think is substantival rather than merely relational? And if so, how does empirical evidence support this while not supporting time as a substantival concept?

    Thanks!

    ReplyDelete
  168. bud rap;

    Thankyou for your response, it was very helpful.

    Negative empirical evidence is evidence, and the absence of empirical evidence supporting substantival time is strong evidence that time is only a relational concept.

    Negative empirical evidence is significant only if some empirical evidence is expected. I cannot recall you or anyone else on this thread mentioning some empirical evidence that fundamental time should exhibit. In the absence of such expectations, the absence of empirical evidence is not significant.

    Can you specify some empirical evidence we should see if time were fundamental?

    The room became brighter. That statement is complete in itself as a description of a physical change. ...

    The “relational time” aspect of your statement is present in the word became; the room was dimmer for a period of time and then became brighter.

    What relationship does “relational time” refer to then? A relationship in time.

    Time is a relationship in time?

    “Relational time” could refer to time only if time exists apart from a relationship. This implies that time exists of itself and not as a derivative of a relationship. In short: time is fundamental.

    You continued, “ ... It can be qualified by invoking the time and duration of the event in the local frame but that elaboration is not necessary.

    Correct. The reference to time is already present in the word became. Further references to time are redundant (unless the timing or duration are independently significant).

    You have to invoke a physical change to give the relational concept of time meaning.

    The “relational concept of time” only indicates a relationship in time, which invokes fundamental time. The “relational” aspect of time refers to its fundamental nature.

    sean s.

    ReplyDelete
  169. sean s,

    The use of symbolic representation has become second nature for human beings. People rarely notice that they are using symbolic representations all the time. The symbolic representations are words and sentences. Your changing words and sentences have led me to conclude that you are upset about this issue. But you claim that: “How representations change DOES NOT MATTER”. But in fact, symbolic representations (words) are often the only information we have that might allow us to draw conclusions about reality.

    Physics symbolically represents aspects of fundamental-level reality as equations (for law of nature relationships between variables), and numbers (for variables like energy, momentum, relative spatial position). We don’t directly know fundamental-level reality: physics’ symbolic representations of fundamental-level reality (equations, variables and numbers) are often the only information we have that might allow us to draw conclusions about fundamental-level reality.

    Also in order to draw conclusions about reality, one needs to break-up situations and events into their component parts (e.g. something representable as categories/variables and quantities/numbers). I would like Lee Smolin (who says “We live in a universe that is always changing”) and Carlo Rovelli (who describes change as “happenings”) to break “change” into its component parts, as they see them. Otherwise one might conclude that “change” has no component parts, making “change” sound very mystical.

    ReplyDelete
  170. @weristdas

    What do you think about the allegation against Hafele & Keating that their huge citated result is a wrong interpretation of their data?

    The article you link to has not been published in a journal. Now Sabine Hossenfelder is constantly warning us on this blog about confirmation bias among scientists, but when a paper rejects the scientific consensus and has never been published, this nonetheless a major red flag that it might not be of top quality. I couldn't even find any scientific articles that respond to the article in your link, which is even more suspicious. No one bothered to reply.

    When you google the author, you find very few published papers or other recognitions at all, but you do find this on : https://wiki.naturalphilosophy.org/index.php?title=Umberto_Bartocci

    He slowly realized that there was a strong ideological connection between Darwinism, the formalistic foundation of Mathematics which lead to a Mathematics without Intuition, and the "irrationalistic" development of 20th Century Physics, which started with Relativity [..]
    Rather deluded for the negligible results attained in front of a great effort, he did retire from teaching and research in 2006.


    It seems he was trying to disprove Relativity for ideological reasons and failed. Could he be the lone unrecognised genius? I wouldn't bet on it. As dangerous as groupthink in science might be, people who go against the grain and are ignored are often just wrong. They laughed at Einstein, but they also laughed at Bozo the clown.

    ReplyDelete
  171. sean s,

    Negative empirical evidence is significant only if some empirical evidence is expected. I cannot recall you or anyone else on this thread mentioning some empirical evidence that fundamental time should exhibit. In the absence of such expectations, the absence of empirical evidence is not significant.

    That may possibly be an acceptable metaphysical argument but it is definitely absurd in a scientific context. It is not my responsibility to prove to you that angels don't exist just because you happen to believe in them, and the same goes for time. It is not my responsibility to dream up some possibly observable characteristic for your imaginary friend which we should all go searching for in hopes of demonstrating that your metaphysical assertions of time's existence have a physical meaning.

    The responsibility to define time in such a way as to admit the possibility of demonstrating its existence empirically rests entirely with those who, like you, believe it has substantival properties. Something that has substantival properties should be empirically demonstrable; if it is not then the substantival entity being proposed has no place in science, except perhaps, as an unproven hypothesis.

    Your position appears to be that time is substantival but since you can propose no test of this assertion, it falls on science to disprove your belief. That is logical only in the context of your belief system in which the substantival nature of time is a foregone conclusion. It is illogical in the context of science.

    A couple of posts back, you made the following assertion:

    You said humans experience temperature, which is an error. We experience heat and heat-changes; not temperature.

    This was, you seem to have thought, a refutation of my argument that time was analogous to temperature. But the insubstantial nature of temperature, as only a relational concept, that you correctly point out is precisely analogous to the argument against substantival time. Substitute time for temperature in the above quote and you have arrived at the point where both are only relational concepts.

    You, of course, would like to assert that people experience time but not temperature. Unfortunately, every such time experience you might wish to point to consists of an experience of a change of position or state of some observable(s). Time is just as insubstantial a concept as temperature and you have no substantive evidence to support any other view of the matter. All you have are your feelings and beliefs and all your arguments are based on the assumption that your beliefs are correct and so all your arguments are circular in nature:

    What relationship does “relational time” refer to then? A relationship in time.

    Time is a relationship in time?

    “Relational time” could refer to time only if time exists apart from a relationship. This implies that time exists of itself and not as a derivative of a relationship. In short: time is fundamental.


    Whatever that is, it is not a logical, scientific argument.

    ReplyDelete
  172. Lorraine Ford;

    The use of symbolic representation has become second nature for human beings. People rarely notice that they are using symbolic representations all the time.

    It is an error to forget that symbolic representation is just symbolic representation. The symbol is not the thing symbolized; the representation is not the thing represented.

    ... symbolic representations (words) are often the only information we have that might allow us to draw conclusions about reality.

    This is rare. If the “symbol” is actually all we have then it is not a “symbol” because we don’t know what it represents. Such a thing is a placeholder.

    Physics symbolically represents aspects of fundamental-level reality as equations (for law of nature relationships between variables), and numbers (for variables like energy, momentum, relative spatial position).

    We represent fundamental reality with models and theories as well as equations; not all aspects of fundamental reality are quantifiable; so they cannot be well represented with numbers or equations. Atomic theory or quantum entanglement are not equations.

    We don’t directly know fundamental-level reality: physics’ symbolic representations of fundamental-level reality (equations, variables and numbers) are often the only information we have that might allow us to draw conclusions about fundamental-level reality.

    For the quantifiable aspects of fundamental reality, that’s all fine and good, but it remains an error to confuse the representation or the symbol or the measure with the thing represented, symbolized, or measured. It is a blunder to treat the equations, variables, and numbers as all there is or all that matters.

    I would like Lee Smolin ... and Carlo Rovelli ... to break ‘change’ into its component parts, as they see them. Otherwise one might conclude that ‘change’ has no component parts, making ‘change’ sound very mystical.

    Well, good luck with that. But until Smolin or Rovelli comply with your request; life goes on. Change is not mystical, but it is not merely numbers changing either. There are many kinds of changes; many don’t fit into a tidy numerical category.

    sean s.

    ReplyDelete
  173. weristdas said I thought that not the different paths, but the different velocities or different accelerations or different gravitational potentials make the difference.

    The elapsed proper time along any path is just the integral of sqrt[g_mn dx^m dx^n] along the path (summation implied). This accounts for all effects of motion and gravity (if any). Naturally two objects following different paths through space and time between two given events entails that they have (at least some of the time) different states of motion, and may encounter different gravitational potential.

    What do you think about the allegation against Hafele & Keating that their huge citated result is a wrong interpretation of their data?

    It's true that the drift rates of the four individual clocks used by H&K were significantly greater than the relativistic effect they were trying to detect, so it was obviously necessary to correct for that. Averaging over just four samples with tolerances much greater than the signal is admittedly challenging. Also the clocks exhibited some abrupt changes in drift rate, as can be seen in the published ground data. Originally H&K hoped no shifts would occur during the flights, but they actually detected several by cross-comparisons between the four clocks, so they corrected for those.

    Given the large drift rates, numerous rate shifts, and dicey adjustments, their results certainly wouldn’t qualify as a “five sigma detection", but I think they could legitimately claim to have found at least qualitative support for the predicted time dilation. Aside from historical interest, it’s a moot point, because more precise and stable clocks are now available, and similar experiments have been conducted (e.g., in 2010), not to mention GPS satellite clocks, etc., all confirming the relativistic predictions.

    ReplyDelete
  174. As I said previously (so I owe the site the traditional fine for repeating myself), time as we know it is simply the counting of happenings: ticks of a clock (seconds), rotations of the Earth (days), orbits of the Earth around the Sun (years), and so on. Typically we pick some such happening as the standard and measure other happenings against it. Galileo measured the pendulum period of chandeliers, while in church, against his own heart beats. The fundamentals of this procedure appear to be counting, counts (integers), and happenings. It turns out, according to SR and GR, that the counts of some happenings relative to others will vary for different counters (observers) in different circumstances. So the counting itself is not fundamental since it depends on other things, such as velocity and gravity. Depending, of course, on how one defines fundamental.

    That seems to me to be all we can say about time, from present knowledge. (I may be leaving out some speculations based on QM.) Some like to assert deeper knowledge based on philosophical intuition, but the track record of such intuitions by some of the most brilliant people of history has not been good. (Still, even a blind squirrel, et cetera.)

    Dr. Carroll's book, "The Arrow of Time" has been mentioned. I have not read that book, but have read his blog postings on the topic. However, I have not garnered from those readings how it is that entropy can be a driver of time rather than a passenger. For me, subjectively, the direction of time is measured by my memory: I can remember (or look up recordings of) past events; I can't do this for future events. If a broken egg reassembled itself in front of me, reversing entropy briefly, I expect I would be able to remember that event as well as I remember the egg breaking. As I have also said before (but I am not paying a double fine for: one repetitive comment, one fine), I see physical laws with time derivatives as implicitly containing a direction of time, since the forward form of the derivative will have to be used to calculate predictions of future events from those equations.

    ReplyDelete
  175. @JimV

    However, I have not garnered from those readings how it is that entropy can be a driver of time rather than a passenger. For me, subjectively, the direction of time is measured by my memory: I can remember (or look up recordings of) past events; I can't do this for future events.

    Writing or reading memory, like basically any mechanical operations, are processes that decrease entropy locally, at the cost of a far greater increase in the total/global entropy. This is true regardless of the type of memory we are talking about: a computerdisk, an abacus, your brain, writing books, taping film, you name it.

    This is the reason why you remember the past and not the future. Entropy is almost the only law* of nature that introduces an assymetry between the future and the past. This is all explained very well on the wiki page for entropy, in case you are interested.

    *: technically a statistical likelyhood rather than a law; if a freak incident would happen like an egg that spontaneously assembles itself, you would still remember the past, provided that entropy increases in the usual way inside of your brain in the meantime..

    ReplyDelete
  176. bud rap;

    I wrote previously that “Negative empirical evidence is significant only if some empirical evidence is expected. ...

    You replied, “That may possibly be an acceptable metaphysical argument but it is definitely absurd in a scientific context.

    In a scientific context, my statement is completely correct. Anyone familiar with the impact of the findings and NON-findings of the LHC is aware of the correctness of my statement. This is current events, bud rap.

    It is not my responsibility to prove to you that [red herrings omitted] don’t exist ... , and the same goes for time.

    That is true until you claim the lack of evidence is significant. Significance is your claim then, and it becomes something you need to establish. Which you cannot.

    It is not my responsibility to dream up some possibly observable characteristic for your imaginary friend ...

    It is your responsibility if and when you claim that the absence of some observable characteristics is meaningful. If you cannot “dream up” some observable characteristics, then logic says that the lack of evidence of these unknown characteristics is without meaning.

    The responsibility to define time in such a way as to admit the possibility of demonstrating its existence empirically rests entirely with those who, like you, believe it has substantival properties.

    I have never claimed that time has “substantival” properties. I have said it’s fundamental; that’s not the same thing. Other than that, I’m not sure at all how to perform any experiment on time; and apparently you have no idea either. But since WE AGREE that science is not limited to direct observables or empirical evidence, then we can go on with less rigorous but still valid information. Logic says change without time is impossible; deriving time from something else is an incoherent idea.

    Your position appears to be that time is substantival ...

    You appear to erroneously think I am a substantivialist (Gad; what an ugly word!) but that is just a bad assumption on your part. I have never said anything to lead you to that error; so I can ignore this.

    ... but since you can propose no test of this assertion, it falls on science to disprove your belief.

    That is never true and I have not claimed it was. You claim the lack of empirical evidence is significant; that is wrong because NEITHER OF US can specify any experiment or empirical result that we should observe. In this kind of case, null results has no real significance.

    That is logical only in the context of your belief system in which the substantival nature of time is a foregone conclusion.

    Not being a substantivalist (ick!), this is irrelevant.

    Time is just as insubstantial a concept as temperature and you have no substantive evidence to support any other view of the matter.

    I do. “Substantive” means having a firm basis in reality and therefore important, meaningful, or meriting consideration.

    It is a fact of reality that we experience time; but we do not experience temperature. That fact is important, meaningful and worth consideration. That is substantive evidence. It’s not what you’re looking for, but it is substantive evidence.

    You seem to believe that time is derivative of change events, but change events cannot happen unless time already exists.

    When you can explain how an actual change can occur WITHOUT TIME, then you’ll have me, but you can’t. Time without change is stasis; change without time is absurd.

    sean s.

    ReplyDelete
  177. JimV;

    ... time as we know it is simply the counting of happenings: ticks of a clock (seconds), rotations of the Earth (days), orbits of the Earth around the Sun (years), and so on. Typically we pick some such happening as the standard and measure other happenings against it.

    That is a fine description of how we measure time, but time is more than that. Such happenings cannot “happen” unless time exists independently of those happenings. That is not “philosophical intuition” on my part, but plain ol’ logic.

    sean s.

    ReplyDelete
  178. sean s,

    I have never ever suggested that the symbol is the thing symbolized or that the representation is the thing represented or that the model is the thing modelled. I have always made the distinction. The only people who somehow suggest that the representation/model is the reality are the mathematical universe proponents.

    Re “placeholders”: The only “placeholders” are things like random strings of binary digits, and words in foreign languages, because we don’t know what the symbols represent. In the context of my full paragraph, about human beings’ everyday use of word symbols that they already know the meaning of, I never implied that “the “symbol” is actually all we have. We already know the meaning of the individual word symbols (i.e. we know what the symbols represent) and we know how sentences are constructed; but we can draw further conclusions (i.e. acquire further information) by considering the particular words selected, and the order of the words, and the history of word usage by a particular person. In the context of our existing knowledge, misknowledge and false beliefs about reality, we can draw further conclusions about reality from sentences we hear or read. To get back to the issue of time and change: we can indeed draw conclusions about reality from symbolic representations of reality. But due to potential misknowledge and false beliefs, these conclusions would need confirmation, e.g. by experiment. But experimental confirmation of hypotheses about fundamental-level reality can seemingly only be done when this reality is represented by equations, variables and numbers: any hypothesis about aspects of fundamental-level reality that are not representable like this cannot be experimentally confirmed, they can seemingly only be “confirmed” by e.g. philosophical/logical argument.

    Re “We represent fundamental reality with models and theories as well as equations; not all aspects of fundamental reality are quantifiable; so they cannot be well represented with numbers or equations. Atomic theory or quantum entanglement are not equations.”:

    That is exactly what I have been saying. Due to physics’ attempt to categorise, quantify, and model, it is possible to identify aspects of reality that are not able to be categorised, quantified, and modelled. This is why the word “change”, as applied to fundamental-level reality, needs to be further examined, because there are indeed aspects that can be categorised, quantified, and modelled. But as I said above (sorry about quoting myself!): “experimental confirmation of hypotheses about fundamental-level reality can seemingly only be done when this reality is represented by equations, variables and numbers: any hypothesis about aspects of fundamental-level reality that are not representable like this cannot be experimentally confirmed, they can seemingly only be “confirmed” by e.g. philosophical/logical argument.” I agree that there seem to be fundamental aspects of reality that cannot be entirely categorised, quantified, and modelled (like physics does).

    ReplyDelete
  179. JimV,
    Would you agree that a time unit derives from knowledge (possessed by some frame-of-reference aspect of the universe) of definite abrupt change events or happenings: a time unit can seemingly not be evaluated or discerned if number change is “smooth”? Would you agree that in either case, algorithmic information seems to be involved?

    ReplyDelete
  180. In reply to Lorraine Ford,

    I don't think my opinion on such matters is important, but like Zeno, Democritus, and George Ellis, to me the idea that any part of reality is smooth and continuous in an infinitesimal sense does not seem right. Although, the notion of continuity is very useful (in calculus) as an approximation to reality. We apply it to water flow and all sorts of things, while knowing that water is actually not continuous but composed of discrete molecules.

    "Would you agree that in either case, algorithmic information seems to be involved?"

    In an attempt to answer this I googled "algorithmic information", but every hit I have checked so far was a hit on "algorithmic information theory", and none of them start by defining "algorithmic information". I can only assume that "ai" means information generated by an algorithm. Based on that assumption, and the further assumptions that information is only knowingly possessed by sentient beings and that sentient beings gain their sentience by the use of algorithms (trial-and-error being the most important one, in my opinion), then I think my answer is probably yes. However, as this reply suggests, I think I exhausted my useful thoughts on time with my previous comment. Thanks for trying, though.

    ReplyDelete
  181. Lorraine Ford;

    I’m glad we agree that symbols are not the same as the things symbolized. Do we agree that, to understand reality, we need to consider reality? We use symbols as tools, but we must remember that symbols are only tools.

    ... we can draw further conclusions (i.e. acquire further information) by considering the particular words selected, and the order of the words, and the history of word usage by a particular person.

    The information we acquire this way is only about language, mental constructs, and symbols themselves. These considerations also give us psychological information about the particular person. These considerations give us no information about the reality symbolized by the words in question.

    In the context of our existing knowledge, misknowledge and false beliefs about reality, we can draw further conclusions about reality from sentences we hear or read.

    Agreed. Language is how most information is communicated.

    ... experimental confirmation of hypotheses about fundamental-level reality can seemingly only be done when this reality is represented by equations, variables and numbers: any hypothesis about aspects of fundamental-level reality that are not representable like this cannot be experimentally confirmed, they can seemingly only be “confirmed” by e.g. philosophical/logical argument.

    Hypotheses about fundamental reality can also be represented qualitatively; descriptions and concepts, processes, etc. Those representations are as confirmable as equations, variables and numbers.

    ... the word ‘change’, as applied to fundamental-level reality, needs to be further examined, because there are indeed aspects that can be categorised, quantified, and modelled.

    “Change”, as it applies to fundamental things, refers to alterations in attributes of those things (location, velocity, mass, etc.) or composition/decomposition into other things characterized by other attributes (mass, charge, spin, etc.)

    ... or at least that’s my SWAG.

    sean s.

    ReplyDelete
  182. @Amos, responding to your 7:51 Sept. 10 post. This is a multi-part response.

    Part 1:

    Here's a passage from a paper of mine that addresses your points about the 2-way and 1-way speed of light (both of which can only be measured based on certain conventions/assumptions), and I highly recommend Jammer's book Concepts of Simultaneity (and its last chapter in particular) if you haven't already read it:

    Knowledgeable physicists will acknowledge that LR is a viable approach
    given that both use the Lorentz transformations and are thus generally
    not considered to be empirically distinguishable. This dilemma about which theory is better is
    sometimes described as an “aesthetic” debate, not because the
    difference in interpretations is trivial but because without empirical
    data to make a choice we must look to other considerations like
    aesthetics and parsimony. As discussed above, however, there is indeed a
    substantial amount of data that can empirically distinguish these two
    interpretations – but this state of affairs is not widely acknowledged
    yet.

    Physicists who argue in favor of Einstein on this issue (the large
    majority still would today) rest their arguments often on the notion
    that Special Relativity is more parsimonious because it can explain the
    same phenomena with fewer components; namely the absence of an ether.
    A related parsimony argument centers on our assumptions about the speed
    of light. Einstein assumed that the speed of light was constant no
    matter the speed of the observer, as an explanation of the
    Michelson-Morley data. We can assume otherwise, however, particularly
    considering that no other speeds in the universe behave this way. That
    is, all other speeds do in fact change based on the speed of the
    observer.

    In the context of measuring the speed of light, Einstein’s assumption
    leads to an ε value of 0.5 in the following light speed measurement
    equation first formulated by Reichenbach 1924:

    t2 = t1 + ε(t3 −t1)                                                                                                                            (1)

    ε of 0.5 is a prima facie parsimonious assumption, despite it
    being highly counterintuitive, because it allows for an easier
    measurement of the speed of light by using mirrors to reflect light back
    to the source (see Jammer 2006 for more, particularly the last chapter).
    Operationally, this allows an experimenter to shoot a ray of light, at
    t1, at a mirror, reached at t2, and measure the time elapsed for the light’s return at t3.

    ReplyDelete
  183. @Amos, part 2 (continued, this language is me quoting one of my papers on these topics):

    ...By dividing the elapsed time by two, we can derive the average speed of
    light over this distance. This technique only works for determining the
    one-way speed of light if one assumes that the speed out is the same as
    the speed back, which equates to an ε of 0.5. But, again, any other
    speeding object would not have the same speed in both directions because
    of other forces impacting the object differently based on its direction,
    such as air speed. And if the observer was herself moving during this
    experiment, any other moving object would indeed show a different speed
    out and back.

    Einstein’s assumed isotropic speed of light made sense at the time
    operationally, but we now know that many problems arise from this
    assumption, including how to reconcile the obvious flow of time in
    quantum mechanics, in the cosmic frame, and in everyday experience with
    this assumption. Reichenbach stated: “If the special theory of
    relativity prefers the first definition, i.e., sets ε equal to 1/2, it
    does so on the ground that this definition leads to simpler relations.”

    We are then left with an ostensibly simplifying assumption that leads
    ultimately to a more complex and sometimes self-contradictory and
    empirically-challenged system than other alternative assumptions,
    including the more intuitive notion that the speed of light does in fact
    change based on the speed of the observer.

    In order to explain various phenomena, it is in my view more
    parsimonious to adopt the LR version of relativity and an ε different
    than ½. This approach accords with the various lines of evidence (CMB frame of reference,
    large-scale non-homogeneity, quantum collapse, octupole deformation, etc.), as well as
    the non-illusory passage of time. Under this approach, the specific value of ε depends on the speed
    of the moving frame through the ether. C is only invariant with respect to the preferred "ether" frame.

    ReplyDelete
  184. @Amos, Part 3.

    Your write:

    The use of light signals is not essential; we get exactly the same synchronization using mechanical inertia or any other physical process. The equations of every physical law - not just electromagnetism but mechanics as well - take the same form in terms of any system of inertial coordinates. This was shown by the failure of the “ether wind” experiments, which were designed to detect a difference between the (presumed) Galilean relativity of mechanics and the Lorentzian relativity of electomagnetism and optics. The absence of any difference shows that mechanics and electromagnetism (and all other physical laws) are Lorentz invariant, not Galilean invariant, and this has hugely significant physical consequences. You can read about this, and the underlying reason for it, in any good book on relativity (using the true Scotsman definition of “good book”).

    You're begging the question here b/c the discussion we've been having is how to interpret the "ether wind" experiments. Lorentz proposed that it was in fact length contraction of the interferometer arms that led to the apparent null result. Einstein argued that it was the result of an invariant c in all inertial frames. These two explanations are thought to be empirically equivalent (they're actually not, which I'll discuss below, which is why I've said in these discussions with you that they're "thought to be" empirically equivalent, because that is the general view, of course).

    Yes, of course, the Lorentz transformations form the basis for Lorentz invariance, but again: Lorentz argued NOT that c was invariant, but that the apparent invariance of c was actually due to length contraction, resulting from interaction with the preferred frame, not an actually invariant c.

    And in terms of your suggestion that any kind of synchronization technique yields the same speed of light figures, I don't believe this is correct. Are you referring to Will 1992 "Clock Synchronization and Isotropy..."? If so, see p. 45 and its statement about the result being independent of the clock synchronization technique BUT also requiring an assumption about length contraction, so the assumption chosen again dictates the outcome, which is the same as my earlier point about being unable to measure 1-way or 2-way speed of light without using some convention/assumption about synchronization.

    You write: "It’s an empirical fact that the speed or light in vacuum is c in terms of any system of inertial coordinates, meaning coordinates in which the equations of Newtonian mechanics hold good in the low speed limit."

    But this is again begging the question. I also refer you to my question to Rovelli about Einstein's own statements regarding the tautological nature of inertial frames in his 1938 book The Evolution of Physics. You can get it on Kindle if you don't have it in your library.

    More to come later...

    ReplyDelete
  185. Doctor Boktor, thanks for trying to explain to me how entropy drives the direction of time. Unfortunately, I still don't grasp how the tendency of entropy to increase for all processes drives time rather than being a consequence (rather than a cause) of the happenings we count as time.

    For example, suppose we record some event at the cost of some entropy and that entropy randomly reverses itself as is statistically possible so that the universe has less entropy after the recording than it had before. The recording will not necessarily disappear, will it? (There are plenty of other places to recoup the entropy from.) Or are you saying that at different places in the universe time (counts of happenings) are proceeding in a different direction locally as local entropy increases or decreases? Or that time is both increasing and decreasing locally (mostly increasing) as entropy increases and decreases, that is, some counts advance and others decrease?

    That also raises the question whether if counts of happenings are decreasing (locally) because entropy has decreased and erased the records of previous counts, does that really mean that time is going backwards? As Bishop Berkeley might say, if trees fall in the forest but nobody counts them, did they not really fall?

    On the laws of nature and time-symmetry, I won't repeat myself and incur another fine, but as an engineer trying to analyse times to failure, heat transfer, vibration cycles, and other things involving time, the laws of nature I use do distinguish between past and future. (Fourier's Law of Heat Conduction, for example, explicitly, but even Newton's Laws, implicitly.) Take the famous case of reversing the video of pool balls colliding. If you run the video long enough I can tell which way the past is, by the effect of friction. (If it were a frictionless surface, the cases could be indistinguishable but only if either direction was physically possible; to my mind this does not prove that nature and its laws don't care about the direction.)

    Digression: I have programmed simulations of pool for video games, and in my experience those games won't work well without friction. (Some randomness is good also.) I tend to extrapolate such experiences to the point of thinking the universe needs friction and some randomness--if you want an interesting universe.

    Well, you have read and pondered "The Arrow of Time" and I haven't, so maybe my confusion is not worth your time. Sometimes the teacher just has to say, "Think about it some more." (And read the ___ manual.)

    ReplyDelete
  186. @JimV

    Giving a full explanation would take a lot of time and stray too far off-topic for this comment section. I will try to give you some brief answers:

    The other time-dependent processes you mentioned are all special cases of entropy.
    For example, take friction. Friction is transfer of impuls from objects into heat. But what is heat? The impulses of elementary particles.
    A full classical description involves not just the balls, but the speed and location of every particle in the room. And then you see that nothing really stops moving, the particles are moving and colliding without any friction. If you take the final situation (balls stopped moving) and exactly reverse the speed of every particle, you will see the whole situation play backwards. The particles will collide in such a way that the balls start moving, while the table cools down (the opposite of friction happens).

    In terms of entropy, you can explain friction as the fact that particles are far more likely to distribute their impulse roughly evenly, rather than having a single 'object', a big lump of particles, with higher impuls than the rest.

    Hope that helps. If you would like to know more, I can recommend "The Refrigerator and the Universe" by Goldstein & Goldstein.

    ReplyDelete
  187. Doctor B, that is a good point about friction being an aspect of entropy. I accept that, but in the frictionless state as I said I don't see how the existence of two physically-possible scenarios, one more likely than the other, proves that the counting of events (time) depends on which is more probable. If it doesn't, then all the collision laws (mechanics) implicitly contain a direction of time in the form of the time-derivative which must be used for a forward-time calculation. (Or more generally, for a calculation starting from an initial condition and proceeding into unknown territory, but, by my definition, the future is always unknown territory.)

    Summary: it is understandable to me that as more events happen, entropy is statistically likely to increase, so given two videos which in which the same events happen in a different order, the one in which entropy increases is likely to be the correct order; but it seems to me this could be a correlation rather than a cause.

    (As you said, this is getting off-topic, so I will pay both our fines for that.)

    ReplyDelete
  188. ...as discussed above, there is indeed a substantial amount of data that can empirically distinguish these two interpretations – but this state of affairs is not widely acknowledged yet.

    No, the Lorentzian and Einsteinian interpretations do not differ empirically, because all the equations for making empirical predictions are identical. (As explained before, contrary to your beliefs, time dilation is an empirical fact, and neither the CMBR nor quantum entanglement imply any violation of special relativity.)

    Einstein assumed that the speed of light was constant no matter the speed of the observer, as an explanation of the Michelson-Morley data. We can assume otherwise, however, particularly considering that no other speeds in the universe behave this way. That is, all other speeds do in fact change based on the speed of the observer.

    First, Michelson-Morley (as one of the null ether drift results) was cited by Einstein in support of the relativity principle, not the light speed principle. Second, the invariance of the speed of light in terms of inertial coordinates (in which the equations of Newtonian mechanics hold good in the low speed limit) is an empirical fact, and we cannot assume otherwise. Third, every speed behaves this way, in the sense that, since inertial coordinate systems are related by Lorentz transformations, if S’ is moving with speed v in terms of S, and if an object is moving at speed u in terms of S’, then that object is moving at speed (v+u)/(1+vu/c^2) in terms of S. This applies to all speeds.

    This technique only works for determining the one-way speed of light if one assumes that the speed out is the same as the speed back…

    It is a measurement of the two-way speed (with distances measured by rulers at rest in the frame), and to infer the one-way speed we need some other stipulation, such as to stipulate an inertial coordinate system. These are the coordinates we would naturally use when in any given state of motion, since these are the coordinates in which Newton’s equations of mechanics hold good in the low speed limit (which is sufficient to define the coordinates). In terms of such coordinates, the one-way speed of light (in vacuum) is unambiguously equal to c. (Values of Reichenbach’s “ε” other than 1/2 simply correspond to coordinates with non-isotropic inertia.)

    any other speeding object would not have the same speed in both directions because of other forces impacting the object differently based on its direction, such as air speed.

    Air speed? If there is some substance like air moving between source and receiver, then even light in inertial coordinates would have a different speed out and back. The invariant speed of light refers to propagation in vacuum. Light propagation and mechanical inertia are both isotropic in terms of inertial coordinates.

    And if the observer was herself moving during this experiment, any other moving object would indeed show a different speed out and back.

    Regardless of what system of inertial coordinates we use, and regardless of whether the object is a pulse of light or a material particle, the composition of speeds is always given by (u+v)/(1+uv/c^2). This is a direct algebraic consequence of inertial coordinate systems being related by Lorentz transformations.

    I also refer you to my question about Einstein's own statements regarding the tautological nature of inertial frames in his 1938 book The Evolution of Physics.

    That book was written by Infeld, and Einstein wasn’t happy with it, so please don’t quote it as Einstein’s statements. Einstein actually wrote: “With the given physical interpretation of coordinates and time, this is by no means merely a conventional step, but implies certain hypotheses concerning the actual behavior of moving measuring rods and clocks, which can be experimentally validated or disproved”.

    ReplyDelete
  189. Thanks anyway, JimV, and I agree with most of what you said, and also with most of what you said at 11:50 AM, 14th September.

    ReplyDelete
  190. sean s,

    To clarify what I am saying: I see the universe as a system. If you attempt a systems analysis of the universe, it is clear that there are aspects of reality/the universe which are not representable as numbers and mathematical equations. The aspects that ARE representable as mathematical equations, algorithms and numbers is the fundamental-level relationship infrastructure of the system. What is NOT representable in this way are (e.g.) the aspects that create the relationships and the aspects that know about the relationships. The same is true for any computer system created by human beings (that are external to the system created); but in the context of the universe there is nothing external to the system: the creative, knowledgeable and subjective (i.e. frame of reference) aspects can only be matter itself (in certain configurations), and obviously these creative, knowledgeable and subjective aspects were around well before living things came on the scene.

    In a computer system, the algorithms (and the electricity that powers the system) are the direct cause of a changing stream of particular numbers that are output by the system, in one direction only. But in our actual universe, the question is “why are the numbers changing?” And given what we know about the universe-system, the answer seems to be that new relationships (e.g. what we might represent as new number assignments to variables) are created in quantum events; and these numbers have a consequential, flow-on effect to the numbers for the other variables due to the law of nature relationships between variables. It is this one-way assignment of (what we would represent as) new numbers in quantum events that is the equivalent what happens in computer systems: quantum events are driving “change” in the universe-system. Note that in quantum events, energy is not necessarily conserved.

    Another thing to mention about systems is that the basic rules that drive the system (e.g. what we would represent as law of nature relationships and number assignments to variables) don’t “emerge” from the system: they are created i.e. added to the system, as if by an external hand. Any new rules, that seem to “emerge” from the statistics of large numbers of nanoscale and sub-nanoscale events in the universe, are not actually the rules that drive the system. The rules that drive the system never “emerge” from the system.

    ReplyDelete
  191. @Amos, part 4.

    You state:

    The two-way speed of light (with the distance measured by stationary rulers) can be measured with a single clock, and doesn’t involve any distant synchronization. But even after we’ve established that the two-way speed is always c, there remains the question of the one-way speed, since there are self-consistent coordinate systems in which the one-way speed varies even though the two-way speed is always c. However, in terms of inertial coordinate systems, operationally defined as above, the one-way speed of light is empirically c.

    It seems you're missing a key point here: the epsilon factor referred to above, which is assumed to be 1/2 in SR, indicating the same speed out and back. That's an untestable assumption and it would be more natural (since all other physical phenomena behave this way) for epsilon to be some value other than 1/2, indicating interaction with whatever medium the physical phenomenon at issue is passing through. It's only if we assume that light is completely unaffected by other physical media, whether it's air, fields, ether, what have you, that epsilon is actually 1/2. And that's the point of Einstein synchronization, per the quote I posted from Einstein: assuming epsilon is 1/2 is just a simplifying assumption, which Einstein made in order to have a workable synchronization technique and a workable definition of time. But we err when we assume that these assumptions necessarily lead to ontological conclusions.

    You state:

    Hold on, your whole position is based on the denial of the physical reality of time dilation, i.e., you claim that actual physical time dilation is a false prediction of special relativity but not of neo-Lorentzian relativity, and yet now you are invoking empirical equivalence between those two interpetations. You can’t have it both ways. Neo-Lorentzian relativity and special relativity are just two interpretations of the same theory, with all the same physical phenomena and predictions, so your claims are self-contradictory. Also, you keep alluding to “problem-inducing consequences” of special relativity, but you haven’t identified any. The physical phenomena of neo-LorentzIan relativity are identical to those of special relativity, so it is no more or less “weird and mind-bending”.

    There is no contradiction. As I stated above, experiments that involve a single clock and purport to find time dilation don't actually find such, when we look deeply (either through data fudging or other experimental issues), and experiments that involve two separated clocks find time dilation based on the tautological assumptions of Einstein synchronization or slow clock synchronization. So my point is that there is no actual time dilation. The Lorentz transformations show "time dilation" in separated clock experiments, but there is not a physical effect of actual time dilation. It's just a coordinate effect that disappears when we remove tautological assumptions or synchronization techniques.

    ReplyDelete
  192. @Amos, Part 5

    You state:

    Again, your beliefs are based on denial of the facts, e.g., the different elapsed proper time along different paths between two events… not to mention E=mc^2 and all the other consequences of the inertia of energy. Lorentz invariance is not a mathematical fiction nor an artifact of a peculiar choice of coordinate systems. It represents real physical phenomena, and it can be (and has been) thoroughly tested to high precision.

    Not really. See above. Here's another example: I had a recent dialogue with a CalTech geophysicist who studies Earth's gravity fields. In discussing one of the recent experiments that seems to find evidence of time dilation based on lab clocks at different heights (separated by as little as half a meter), he stated to me in our dialogue that "Einstein is not to be questioned." Rather, time dilation is taken as a given. So these experiments, which have been cited frequently as evidence of time dilation, actually just assumed time dilation and are looking instead to use time dilation to measure differences in terrestrial gravity fields. That would be fine if it were stated in the papers, but it's not. Rather, it's implicit and many readers assume instead that the papers are actually testing time dilation. The same geophysicist acknowledged to me that these experiments can't even test time dilation in this type of experiment b/c the clocks weren't accurate enough until very recently.

    ReplyDelete
  193. @JimV and doctor Boktor, on the notion that entropy results in time as we know it there are all sorts of problems here. Carroll actually discusses this idea in depth in his book From Eternity to Here and he argues (to sum up a detailed book very briefly) that we experience time (the past is known and the future isn't, with a now sandwiched between the two) as a result of a very large and mysterious free lunch of order represented by the Big Bang. That initial universal clock winding isn't explained (Carroll acknowledges this), but our particular experience of time is all due to that very ordered state slowly unwinding.

    A few big problems:

    1) Any theory that requires such a large free lunch at its beginning should prompt serious inquiry into alternative approaches

    2) Suggesting that a steady increase in entropy/disorder results in the apparent passage of time and the psychological explanation of time ignores the fact that assessing the direction of entropy requires measurements in time. So how could measurements of phenomena in time be the physical/causal mechanism of time? This is another tautology (there are many in modern physics unfortunately).

    3) Entropy is not actually a universal process. Only very simple processes like molecules in air in an enclosed box show a tendency toward maximum disorder over time. At larger scales, gravity leads to an increase in order through clumping, star formation, galaxy formation, and ultimately life formation. If the universe sees a tendency for life to spread outward (even if we are the only planet with life on it) we will surely lead over the next few billion years to negentropic tendencies as we appropriate more and more matter for computing power and of course create more and more life forms (human, animal, etc.) on innumerable planets. So entropy is variable at different periods of the universe and at different physical scales.

    The better approach is to accept that there seems to be an innate universal clock cycle (Whitehead called this the creative advance into novelty), which we can call change or time, regardless of how we choose to measure time. Any attempt to craft some subsidiary process as a causal mechanism for time ends up in the same problem of trying to explain the emergence of time in time, yet another tautology.

    ReplyDelete
  194. Amos,

    It's nice to say somebody would had found an qualitative support for something when he only shows that he is able to change his calculations as long until his result fits his expectations. ;-)



    I would prefere if this probable bogus experiment is wiped out from the collective memory as allegedly strong indicator for time dilation, expecially in respect of kinetic time dilation.



    Note: "hafele keating relativity" returns nearly 30,000 hits e.g. with bing.



    Diffferent paths:

    There is a methode of synchronisation of distant clocks: the slow clock motion methode. You should easily see that any different path of a slow motion (without passing different gravitational potentials) is identic concerning time dilation. I think you had made some simplifications.


    Two kinds of presumed time dilation effects:

    “Given the large drift rates, numerous rate shifts, and dicey adjustments, their results certainly wouldn’t qualify as a “five sigma detection", but I think they could legitimately claim to have found at least qualitative support for the predicted time dilation”


    You claim that the experiment can at least hold as a qualitative support of time dilation. But of which kind of time dilation? The effect of gravitational time dilation is expected in a about ten time larger effect than the effect of "kinetic time dilation". So what will this all mean about the "qualitative support" of the effect of "kinetic time dilation" which is much more issue in question, I think.

    ReplyDelete
  195. sean s,

    [ss]That is true until you claim the lack of evidence is significant. Significance is your claim then, and it becomes something you need to establish. Which you cannot.

    In science, a lack of evidence for the physically meaningful existence of a thing that is claimed to have such existence is always significant. It is how science avoids pointless arguments with fundamentalist believers (either secular or religious) - by stating that since the matter lies beyond the realm of the physical, it therefore lies beyond the realm of science.

    [ss]You claim the lack of empirical evidence is significant; that is wrong because NEITHER OF US can specify any experiment or empirical result that we should observe. In this kind of case, null results has no real significance.

    You are free to believe in any and all things you might conjure in your imagination especially if they are defined as having no empirically observable characteristics. Such beliefs, however, have no claim on science because, as we agreed, science is based on empiricism. Which brings me to this:

    [ss}But since WE AGREE that science is not limited to direct observables or empirical evidence, then we can go on with less rigorous but still valid information.

    I have never agreed to such a thing. You have an annoying habit of changing things that are plainly said to suit your own proclivities. The only thing I agreed to was that empiricism is not the only basis of science. I did not in any way mean to imply that it is an optional basis. No empiricism, no science.

    But what then are the other bases of science? There is only one, logic (broadly inclusive of math and other forms thereof). Empiricism and logic are the complementary probes with which we humans have, with some success, investigated the nature of physical reality. Science can only be done when both probes are used in complementary fashion however. Without logic empiricism just amasses a jumble of disconnected facts. Without the constraint of empiricism, logic emits only insubstantial and incoherent effluvia from the dim but florid recesses of the human imagination.

    [ss]It is a fact of reality that we experience time; but we do not experience temperature. That fact is important, meaningful and worth consideration. That is substantive evidence. It’s not what you’re looking for, but it is substantive evidence.

    You seem to believe that time is derivative of change events, but change events cannot happen unless time already exists.


    First of all, we do not experience time in the exact same way we do not experience temperature. Both are simply human concepts that allude to the underlying physical phenomena we do experience.

    The arguments you present for the necessary existence of time are, again, circular. They make no sense as a logical argument because they assume that time exists, as something other than a human concept, to begin with.

    You insist that time must exist otherwise change wouldn't happen. But it is a demonstrable fact that change happens. And you admit that there is no scientific, empirical evidence that demonstrates the existence of time. You say that lack of evidence doesn't matter. All you have to offer, then, are your circular arguments which assume what you wish to conclude - that time exists. Circular argumentation is a logical error.

    ReplyDelete
  196. Doctor B, that is a good point about friction being an aspect of entropy. I accept that, but in the frictionless state as I said I don't see how the existence of two physically-possible scenarios, one more likely than the other, proves that the counting of events (time) depends on which is more probable. If it doesn't, then all the collision laws (mechanics) implicitly contain a direction of time in the form of the time-derivative which must be used for a forward-time calculation.

    The basic laws of classical mechanics (that describe collisions and the movement of particles, with or without forces like gravity and electromagnetism) all remain unchanged when you reverse the arrow of time. They do not contain any special property which is only true when time is moving forward. When you use a time-derivatives in a forward-time calculation, you will also have to use it when calculating backwards.

    A few examples as illustration. Take the symmetric parabola that you get from an object being thrown up, when air friction can be ignored. Or the elliptic orbit of a planet around the sun. Or one of the frictionless game simulations you have done yourself. ;)

    Now record one of the events above on film and play the movie backwards. What do you see? You see the same movie, mirror imaged.

    And not only classical mechanics is time-reversible. That also holds for special relativity (regardless whether you choose the Lorentz or Einstein interpretation), it holds for GR and even for QM.

    For the final time I will refer to another source and then I will leave this discussion for what it is. Jim, if you have more questions, please consult the wiki page below. It explains it all much better and more thoroughly than I ever could.

    https://en.wikipedia.org/wiki/T-symmetry

    ReplyDelete
  197. Bud Rap,

    Again, I agree with your arguments. However, I can use the exact same arguments to argue against the substantial true existence of any thing whatsoever.

    I ask again, do you believe that any thing whatsoever exists in a substantial way that you deny for time?

    Cheers,
    Atreat

    ReplyDelete
  198. Tam Hunt wrote: …the epsilon factor referred to above, which is assumed to be 1/2 in SR, indicating the same speed out and back. That's an untestable assumption…

    It isn’t an assumption, it’s a stipulation, representing the choice to use inertial coordinates, i.e., coordinates in which the equations of Newtonian mechanics hold good in the low speed limit. Naturally we’re free to use a different value of epsilon, but then we aren’t using inertial coordinates. The speed of light can have any value we like, depending on what coordinates we choose, but the speed of light in terms of standard inertial coordinates is empirically equal to c.

    There is no contradiction…. my point is that there is no actual time dilation.

    The contradiction is that you claim to believe in Lorentzian relativity and yet you deny the physical reality of time dilation, which is a prediction of Lorentzian relativity. The empirical fact that inertial coordinate systems are related by Lorentz transformations (which is the foundation of Lorentzian relativity) implies the physical reality of time dilation. So your beliefs are self-contradictory and empirically falsified.

    It's just a coordinate effect that disappears when we remove tautological assumptions or synchronization techniques.

    No, you’re failing to distinguish between passive and active transformations. Passive transformations are just coordinate effects, but active transformations are real physical effects. We choose our passive transformations to match the physical active transformations. Using Lorentzian relativity, work out the elapsed time on an ideal clock moving inertially from t=0, x=0 to t=10, x=5 and the elapsed time on an ideal clock moving inertially from t=10,x=5 to t=20,x=0 (all times in units of seconds and distances in light-seconds). How does this compare with the elapsed time for an ideal clock moving inertially from 0,0 to 20,0?

    …recent experiments that seems to find evidence of time dilation based on lab clocks at different heights (separated by as little as half a meter)… these experiments, which have been cited frequently as evidence of time dilation, actually just assumed time dilation and are looking instead to use time dilation to measure differences in terrestrial gravity fields.

    No, the experiments involved elevating the clock by a measured distance and noting the change in the rate of the clock in agreement with the prediction of general relativity. They also set the clock into motion at constant elevation and demonstrated the effect of the special relativistic time dilation due purely to motion. Again, you are simply in denial of the empirical evidence.

    The same geophysicist acknowledged to me that these experiments can't even test time dilation in this type of experiment b/c the clocks weren't accurate enough until very recently.

    Is there a typo in that sentence? Surely you meant to say that the time dilation for such small differences in elevation (half a meter) or such low speeds (20 miles per hour) could never previously have been detected because sufficiently precise clocks were not available until recently.

    Again, Lorentzian and Einsteinian interpretations of special relativity are empirically equivalent, and the real physical effects have been abundantly demonstrated, so your beliefs are self-contradictory and empirically falsified.

    ReplyDelete
  199. Doctor Boktor, I am sorry that I seem to be annoying you, but perhaps I have not been expressing my point clearly enough because I am not seeing how you are addressing it.

    Again/also, I accept that if one does not add the assumption that time-derivatives must be interpreted taking into account that events happen in the order of past-to-future, then natural laws appear to be time-symmetric. What I don't see as necessary is that entropy is forcing that order. Take your time-symmetric parabola example, use the possible-reverse order, and add friction. As I see it, entropy would also increase if that reverse order were the actual order of events. It is the fact that new events statistically add chances for entropy to increase that makes the arrow of entropy align with the arrow of time, to a high correlation factor--or at least that seems quite possible to me, and I am not aware of a proof to the contrary. If one flips a fair coin a thousand times, the chances that the ratio will be close to 50% heads are greater than if you only flipped the coin ten times. Doesn't this statistical fact explain the correlation satisfactorily?

    Maybe that is all you and Dr. Carroll are saying, but I have the impression that a causal relation is being suggested, which I don't understand the necessity of.

    (I am repeating myself with different words, so I guess a fine is warranted.)

    ReplyDelete
  200. manyoso,

    Again, I agree with your arguments. However, I can use the exact same arguments to argue against the substantial true existence of any thing whatsoever.

    I ask again, do you believe that any thing whatsoever exists in a substantial way that you deny for time?


    In all honesty, I do not understand what you are asking me. Do you think that my arguments against a substantival time could be used to call into question the existence of the moon (or any other observable)?

    ReplyDelete

COMMENTS ON THIS BLOG ARE PERMANENTLY CLOSED. You can join the discussion on Patreon.

Note: Only a member of this blog may post a comment.